texts
sequence
meta
dict
scores
sequence
avg_score
float64
0
0.5
num_sents
int64
5
5
[ "Q:\n\nHow can I sanitize a string while maintaining all non-English alphabet support\n\nGenerally, I would strip all characters that are not English using something like :\n$file = filter_var($file, FILTER_SANITIZE_STRING, FILTER_FLAG_STRIP_LOW | FILTER_FLAG_STRIP_HIGH );\n\nhowever, I am tired of not providing support for user input from other languages which may be in the form of an uploaded file (the filename may be in Cyrillic or Chinese, or Arabic, etc) or a form field, or even content from a WYSIWYG.", "\nThe examples for sanitizing data with regards to this, come in one of two forms\n\nThose that strip all chars which are non-English\nThose that convert all chars which are non-English to English letter substitutes.", "\n\nThe problem with this practice, is that you end up with a broken framework that pretends it supports multiple languages however it really doesn't aside from maybe displaying labels or content to them in their language.", "\nThere are a number of attacks which take advantage of unicode/utf-8/utf-16/etc support passing null bytes and so on, so it is clear that not sanitizing the data is not an option.", "\nIs there any way to clean up a variable from arbitrary commands while maintaining the full alphabets/chars of these other languages, but stripping out (in a generic manner) all possible non-printable chars, chars that have nulls in them as part of the char, and other such exploits while maintaining the integrity of the actual characters the user input ? ", " The above command is perfect and does everything exactly as it should, however it would be super cool if there were a way to expand that to allow support for all languages.", "\n\nA:\n\nNull bytes are not(!) ", "UTF-8, so assuming you use UTF-8 internally, all you need to do is to verify that the passed variables are UTF-8. ", "There's no need to support UTF-16, for example, because you as author of the according API or form define the correct encoding and you can limit yourself to UTF-8. ", "Further, \"unicode\" is also not an encoding you need to support, simply because it is not an encoding. ", "Rather, Unicode is a standard and the UTF encodings are part of it.", "\nNow, back to PHP, the function you are looking for is mb_check_encoding(). ", "Error handling is simple, if any parameter doesn't pass that test, you reply with a \"bad request\" response. ", "No need to try to guess what the user might have wanted.", "\nWhile the question doesn't specifically ask this, here are some examples and how they should be handled on input:\n\nnon-UTF-8 bytes: Reject with 400 (\"bad request\").", "\nstrings containing path elements (like ../): Accept.", "\nfilename (not file path) containing path elements (like ../): Reject with 400.", "\nfilenames شعار.jpg, 标志.png or логотип.png: Accept.", "\nfilename foo <0> bar.jpg: Accept.", "\nnumber abc: Reject with 400.", "\nnumber 1234: Accept.", "\n\nHere's how to handle them for different outputs:\n\nnon-UTF-8 bytes: Can't happen, they were rejected before.", "\nfilename containing path elements: Can't happen, they were rejected before.", "\nfilenames شعار.jpg, 标志.png or логотип.png in HTML: Use verbatim if the HTML encoding is UTF-8, replace as HTML entities when using default ISO8859-1.", "\nfilenames شعار.jpg, 标志.png or логотип.png in Bash: Use verbatim, assuming the filesystem's encoding is UTF-8.", "\nfilenames شعار.jpg, 标志.png or логотип.png in SQL: Probably just quote, depends on the driver, DB, tables etc. ", "Consult the manual.", "\nfilename foo <0> bar.jpg in HTML: Escape as \"foo <0> bar.jpeg\". ", "Maybe use \" \" for the spaces.", "\nfilename foo <0> bar.jpg in Bash: Quote or escape \" \", \"<\" and \">\" with backslashes.", "\nfilename foo <0> bar.jpg in SQL: Just quote.", "\nnumber abc: Can't happen, they were rejected before.", "\nnumber 1234 in HTML: Use verbatim.", "\nnumber 1234 in Bash: Use verbatim (not sure).", "\nnumber 1234 in SQL: Use verbatim.", "\n\nThe general procedure should be:\n\nDefine your internal types (string, filename, number) and reject anything that doesn't match. ", "These types create constraints (filename doesn't include path elements) and offer guarantees (filename can be appended to a directory to form a filename inside that directory).", "\nUse a template library (Moustache comes to mind) for HTML.", "\nUse a DB wrapper library (PDO, Propel, Doctrine) for SQL.", "\nEscape shell parameters. ", "I'm not sure which way to go here, but I'm sure you will find proper ways.", "\n\nEscaping is not a defined procedure but a family of procedures. ", "The actual escaping algorithm used depends on the target context. ", "Other than what you wrote (\"escaping will also screw up the names\"), the actual opposite should be the case! ", "Basically, it makes sure that a string containing a less-than sign in XML remains a string containing a less-than sign and doesn't turn into a malformed XML snippet. ", "In order to achieve that, escaping converts strings to prevent any character that is normally not interpreted as just text from getting its normal interpretation, like the space character in the shell.", "\n\n" ]
{ "pile_set_name": "StackExchange" }
[ 0, 0, 0, 0, 0, 0, 0, 0, 0.006097560975609756, 0, 0.014925373134328358, 0.013157894736842105, 0, 0, 0, 0, 0, 0.0196078431372549, 0, 0.034482758620689655, 0, 0, 0, 0.006666666666666667, 0.00909090909090909, 0.009009009009009009, 0, 0, 0, 0.011764705882352941, 0, 0.018867924528301886, 0, 0, 0, 0, 0, 0.01694915254237288, 0.034482758620689655, 0, 0, 0, 0, 0, 0, 0, 0 ]
0.004151
5
[ "Q:\n\nDisplay alert based on characters in placeholder in jQuery\n\nWhat I'm trying to achieve:\n1) user hovers over placeholder\n2) if user typed more than 3 characters in placeholder and mouse leaves of placeholder > display #alert\n3) if user removes characters in placeholder to less than 3 > hide alert\n\njQuery('#mydiv').after('<span id=\"alert\">alert</span>');\r\njQuery('#myinput').mouseleave(function() {\r\n jQuery('#alert').toggle(this.value !", "== '');\r\n});\n#alert {\r\n display: block;\r\n display: none;\r\n}\n<script src=\"https://cdnjs.cloudflare.com/ajax/libs/jquery/3.2.1/jquery.min.js\"></script>\r\n\r\n<div id=\"mydiv\">\r\n <input id=\"myinput\" placeholder=\"\">\r\n</div>\n\nA:\n\ncheck the length of the input, then display the alert\non input if length < 3, hide alert.", "\n\n$('#mydiv').after('<span id=\"alert\">alert</span>');\r\n\r\n$('#myinput').mouseleave(function() {\r\n if(this.value.length >= 3) $('#alert').show()\r\n});\r\n\r\n$('#myinput').on('input', function(){\r\n if(this.value.length < 3) $('#alert').hide();\r\n})\n#alert {\r\n display: block;\r\n display: none;\r\n}\n<script src=\"https://cdnjs.cloudflare.com/ajax/libs/jquery/3.2.1/jquery.min.js\"></script>\r\n\r\n<div id=\"mydiv\">\r\n <input id=\"myinput\" placeholder=\"\">\r\n</div>\n\n" ]
{ "pile_set_name": "StackExchange" }
[ 0.0022522522522522522, 0.006269592476489028, 0.004366812227074236 ]
0.004296
5
[ "No. ", "15 Bruins Too Much for Gauchos\n\nLOS ANGELES -\nThe No. ", "15 UCLA women's basketball team proved to have a much better full-court defense than UC Santa Barbara's last opponent.", "\n\nThe Gauchos, despite losing to Pepperdine on Friday, had little trouble with the Waves' full-court, pressure defense however on Sunday at Pauley Pavilion it was a different story.", "\n\nThe Bruins forced 23 turnovers by the Gauchos and shot 43.8 percent from the floor to hand UCSB its second defeat, 66-52.", "\n\nAn 18-0 run by UCLA early in the second half put the Gauchos in a 55-26 hole that they couldn't recover from.", "\n\n\"I told their head coach Nikki Caldwell they have Final Four potential,\" UCSB head coach Lindsay Gottlieb said. \"", "They're really good. ", "They have a lot of weapons. ", "I thought there were times where we competed against them. ", "But there were other times we let them do what they're really good at.\"", "\n\nEmilie Johnson led UCSB in scoring for the second straight game with 14 points as she was the lone Gauchos to reach double figures. ", "Underwood had nine points and eight rebounds for UCSB, which shot 37.7 percent from the floor.", "\n\nKelsey Adrian added eight points while Nesbit had five to go with three assists and Okolie had four points and four boards, all on the offensive end.", "\n\nThe Gauchos play their next three games at home, beginning with a matinee 1 p.m. start on Thursday vs. Portland State.", "\n\nFans can listen to the game on AM 990 KTMS in Santa Barbara or streaming live online by clicking here.", "\n\nTickets for UCSB home games are available by calling (805) 893-UCSB or by clicking here.", "\n\nThe Intercollegiate Athletics Ticket Office has two parking spaces adjacent to the building and is now open on Saturdays from 10 a.m. to 1 p.m., making it easier than ever for fans to purchase tickets. ", "The two parking spots, designated specifically for the Ticket Office, are also available during weekday hours, which are 9 a.m. to 4 p.m. Monday through Friday.", "\n\nThe Gaucho Fund, launched as part of the Dare to Be Great strategic plan, is UCSB Athletics' annual giving program to provide vital funding support for scholarships, academic success and leadership development for Gaucho student-athletes. ", "The Gaucho Fund is the anchor to the Dare to Be Great campaign." ]
{ "pile_set_name": "Pile-CC" }
[ 0, 0.018518518518518517, 0.00847457627118644, 0.0055248618784530384, 0.016260162601626018, 0.018018018018018018, 0.02608695652173913, 0, 0, 0, 0, 0.014925373134328358, 0.010638297872340425, 0.013245033112582781, 0.016666666666666666, 0, 0.011111111111111112, 0.004901960784313725, 0.00625, 0.012448132780082987, 0.015873015873015872 ]
0.009473
5
[ "Fort Dinwiddie\n\nFort Dinwiddie (1755–1789) was a base for the Virginia Militia during the French and Indian War and Revolutionary War. ", "It was located on the Jackson River, five miles west of Warm Springs, Virginia, in present-day Bath County.", "\n\nHistory\nThe fort was first built in 1755 around the house of William Warwick during the French and Indian War. ", "It was named for the lieutenant governor of colonial Virginia, Robert Dinwiddie. ", "Initially the fort was under the command of Captain Andrew Lewis who was relieved by Captain Peter Hog on September 21, 1755. ", "Colonel George Washington inspected the fort on September 24, 1755.", "\n\nThe fort was garrisoned by 60 to 100 men in 1756. ", "The fort featured a log-covered passageway that ran from a corner blockhouse to a spring within the fort.", "\n\nThe stockaded structure became one of a chain of forts, 15 to 30 miles apart, established in the mid-1700s to guard the Virginia frontier against the Native Americans of the Ohio River Valley. ", "George Washington, then a colonel and in charge of the Virginia Militia’s fort project, wrote to Captain Hog of the Virginia Regiment, July 21, 1756: \"As the Assembly has voted a chain of Forts to be built on the Frontiers, the Governor has ordered out the militia of Augusta to assist you in erecting them ... to the Southward of Fort Dinwiddie, extending the Line towards Mayo River as directed by the Assembly.\" ", "Captain Hog was to make use of his own company of men and of the militia he might raise, \"except about 30 private wh. ", "you are to leave under the command of Lt. ", "Bullett, at Fort Dinwiddie. . .\"", "\n\nGeorge Washington visited the fort again in October 1756, while the fort was under the temporary command of Lieutenant Bullet.", "\n\nThe fort was used primarily as a muster point but was laid siege upon by a group of Delaware and Shawnee for six hours in mid-June 1764.", "\n\nThe fort was abandoned in 1789. ", "The site was confirmed by a team of archaeologists from the Virginia State Library in April 1971.", "\n\nFort Dinwiddie was also known as Warwick's Fort, Hogg's Fort and Byrd's Fort.", "\n\nA historical marker for the fort stands at the intersection of Hwy 39 and Dinwiddie Trail in Bath County, Virginia.", "\n\nReferences \n\nDinwiddie\nDinwiddie\nDinwiddie\nCategory:1750s in Virginia" ]
{ "pile_set_name": "Wikipedia (en)" }
[ 0.014814814814814815, 0, 0.008849557522123894, 0.012345679012345678, 0.015873015873015872, 0.014925373134328358, 0, 0, 0, 0.014457831325301205, 0.00847457627118644, 0, 0.03125, 0.015625, 0.007246376811594203, 0, 0.010309278350515464, 0.0379746835443038, 0.008547008547008548, 0 ]
0.010035
5
[ " SYSINIT is a module linked behind the OEM bios. ", " It takes\r\nover the system initialization after the OEM bios has\r\nperformed any initialization it needs to do. ", " Control is\r\ntransfered with a long jump to the external variable SYSINIT\r\n\r\n\r\n The OEM has the following variables declared external:\r\n\r\n CURRENT_DOS_LOCATION WORD\r\n\r\nThis word contains the segment number of the DOS before it\r\nis relocated. ", " The OEM bios must set this value.", "\r\n\r\n FINAL_DOS_LOCATION WORD\r\n\r\nThis word contains the segment number of the DOS after SYSINIT\r\nmoves it. ", " The OEM bios must set this value.", "\r\n\r\n DEVICE_LIST DWORD\r\n\r\nThis double word pointer points to the linked list of\r\ncharacter and block device drivers. ", " The OEM must set this\r\nvalue.", "\r\n\r\n MEMORY_SIZE WORD\r\n\r\nThis word contains the number of RAM paragraphs. ", " If the\r\nbios doesn't set this variable SYSINIT will automatically\r\ncalculate it. ", " NOTE: systems with PARITY checked memory must\r\nsize memory in the BIOS. ", " SYSINITs method is to write memory\r\nand read it back until it gets a mismatch.", "\r\n\r\n DEFAULT_DRIVE BYTE\r\n\r\nThis is the initial default drive when the system first comes\r\nup. ", " drive a=0, drive b=1, etc. ", " If the bios doesn't set\r\nit then drive a is assumed.", "\r\n\r\n BUFFERS BYTE\r\n\r\nThis is the default number of buffers for the system. ", " This\r\nvalue may be overridden by the user in the CONFIG.SYS file.", "\r\nIt is DBed to 2 in SYSINIT it should be greater than 1.", "\r\n\r\n FILES BYTE\r\n\r\nThis is the default number of files for the system. ", " This\r\nvalue may be overridden by the user in the CONFIG.SYS file.", "\r\nIt is DBed to 8 in SYSINIT, values less than 5 are ignored.", "\r\n\r\n SYSINIT FAR\r\n\r\nThe entry point of the SYSINIT module. ", " OEM BIOS jumps to\r\nthis label at the end of its INIT code.", "\r\n\r\n The OEM has the following variables declared public:\r\n\r\n RE_INIT FAR\r\n\r\nThis is an entry point which allows the BIOS to do some INIT\r\nwork after the DOS is initialized. ", " ALL REGISTERS MUST BE\r\nPRESERVED. ", " On entry DS points to the first available memory\r\n(after the DOS). ", " DS:0 points to a 100H byte program header\r\nprefix which represents the \"program\" currently running.", "\r\nThis program should be thought of as the OEM BIOS and\r\nSYSINIT taken together. ", " This is not a normal program in\r\nthat no memory is allocated to it, it is running in free\r\nmemory.", "\r\nNOTES:\r\n At the time this routine is called SYSINIT occupies the\r\nhighest 10K of memory (\"highest\" is determined by the value\r\nof the MEMORY_SIZE variable), DO NOT DO WRITES THERE.", "\r\n Since this is called AFTER DOS is initialized, you can\r\nmake system calls. ", " This also implies that the code for this\r\nroutine CANNOT be thrown away by use of the\r\nFINAL_DOS_LOCATION since the DOS has already been moved.", "\r\n If you don't want anything done just set this to point\r\nat a FAR RET instruction.", "\r\n\u0000\u0000\u0000\u0000\u0000\u0000\u0000\u0000\u0000\u0000\u0000\u0000\u0000\u0000\u0000\u0000\u0000\u0000\u0000\u0000" ]
{ "pile_set_name": "Github" }
[ 0.018867924528301886, 0, 0.003816793893129771, 0, 0.017543859649122806, 0, 0, 0, 0, 0.011494252873563218, 0.013333333333333334, 0, 0.009259259259259259, 0, 0, 0, 0, 0.017543859649122806, 0, 0, 0.015625, 0, 0.01694915254237288, 0.0049261083743842365, 0.02857142857142857, 0, 0.009615384615384616, 0.011363636363636364, 0, 0.0053475935828877, 0, 0.006211180124223602, 0, 0 ]
0.005602
5
[ "/* SPDX-License-Identifier: GPL-2.0 */\n/*\n * A security identifier table (sidtab) is a hash table\n * of security context structures indexed by SID value.", "\n *\n * Author : Stephen Smalley, <sds@tycho.nsa.gov>\n */\n#ifndef _SS_SIDTAB_H_\n#define _SS_SIDTAB_H_\n\n#include \"context.h\"\n\nstruct sidtab_node {\n\tu32 sid;\t\t/* security identifier */\n\tstruct context context;\t/* security context structure */\n\tstruct sidtab_node *next;\n};\n\n#define SIDTAB_HASH_BITS 7\n#define SIDTAB_HASH_BUCKETS (1 << SIDTAB_HASH_BITS)\n#define SIDTAB_HASH_MASK (SIDTAB_HASH_BUCKETS-1)\n\n#define SIDTAB_SIZE SIDTAB_HASH_BUCKETS\n\nstruct sidtab {\n\tstruct sidtab_node **htable;\n\tunsigned int nel;\t/* number of elements */\n\tunsigned int next_sid;\t/* next SID to allocate */\n\tunsigned char shutdown;\n#define SIDTAB_CACHE_LEN\t3\n\tstruct sidtab_node *cache[SIDTAB_CACHE_LEN];\n\tspinlock_t lock;\n};\n\nint sidtab_init(struct sidtab *s);\nint sidtab_insert(struct sidtab *s, u32 sid, struct context *context);\nstruct context *sidtab_search(struct sidtab *s, u32 sid);\nstruct context *sidtab_search_force(struct sidtab *s, u32 sid);\n\nint sidtab_map(struct sidtab *s,\n\t int (*apply) (u32 sid,\n\t\t\t struct context *context,\n\t\t\t void *args),\n\t void *args);\n\nint sidtab_context_to_sid(struct sidtab *s,\n\t\t\t struct context *context,\n\t\t\t u32 *sid);\n\nvoid sidtab_hash_eval(struct sidtab *h, char *tag);\nvoid sidtab_destroy(struct sidtab *s);\nvoid sidtab_set(struct sidtab *dst, struct sidtab *src);\nvoid sidtab_shutdown(struct sidtab *s);\n\n#endif\t/* _SS_SIDTAB_H_ */\n\n\n" ]
{ "pile_set_name": "Github" }
[ 0.013071895424836602, 0.007246376811594203 ]
0.010159
5
[ "Q:\n\nProving Euler’s Totient Theorem\n\nI’m trying to self-learn some number theory. ", "Currently, I’m trying to prove Euler’s Totient Theorem (that is, $a^{\\varphi(n)} \\equiv 1 \\pmod{n}$ for $a,n\\in\\mathbb{Z}$, where $\\varphi$ denotes Euler’s Totient Function.", "\nUnfortunately, I’m not sure where to start. ", "I’ve tried a few different things, but I haven’t gotten very far. ", "They’re all (seemingly) dead ends. ", "Any hints or good starting points?", "\n\nA:\n\nI'll try to give you a loose proof, assuming you know some things about modular arithmetic, and hopefully you can flesh out the details yourself. ", "I've adapted this mostly from Brilliant.org.", "\nLet's start with modular multiplication. ", "In general, it's not always possible to solve the relation:\n$$\na \\times k \\equiv b\\space (\\textrm{mod}\\space n)\n$$\nfor $k$ when you know $a$, $b$, and $n$. For example consider the situation where $a = 6$, $b=2$, and $n = 15$. Try as you might, you will not be able to find a $k$ between $0$ and $14$ that satisfies that relation.", "\nIn normal arithmetic, what we would do is multiply by the multiplicative inverse of $a$ (i.e. the number $a^{-1}$ such that $a^{-1}\\times a = 1$, or in this case, $a^{-1}\\times a \\equiv 1\\space (\\textrm{mod}\\space n)$) on both sides to find $k$. In modular arithmetic, $a$ has a multiplicative inverse if and only if $a$ is relatively prime to $n$. (Can you see why? ", "Additive cycles in modular arithmetic might be useful.)", "\nNow let's go to repeated multiplication in modular arithmetic. ", "If you start with some integer $m$ and repeatedly multiply it by some factor $k$ some number of times $\\textrm{mod} \\space n$, you will eventually end up in a multiplicative cycle. ", "The cycle will only include the first repeat and the integers that appear after it. ", "There are usually multiple multiplicative cycles for each choice of $n$ and $k$, and all of them together contain each integer from $0$ to $n-1$ once and only once.", "\nAn example for 4 modulo 14\nLet's examine the case of starting at $1$ and repeatedly multiplying by $2$ modulo\n$20$. This generates the sequence $1, 2, 4, 8, 16, 12, 4, 8, 16, 12, 4, \\ldots$\nNotice there is more than one integer that leads to $4$ when multiplying by $2$ in this sequence, namely $2$ and $12$. There is a \"tail\" which includes $1$ and $2$ before coming to our cycle.", "\nHowever, if we are instead multiplying by some integer $a$ that is relatively prime to $20$ i.e. has a multiplicative inverse $\\textrm{mod} \\space 20$, this can't happen. ", "That means that all the multiplicative cycles containing integers relatively prime to $n$ don't have any \"tails\" like the given example or the earlier example of 4 modulo 14. ", "The existence of the inverse implies reversibility which forbids two separate numbers multiplying to the same number. (", "Try it out!)", "\nAlso notice that this reversibility implies the cycle containing $a$ also contains $1$. We are now getting tantalizingly close to Euler's totient theorem.", "\nNotice that all the multiplicative cycles containing integers relatively prime to $n$ are of the same length. ", "If every integer in the cycle contains a multiplicative inverse, they can all be multiplied by some constant to obtain another cycle of the same length. ", "An example for 3 modulo 16\nFinally, we get to the crux of the argument. ", "The length of these cycles is a divisor of $\\phi(n)$ (Can you see why?). ", "If we call this length $l$ and the total number of these cycles $c$, then\n$$\na^l\\equiv a^0\\equiv 1 \\space (\\textrm{mod} \\space n)\n$$\nand\n$$\nl\\times c = \\phi(n)\n$$\nand finally\n$$\na^{\\phi(n)} = a^{l\\times c} = ({a^l})^c \\equiv 1^c (\\textrm{mod} \\space n) \\equiv 1(\\textrm{mod} \\space n)\n$$\n\n" ]
{ "pile_set_name": "StackExchange" }
[ 0, 0.017341040462427744, 0, 0, 0, 0, 0, 0.022727272727272728, 0, 0.0030303030303030303, 0, 0, 0, 0, 0, 0, 0, 0, 0, 0, 0, 0.0064516129032258064, 0, 0, 0, 0, 0 ]
0.001835
5
[ "The discovery, characterization and crystallographically determined binding mode of an FMOC-containing inhibitor of HIV-1 protease.", "\nA pharmacophore derived from the structure of the dithiolane derivative of haloperidol bound in the active site of the HIV-1 protease (HIV-1 PR) has been used to search a three-dimensional database for new inhibitory frameworks. ", "This search identified an FMOC-protected N-tosyl arginine as a lead candidate. ", "A derivative in which the arginine carboxyl has been converted to an amide has been crystallized with HIV-1 PR and the structure has been determined to a resolution of 2.5 A with a final R-factor of 18.5%. ", "The inhibitor binds in an extended conformation that results in occupancy of the S2, S1', and S3' subsites of the active site. ", "Initial structure-activity studies indicate that: (1) the FMOC fluorenyl moiety interacts closely with active site residues and is important for binding; (2) the N(G)-tosyl group is necessary to suppress protonation of the arginine guanidinyl terminus; and (3) the arginine carboxamide function is involved in interactions with the water coordinated to the catalytic aspartyl groups. ", "FMOC-protected arginine derivatives, which appear to be relatively specific and nontoxic, offer promise for the development of useful HIV-1 protease inhibitors." ]
{ "pile_set_name": "PubMed Abstracts" }
[ 0.007633587786259542, 0, 0.012658227848101266, 0, 0.007874015748031496, 0.0026041666666666665, 0.00625 ]
0.005289
5
[ "Q:\n\nIs JSON web token further secured in OpenID, and how?", "\n\nI have read the details of pure JSON web token (JWT), and found that it is signed (e.g. by SHA256) but not encrypted. ", "Hence, attack can read the sensitive information by decoding the header and payload.", "\nI am not very familiar the details of OpenID, but I know that OpenID uses JWT as data format to pass the token.", "\nMy question is:\nIn OpenID (e.g. OpenID 1.0, 2.0, OpenID connect), is JWT further secured by applying additional encryption or other approach, to prevent token stolen or information leakage (I believe the answer should be yes)? ", "And how?", "\n\nA:\n\nEarlier versions of OpenID don't use JWTs, so in a word: no.", "\nOpenID Connect does not mandate the use of encrypted tokens as it prioritizes adoptability over security. ", "However, it does allow you to optionally encrypt the token as a supported protection.", "\n\nID Tokens MUST be signed using JWS and optionally both signed and then encrypted using JWS and JWE respectively, thereby providing authentication, integrity, non-repudiation, and optionally, confidentiality, per Section 16.14. (", "Source)\n\nOtherwise, the better way to think of it is that the protocol uses JWT to protect itself, not the other way around.", "\nWith that said, the protocol spec does list off a number of considerations. ", "These include mitigations like \n\nusing HTTPS\nverifying the token is actually signed with the right key\nsigning token requests\nthe token should include token restrictions\nBlocking open redirects\nusing a cache to prevent token reuse\n\nAnd so forth. ", "A lot of these are common sense mitigations that should apply to any number of token-based protocols.", "\n\n" ]
{ "pile_set_name": "StackExchange" }
[ 0, 0.008333333333333333, 0, 0.008928571428571428, 0.0043859649122807015, 0, 0.015151515151515152, 0, 0, 0.004347826086956522, 0.008064516129032258, 0, 0.0040650406504065045, 0, 0 ]
0.003552
5
[ "GPU Computing in URCS\n\nIntroduction\n\nGPU Computing (GPUc) refers to the use of Graphics Processing Units (GPUs) for General-Purpose computations (i.e. not necessarily related to some fixed graphics-programming API). ", "Alternatively, and maybe more often, GPU-Computing is referred to as GPGPU. ", "The main target of GPU-Computing are heavy, mostly-embarrassingly-, data-parallel applications, which can efficiently exploit the massively-parallel architecture of GPUs. ", "An impressive number of applications falls in this category, and an impressive number of those intersects with the research interests of URCS.", "\n\nnode1x4x2a ( node1x4x2a.cs.rochester.edu ) is our specialized, GPU-Computing-capable server, accessible through the grad-network (NFS-mount), and the target of (at least) systems research on GPU-Computing. ", "This page is a temporary description of the hardware and software installed, and of the necessary steps you have to take before you can make use of the system for your GPUc applications. ", "Though the system is administered by JamesRoche, due to it being the focus of systems research, it might be frequently reboot or heavily in use, so you should consult Jim and/or KonstantinosMenychtas before you try to make use of it for your own research purpose.", "\n\nHardware\n\nnode1x4x2a is a Dell T7500n Workstation, with the following characteristics :\n\nSoftware\n\nIn order to make use of the graphics devices for GPU-Computing, you would need to be familiar with the Compute Unified Device Architecture ( CUDA ) of NVIDIA. ", "Notice that with the NVIDIA CUDA 2.3 you should be able to write code for GPUs both using CUDA and OpenCL. ", "To make use of either API, you should read the respective Documentation ( trivial list under further resources ).", "\n\nNotice that Fedora 11 is not officially supported by NVIDIA CUDA 2.3. ", "However, this shouldn't cause any trouble to the use of the devices for GPU Computing.", "\n\nAt any one point on node1x4x2a :\n\nThe only software that is guaranteed to be installed is the NVIDIA GPU driver (2.3 or later).", "\n\nSimilarly, the only configuration that is guaranteed to have taken place is the configuration needed for the graphics cards to be usable for GPU computing.", "\n\nAdditional Software\n\nIn order to make actual use of the GPUs for GPU-Computing, you will have to install additional software. ", "You can do this at you home directory (NFS-mount).", "\n\nInstall the NVIDIA CUDA Toolkit.", "\n\nGrab the NVIDIA Toolkit Version 2.3 for Fedora 10 x86_64 from here. ", "This package is absolutely necessary, as it includes the compiler (nvcc) and run-time system to run your CUDA programs.", "\n\nInstall the NVIDIA Toolkit under some directory in your home directory (you will prompted during installation). ", "For the following, the toolkit is assumed to be installed under $HOME/Applications/gpu_computing/cuda_x86_64. ", "This is the CUDA_INSTALL_PATH .", "\n\nAdd the following lines to your .bashrc file - or do similarly if you are using CSH or any other shell (for the following we assume you use bash and that .bashrc will be properly loaded). ", "These will make nvcc and the respective dev-files ready to use.", "\n\nBecause of lack of support for Fedora 11, which in fact is lack of support for gcc4.4 (the default gcc in F11), we will be using the \"compatibility-version\" of the gcc compiler, 3.4, which is already installed on node1x4x2a. ", "Any .c/.cpp files compiled with gcc/g++ and linked against .cu files compiled with nvcc, won't work if you use gcc4.4. ", "To make this customary change only valid for use with nvcc, do the following :\n\nCreate a new directory - we 'll call it gcc_compat for this example - inside the CUDA Toolkit directory (the aforementioned CUDA_INSTALL_PATH ). ", "For example, mkdir $CUDA_INSTALL_PATH/gcc_compat\n\nCreate symbolic links to gcc/g++34 in this directory. ", "A valid set of links should look as follows :\n\nInstall somewhere in your home directory. ", "For the following, the toolkit is assumed to be installed under $HOME/Documents/workspace.", "\n\nInstall the following packages\n\nfreeglut-2.4.0-16.fc11.x86_64.rpm\n\nfreeglut-devel-2.4.0-16.fc11.x86_64.rpm\n\nmesa-libGL-7.6-0.1.fc11.x86_64.rpm\n\nmesa-libGL-devel-7.6-0.1.fc11.x86_64.rpm\n\nmesa-libGLU-7.6-0.1.fc11.x86_64.rpm\n\nmesa-libGLU-devel-7.6-0.1.fc11.x86_64.rpm\n\nTo do this, you can just download the aforementioned rpms to your $CUDA_INSTALL_PATH and then manually extract them under the same directory. ", "One way to do this is as follows :\nwhile inside $CUDA_INSTALL_PATH, where the *.rpm files also lie, do\n\nfor file in *.rpm\ndo\nrpm2cpio $file | cpio -idv\ndone\n\nThis will put the libraries and include files under the directories you have already set to your bashrc, hence make them usable without any further configuration.", "\n\nGo back to $HOME/Documents/workspace .Make the following changes (below is a diff you can apply) to the file C/common/common.mk in the SDK\n\nThe only SDK project that cannot be built under this configuration, is scanLargeArray, which you should move outside of the SDK projects directory ( C/src/ ) temporarily.", "\n\nAfter invoking make under the C directory in the SDK, you should be able to make all projects.", "\n\nIf some projects, like projects with OpenGL visualizations, don't work, don't panic; that is natural because of the lack of support for OpenGL through VNC/ssh-X (yet). ", "Same holds for any project which uses non-installed libraries/software. ", "However, a few we have tested and should work include the following ( under C/bin/linux/release/ )\n\ndeviceQuery\n\nbandwidthTest\n\ntranspose\n\nradixSort\n\nmatrixMul\n\nAlternatively\n\nThis is only temporary :\n\nLogin to node1x4x2a and take a look under /localdisk/NVIDIA . ", "You will find the CUDA Toolkit ( under gpu_computing ), the SDK and a .bashrc sample. ", "Do not make any changes in this directory.", "\n\nIf you are just curious to see some GPU-applications, take a look at the NVIDIA SDK, under /localdisk/NVIDIA/NVIDIA_GPU_Computing_SDK/ . ", "Under C/src/ you will find the source code and under C/bin/linux/release the binaries to sample projects. ", "To run the binaries, fix your LD_LIBRARY_PATH first, with something like this: export LD_LIBRARY_PATH=$LD_LIBRARY_PATH:/localdisk/NVIDIA/gpu_computing/cuda_x86_64/lib64/ . ", "Some projects don't work yet.", "\n\nIf you want to make changes, make a copy of /localdisk/NVIDIA either locally ( /localdisk ) or to your home directory, and adjust your shell init file taking into account at least the CUDA-related paths of the sample.bashrc file under /localdisk/NVIDIA .", "\n\nFurther resources\n\nFor questions and help, contact KonstantinosMenychtas, provided that you first have tried the following and have not found the answer you needed.", "\n\nIf you are just starting with CUDA and want a couple of hands-on resources, you might want to try UIUC ECE 498 (Programming Massively Parallel Processors)class notes and Dr. Dobbs \"super-computing for the masses\" tutorials (7 parts, starting here ). ", "Notice though that there is no better manual for the rapidly changing software/hardware platform than the CUDA Programming Guide itself - all other resources can and will probably be slightly outdated." ]
{ "pile_set_name": "Pile-CC" }
[ 0.018518518518518517, 0.02631578947368421, 0.005847953216374269, 0.007042253521126761, 0.019230769230769232, 0, 0.011406844106463879, 0.007692307692307693, 0.009345794392523364, 0.017699115044247787, 0.013888888888888888, 0.011627906976744186, 0.007751937984496124, 0.006369426751592357, 0.0078125, 0.02, 0, 0, 0.008403361344537815, 0, 0, 0, 0, 0, 0.00881057268722467, 0.008403361344537815, 0.0044444444444444444, 0, 0, 0, 0, 0, 0.009615384615384616, 0.010416666666666666, 0.0058823529411764705, 0, 0.003787878787878788, 0.023255813953488372, 0, 0.014388489208633094, 0.009433962264150943, 0, 0, 0, 0.006024096385542169, 0.007936507936507936, 0.004975124378109453 ]
0.00673
5
[ "Americans spent over $53 billion on pets last year\n\nLOS ANGELES -- The economy may have remained sluggish last year but Americans refused to scrimp on their pets, with animal lovers spending upwards of $53 billion on food, veterinary care, kennels and other services in 2012.", "\n\nThat's up 5 percent from 2011, when spending first broke the $50 billion barrier, says the American Pet Products Association, a trade group based in Greenwich, Conn.\n\nAnd APPA President and CEO Bob Vetere predicts another 4 percent gain this year. ", "At about $34.3 billion, food and vet care represented about two-thirds of total spending, with money spent on supplies and over-the-counter medications rising by more than 7 percent.", "\n\nSpending on the growing market of alternative vet care, such as acupuncture, totaled about $12.5 billion. ", "Vetere says spending on services like grooming, boarding, hotels and pet-sitting grew nearly 10 percent during 2012 to almost $4.4 billion." ]
{ "pile_set_name": "Pile-CC" }
[ 0, 0.008, 0, 0, 0 ]
0.0016
5
[ "Evaluation of a microbiological method for detection of irradiation of spices.", "\nThe suitability of a microbiological method, a combination of the direct epifluorescent filter technique (DEFT) and the total aerobic plate count (APC), for the detection of irradiation of spices was evaluated. ", "The evaluation was carried out using different kinds of spices including peppers, paprika, cardamom, cinnamon, ginger; herbs such as thyme, marjoram, basil, and oregano; and garlic powder. ", "Ground spices were tested as well as cut and whole spice samples. ", "Samples were analysed before and after gamma-irradiation mainly with doses of 5 and 10 kGy. ", "Two different methods (centrifugation, filtration) for sample pretreatment were compared. ", "Differences between the DEFT count and the APC with doses of 5 and 10 kGy varied between 3.9 and 6.8 and between 5.7 and 7.5 log units, respectively. ", "The duration of storage after irradiation had no effect on the microbiological results. ", "When applied to fumigated samples, the method might cause confusion in the absence of chemical analyses. ", "The results of the analyses of samples treated by an unknown process indicated that the method could, in fact, detect irradiated samples, although it could not differentiate between irradiated and heat-treated samples." ]
{ "pile_set_name": "PubMed Abstracts" }
[ 0, 0.0047169811320754715, 0.005291005291005291, 0, 0, 0, 0.006666666666666667, 0, 0, 0 ]
0.001667
5
[ "-1/3))**(-13) assuming x is positive.", "\nx**(91/15)\nSimplify (f*((f**3*f)/f)/f*f)**(-2/35)*f**(2/17)/f*f**1 assuming f is positive.", "\nf**(-66/595)\nSimplify (h**(-3)*h)**(-19)/(h**2*h/(h**17*h)) assuming h is positive.", "\nh**53\nSimplify ((r*r/((r*r**(-2/55)*r*r)/r)*r)**(6/7))**23 assuming r is positive.", "\nr**(7866/385)\nSimplify (b/b**29*b**(1/8))/(b**(-10/3))**(-3/7) assuming b is positive.", "\nb**(-1641/56)\nSimplify ((b*b**(2/13))**(-14))**(-22) assuming b is positive.", "\nb**(4620/13)\nSimplify (u**1*u*u)**(2/49)*(u*(u/u**3)/u)**(-4/21) assuming u is positive.", "\nu**(74/147)\nSimplify (q**0/q*q*q**(1/3)*(q*q/(q/(q*(q/(q**(2/5)/q))/q*q)))/q*q**(-6)/q)**(2/23) assuming q is positive.", "\nq**(-122/345)\nSimplify (p**7/(((p*p**11)/p*p*p)/p*p))/(p**(2/5)*p**12) assuming p is positive.", "\np**(-92/5)\nSimplify ((t/(t/(t*(t/(t/((t**(1/4)*t)/t*t)))/t*t)))**(-4/11)*(t**2)**(-42))**(-12) assuming t is positive.", "\nt**(11196/11)\nSimplify ((z/((z/(z*z**(-1)*z))/z)*z/(z**4/z))/(z**3*(z*z**(-1/8)*z)/z))**(3/7) assuming z is positive.", "\nz**(-93/56)\nSimplify ((f/f**(-10))**(5/2))**(-6/11) assuming f is positive.", "\nf**(-15)\nSimplify g**(-1/5)/(g**(3/7)/g)*(g**(5/4))**17 assuming g is positive.", "\ng**(3027/140)\nSimplify (w**(5/6)*w*w**(3/5))/((w/(w**(-10)*w))/w**(-3/10)) assuming w is positive.", "\nw**(-118/15)\nSimplify ((((((v/(v/v**(-2/5)))/v)/v)/v)**37)**(-39))**13 assuming v is positive.", "\nv**(318903/5)\nSimplify (s*(s/(s/(s*s**(-6/7))))/s*s*s**(11/4))/(s**(-16)*s**8*s) assuming s is positive.", "\ns**(305/28)\nSimplify (u*u**2*u)**(-3/22)/((u**(-2)/u)/u**0) assuming u is positive.", "\nu**(27/11)\nSimplify d*d**8*d*d*d**(3/5)*d*d**(2/5)*d**11 assuming d is positive.", "\nd**24\nSimplify ((l*l**(-2))/l)**(-43)*l**(-2/11)/((l/(l/l**(-3/8)*l))/l) assuming l is positive.", "\nl**(7761/88)\nSimplify ((s**(-18)/s)**(16/9))**(3/25) assuming s is positive.", "\ns**(-304/75)\nSimplify (b**(-11)*b*b*b)/((b*b/(b**(-4/7)/b*b*b)*b)/b)*(b**(-4))**(-37/4) assuming b is positive.", "\nb**(192/7)\nSimplify ((l**(2/29)*l*l)/l**(2/41))**(-39) assuming l is positive.", "\nl**(-93678/1189)\nSimplify ((l/(l/(l/(l*l**(-2/7)/l))*l))/(l**(-13/2)/l*l))/(l**23/l*(l**(-1/11)/l)/l) assuming l is positive.", "\nl**(-2021/154)\nSimplify v**(-19)/v*v**(-3/7)*v**(-2)*v*v**(-1/27)/v assuming v is positive.", "\nv**(-4246/189)\nSimplify (((t**(1/12)*t)/t*t/(((t**(3/11)/t*t*t)/t*t)/t)*t)/(t**1)**(3/17))**(8/7) assuming t is positive.", "\nt**(7334/3927)\nSimplify (j/(j/(j**(-1/3)/j))*j)**(-31)*(j*j**(1/4))/(j**40*j) assuming j is positive.", "\nj**(-353/12)\nSimplify (g**(-1/13)/g**(3/2))/((g**(-25)/g)/g**(2/69)) assuming g is positive.", "\ng**(43867/1794)\nSimplify ((f/(f/f**(-7))*f/((f*(f/(f/(f*f**(2/27))))/f)/f))**(-35))**0 assuming f is positive.", "\n1\nSimplify (a/(a/a**(-2/45)*a))/a*a**19*a*(a**0/a)**(-1/14) assuming a is positive.", "\na**(11357/630)\nSimplify (b**(-3/2)/b**(7/2))/(b**(-1/5)*b)**(-2/55) assuming b is positive.", "\nb**(-1367/275)\nSimplify (g**(2/7))**(-31/4)/((g/g**(-1/16))/(g*g*g**(-10)/g*g)) assuming g is positive.", "\ng**(-1263/112)\nSimplify ((t*t*t*t*t/(t**(5/4)*t)*t*t)/t)**(-6/25)*(t**(-1))**(8/5) assuming t is positive.", "\nt**(-5/2)\nSimplify ((x**(-3)/(x*x**12))**(4/5))**(-49) assuming x is positive.", "\nx**(3136/5)\nSimplify (q**(-2/9))**(-25)/((q/(q/q**(-8))*q)/q**(-1/18)) assuming q is positive.", "\nq**(25/2)\nSimplify ((s/(s/s**(-5/3)))/s)**(5/8)/(s**15/(s**(-12)*s)) assuming s is positive.", "\ns**(-83/3)\nSimplify (j**(-21)*j/j**(-11/5))/(j*j*(j/(j*j*j/(j/j**(-1/17))))/j*j*j/((j*j**6)/j*j)) assuming j is positive.", "\nj**(-1093/85)\nSimplify (k**(16/5)*k/k**(-3/10))/(k**(-2/27)*k*k*k)**7 assuming k is positive.", "\nk**(-863/54)\nSimplify ((a*a**(-10/9))/(a*a**(-18)))**46 assuming a is positive.", "\na**(6992/9)\nSimplify (y/(y*y**(-3/5))*(y*y*y/(y/(y*y**30)))/y)/(y*y*y*(y**(5/12)/y)/y*y*y*y*y)**(-42) assuming y is positive.", "\ny**(2601/10)\nSimplify ((((s*s*s/(s**17/s))/s)/(s*s**(1/4)))**(2/17))**(17/5) assuming s is positive.", "\ns**(-61/10)\nSimplify ((d/(d*d/d**19*d*d))/((d*(d/(d/(d**3/d)))/d*d)/d))/(d**(2/109)*((d*d**(-33)/d)/d)/d) assuming d is positive.", "\nd**(5339/109)\nSimplify (u*u**0)**(5/14)*u**(2/5)/u**(-3/4) assuming u is positive.", "\nu**(211/140)\nSimplify (s*s**4/s*s*s**(-2/87))/((s*s/(s/s**12)*s)/(s*(s**(18/5)/s)/s*s*s*s*s)) assuming s is positive.", "\ns**(-1054/435)\nSimplify h**(4/5)/h**(-8/3)*h*h**(-2/19)*h*h*h*h**(-11) assuming h is positive.", "\nh**(-1037/285)\nSimplify (m**(2/7)/m*m*m*m**3)/(m/(m*m**0))**43 assuming m is positive.", "\nm**(30/7)\nSimplify ((i**(-1/17)/i)/(i/i**(-10)))/((i**27*i)/(i*i**(-3/4)*i)) assuming i is positive.", "\ni**(-2639/68)\nSimplify ((((z/(z/z**(-1/23)))/z)/z)/z*z*z*z*z**(-8/9)/z*z)/(z**(-2/15))**(-3/10) assuming z is positive.", "\nz**(-5032/5175)\nSimplify (q*(q/(q/q**(-5/6)*q))/q)**(-1/5)*(q/((q*q**(-16)/q)/q))/(q**(1/3)/q) assuming q is positive.", "\nq**(571/30)\nSimplify (((l/l**(-1/2))/((((l/l**(-2/9))/l*l)/l*l)/l))/(l**(1/5)/l)**24)**(-38) assuming l is positive.", "\nl**(-35017/45)\nSimplify ((c*c*c*c*c/(c**(-1)*c)*c)**(-44)/((c/(c**11*c))/(c*c**9)))**16 assuming c is positive.", "\nc**(-3888)\nSimplify (a**(2/9)*a)/(a/((a**(2/23)/a)/a*a)*a)*a**(-14)*a*a**(-15) assuming a is positive.", "\na**(-6146/207)\nSimplify ((a/((a/(a**1*a))/a))/(a**(2/25)/a))/((a*((a*a/a**(-10))/a)/a*a)/((a**(-2/7)*a)/a)) assuming a is positive.", "\na**(-1464/175)\nSimplify ((r**(-2/13)*(r**(-6/11)*r*r*r)/r)/((r*r*r**(-3/5))/r*r*r*r**6))**(-24) assuming r is positive.", "\nr**(121824/715)\nSimplify (m/(m/(m*(m/m**23*m)/m))*m**31)/(m/m**(-3))**37 assuming m is positive.", "\nm**(-138)\nSimplify ((q/q**25)/(q/(q/(q/(q*q*q*(q**1*q)/q)))))**(2/51) assuming q is positive.", "\nq**(-14/17)\nSimplify r*((r*r**27)/r*r)/r*r*r*r**26*r**(-5)/(r**(-1/4)*r) assuming r is positive.", "\nr**(201/4)\nSimplify ((t/(t**(-3)*t*t)*t**(-3)*t)**(-31))**(-29) assuming t is positive.", "\n1\nSimplify ((f**(2/5))**20/(f*f*(f/(f*f/(f*f**(2/5))*f))/f)**(-1/41))**(-11/4) assuming f is positive.", "\nf**(-9031/410)\nSimplify (s/s**(-7))**30/(s**(-1/3))**(2/19) assuming s is positive.", "\ns**(13682/57)\nSimplify (b**(-11)*b)/b*b/(b*b/b**(-9)*b)*b/b**(4/9)*b*b**(-14)*b assuming b is positive.", "\nb**(-301/9)\nSimplify (((v**(-4/3)*v)/v*v*v*v*v**0)/(v**(1/3)*v)**(-2/71))**(-2/29) assuming v is positive.", "\nv**(-242/2059)\nSimplify k/((k*k**(-7/4))/k)*k*k*k**(5/4)*(k/(k/(k**(2/17)*k)))/k**13 assuming k is positive.", "\nk**(-100/17)\nSimplify (u**(-5/6)*u)**(-23)/(u**28)**(13/3) assuming u is positive.", "\nu**(-751/6)\nSimplify (x**(-3/4)/x)/(x*x/((x/x**12*x)/x)*x)*(x**(-1/4))**(1/18) assuming x is positive.", "\nx**(-1135/72)\nSimplify b*b*b**(-13)*b/(b*b**(-3/2))*b**(2/5)/(b*b**10*b*b) assuming b is positive.", "\nb**(-221/10)\nSimplify (k*(k*k/k**(-22))/k*k/((k*k**(-1/55)/k)/k))/(k**(-12/5)/k**(-2/63)) assuming k is positive.", "\nk**(98359/3465)\nSimplify (w/(w/(w**(-4/5)/w*w))*w)**(3/2)/(w**(-26)/w**(-21)) assuming w is positive.", "\nw**(53/10)\nSimplify (k**(3/16)/(k*k**15))**36 assuming k is positive.", "\nk**(-2277/4)\nSimplify (d**24*d/(d**(1/10)*d))**(5/6) assuming d is positive.", "\nd**(239/12)\nSimplify ((k/((k**(-2/19)*k*k)/k)*k)/k)**(-35)/(k**19)**(-22/9) assuming k is positive.", "\nk**(7312/171)\nSimplify m/m**(1/24)*m**(-7)*m**(-3/5)*m**(-2) assuming m is positive.", "\nm**(-1037/120)\nSimplify (w**21*w/w**23)/(w**(-4/13)/w*w*w/w**5) assuming w is positive.", "\nw**(43/13)\nSimplify (((d/(d*d**4))/d*d/(d/(d**(-1/3)*d)*d*d))/((d**(-3/5)*d)/(d/(d/(d/d**(-7))))))**(-26) assuming d is positive.", "\nd**(-494/15)\nSimplify (((p*p**1)/p)**(-27)/(p*p*p/((p*p/(p*p**0*p))/p)*p)**(5/2))**(-1/64) assuming p is positive.", "\np**(79/128)\nSimplify (((q/q**0*q)**45)**(-28/5))**(3/13) assuming q is positive.", "\nq**(-1512/13)\nSimplify (((((h/(h**(2/7)/h))/h)/h)/h)**15/((((h*h*h**(2/3))/h)/h)/h)**(-49))**(-6/5) assuming h is positive.", "\nh**(1496/35)\nSimplify ((d/(d/(d/(((d**(-2/21)/d)/d)/d))*d))/(d*d*d*d**(-18)*d))/(((d/d**(-3/7)*d)/d)/d**(-2/57)) assuming d is positive.", "\nd**(297/19)\nSimplify (p*p/(p*p*((p*p*p**13)/p)/p*p*p))**(19/4)/(p**(6/5))**31 assuming p is positive.", "\np**(-2169/20)\nSimplify (x**(-4/3))**(-4/41)/(x**(-3))**(-11/5) assuming x is positive.", "\nx**(-3979/615)\nSimplify r*r**(3/8)*(r**4/r)/r*(r/r**(-7/5))/(r/r**(-6)) assuming r is positive.", "\nr**(-49/40)\nSimplify (r*r**(2/3)*r)/r**(8/7)*(r*(r*(r/r**(-11)*r)/r)/r)**(-10) assuming r is positive.", "\nr**(-2698/21)\nSimplify ((l*l**(-3))/l*l*l**(-2/23))/(l**10*(l**(-29)/l)/l) assuming l is positive.", "\nl**(435/23)\nSimplify (q**(-2/23)*(q/(((q*q**(5/6)*q*q*q)/q)/q))/q)/(q*" ]
{ "pile_set_name": "DM Mathematics" }
[ 0, 0.01098901098901099, 0, 0, 0, 0, 0.011235955056179775, 0.008333333333333333, 0, 0, 0.00847457627118644, 0, 0, 0.010101010101010102, 0, 0, 0, 0, 0.010309278350515464, 0, 0, 0.012658227848101266, 0.007936507936507936, 0, 0.00819672131147541, 0, 0.010752688172043012, 0, 0, 0.010869565217391304, 0.019230769230769232, 0, 0, 0, 0, 0.00819672131147541, 0.02127659574468085, 0, 0.007936507936507936, 0, 0.007692307692307693, 0.012048192771084338, 0, 0, 0.011494252873563218, 0, 0, 0.008403361344537815, 0, 0, 0.009708737864077669, 0, 0, 0.010309278350515464, 0.010638297872340425, 0, 0, 0, 0.011904761904761904, 0.009615384615384616, 0, 0.009174311926605505, 0.012048192771084338, 0.009708737864077669, 0.010101010101010102, 0.008771929824561403, 0, 0.014285714285714285, 0, 0, 0, 0, 0.007692307692307693, 0, 0, 0.008064516129032258, 0.0072992700729927005, 0.0196078431372549, 0.011494252873563218, 0, 0, 0, 0 ]
0.004537
5
[ "Q:\n\nPython: How to make a dict field relative to another dict field\n\nI have a dict looking like this: \nvisits = {'visit_1': {'date': '23-11-2016'},\n 'visit_2': {'date': '23-12-2016'}}\n\nThe dict consists of a lot of visits, where the 'date' is relative to the visit_1 date. ", "\nQuestion: Is it possible for a python dict to refer to self? ", "Something like this:\nfrom datetime import datetime, timedelta\n\nvisits = {'visit_1': {'date': datetime('23-11-2016')},\n 'visit_2': {'date': <reference_to_self>['visit_1']['date'] + timedelta(weeks=4)}\n\nEDIT:\nThe first visit is not known at initialization. ", "The dict defines a fairly complicated visit sequence (used in treatment of cancer patients)\n\nA:\n\nIf I understand the problem correctly, you can define the first visit date prior to creating the dictionary:\nfrom datetime import datetime, timedelta\n\nvisit1_date = datetime(2016, 11, 23)\nvisits = {'visit_1': {'date': visit1_date},\n 'visit_2': {'date': visit1_date + timedelta(weeks=4)}}\n\nprint(visits)\n\nTo extend that a little bit further, you may have a visits factory that would dynamically create a dictionary of visits based on a start date and the number of visits (assuming the visits come every 4 weeks evenly): \nfrom datetime import datetime, timedelta\n\ndef visit_factory(start_date, number_of_visits):\n return {'visit_%d' % (index + 1): start_date + timedelta(weeks=4 * index)\n for index in range(number_of_visits)}\n\nvisits = visit_factory(datetime(2016, 11, 23), 2)\nprint(visits)\n\nA:\n\nPython containers (e.g. dictionaries) can definately refer to them selves. ", "The problem with your example is that you are not just storing a reference, but the result of an expression (which involves the reference). ", "This result will not itself keep any reference to how it was made, and so you loose the reference.", "\nAnother Small problem is that you cannot create an object and reference it in the same expression. ", "Since dicts are mutable though, you can just create some of its elements first, and then add elements to it in further expression, referencing the dict via its variable name.", "\n\n" ]
{ "pile_set_name": "StackExchange" }
[ 0, 0, 0.007575757575757576, 0.0030272452068617556, 0, 0, 0, 0, 0 ]
0.001178
5
[ "Q:\n\nProgramatically check if google protocol buffer field is required\n\nIs it possible to programmatically check if a given proto field is marked as required vs optional? ", "I'm using python and have a FieldDescriptor object, but can't find a way to determine whether the field is required.", "\n\nA:\n\nA quick look at the documentation indicates that your FieldDescriptor should have a label attribute indicating whether it's optional, required, or repeated.", "\nfrom google.protobuf.descriptor import FieldDescriptor\n\nif fd.label == FieldDescriptor.", "LABEL_OPTIONAL:\n # do thing\nelif fd.label == FieldDescriptor.", "LABEL_REQUIRED:\n # do other thing\nelse:\n # do third thing\n\n" ]
{ "pile_set_name": "StackExchange" }
[ 0, 0.008620689655172414, 0.006172839506172839, 0.022727272727272728, 0.015625, 0 ]
0.008858
5
[ "All other trademarks and copyrights are the property of their respective holders. ", "The reviews and comments expressed at or through this website are the opinions of the individual author and do not reflect the opinions or views of JavStick. ", "JavStick is not responsible for the accuracy of any of the information supplied here.", "\n\n\n\n" ]
{ "pile_set_name": "OpenWebText2" }
[ 0, 0.006329113924050633, 0.011764705882352941, 0 ]
0.004523
5
[ "Ephraim Moravian Church\n\nEphraim Moravian Church is located in Ephraim, Wisconsin. ", "It was added to the National Register of Historic Places in 1985.", "\n\nHistory\nThe church was founded by Norwegian immigrant Andreas Iverson. ", "Originally located on the shore of Eagle Harbor in Ephraim, the church was moved to its present location in 1883.", "\n\nReferences\n\nCategory:Churches on the National Register of Historic Places in Wisconsin\nCategory:Moravian churches in the United States\nCategory:Churches in Door County, Wisconsin\nCategory:Norwegian-American culture in Wisconsin\nCategory:Churches completed in 1859\nCategory:National Register of Historic Places in Door County, Wisconsin" ]
{ "pile_set_name": "Wikipedia (en)" }
[ 0.012048192771084338, 0.015384615384615385, 0.0136986301369863, 0.008849557522123894, 0.005934718100890208 ]
0.011183
5
[ "[Spatial and biogeographic characterization of macroalgal assemblages from Bahía del Rincón, Baja California Sur, Mexico].", "\nMacroalgal studies in Baja California Sur have dealt mainly with occurrence and seasonality, but some areas are poorly known even for these basic data. ", "Bahia del Rincón-La Rivera is an important high-productivity fisheries area where coastal infrastructure development is under way. ", "A spatial characterization of the marine flora from Bahia del Rincón-La Rivera was done by intensive sampling at different depths and localities with skin and SCUBA diving. ", "At least 500 m2 were surveyed in each site. ", "Additionally. ", "quantitative sampling was done in ten random 25 cm2 quadrates per site. ", "In the intertidal section, density and cover estimates were used. ", "We also investigated the historical records and geographical affinities. ", "A total of 72 species were identified (most were red algae: 62%). ", "We found no general trend in the biogeographical affinities, which varied with each taxonomic group. ", "Most brown algae species were tropical-endemic; red algae temperate-cosmopolite and green algae tropical-cosmopolite. ", "In the spatial assemblage structure we found a high similarity between the intertidal areas, but a low similarity in shallow or deeper areas (3-5 m). ", "This pattern was the same when we compared the abundance of the main species. ", "We suggest that there are significant spatial differences in recruitment and development of the assemblages in relation to vertical distribution (depth) and position along the shore. ", "There is a clear-cut Gelidium-Jania belt in the intertidal zone and a Padina-Dictyota belt below the low tide. ", "Sporadic and year-round species occur in the intertidal zone, annual and perennial species below the low tide line. ", "Sites differ in recruitment and this affects the abundance of other species (such as coraline and Caulerpa species). ", "Temperature and sedimentation affect seasonality, but community structure is relatively constant throughout the year." ]
{ "pile_set_name": "PubMed Abstracts" }
[ 0.00819672131147541, 0, 0.007633587786259542, 0.005780346820809248, 0, 0, 0, 0, 0, 0, 0, 0, 0, 0, 0, 0.009009009009009009, 0, 0.008547008547008548, 0 ]
0.002061
5
[ "Georgetown University conducts pre-employment screening for all positions. ", "The screening may include, but is not\nlimited to, a criminal background check, verification of academic credentials, licenses, certifications, and/or\nverification of work history. ", "Georgetown utilizes an online system which will require a valid email address. ", "Offers\nof employment are conditional, based on satisfactory completion of the pre-employment background check process.", "\n\nIf you are a qualified individual with a disability and you are unable or limited in your ability to use or access\nour on-line Jobs Posting system as a result of your disability, you can request a reasonable accommodation by clicking\nhere, or contact the Office of Institutional Diversity and Affirmative\nAction (IDEAA) at (202) 687-4798.", "\n\nIf you experience technical difficulties during the application process, please call the HR Front Desk at 202-687-2500." ]
{ "pile_set_name": "Pile-CC" }
[ 0.013333333333333334, 0, 0.012658227848101266, 0, 0.0058823529411764705, 0.008264462809917356 ]
0.00669
5
[ "A Look Back at Robby Anderson\n\nRobby Anderson has had to fight for respect it seems his entire life. ", "Some of that is due to his own shortcomings, but none of those shortcomings were on a football field. ", "Given the fact that Robby was born less than 9 miles from MetLife Stadium and starred at Temple University about 80 miles south, he should have been a natural fit for the green and white. ", "Still he took the long way in getting to the Jets. ", "Now that he is here he is looking forward to becoming an integral part of a new era Jets resurgence, one of the Jets most potent weapons in 2019 and beyond.", "\n\nLets first backtrack to see how Robby Anderson made his way to the NFL.", "\n\nRobby was born in Fair Lawn, New Jersey, in 1993. ", "His family moved to Plantation, Florida, when he was a child. ", "He grew up in a poor section of town where temptation to do unlawful things was all around. ", "He got into some trouble in high school (nothing very serious) and had a choice to continue in that direction or devote himself to sports as a way to possibly help his future.", "\n\nHis high school football coaches Doug Gatewood and Delvin Randolph preached to Robby the virtues of sports so Robby played football and ran track. ", "Everyone noticed he was extremely fast. ", "Robby played DB and WR at Plantation High. ", "He had 25 TDs, over 1,900 yards receiving, and 3 INTs in his career.", "\n\nWhen colleges came looking for players Robby was not very sought after. ", "His petty foibles as a young man contributed plus he was very slender. ", "This combined with the fact that Robby wasn’t a great student hurt his chances with well known programs. ", "He only received scholarship offers from Central Michigan and Temple. ", "Robby enrolled at Temple University in 2011 and was a redshirt that season. ", "He played in only 6 games in 2012 (mostly on special teams) and was listed as a CB not a WR.", "\n\nIn 2013 Robby felt lost at Temple. ", "He had few friends. ", "The climate was completely different than he was used to in Florida, and he was running with the second team defense as a CB. ", "He then quit the team and moved back to Florida. ", "After many discussions with friends and family he called his coach (Matt Rhule) and asked if he could return. ", "He was invited back as a non scholarship athlete late in the summer. ", "When he came back he worked as a WR and reached the starting lineup in the 5th game.", "\n\nHe had a breakout game against SMU that set a new school record for receiving yards at 239. ", "He ended the year with a 3 TD game against Memphis., ", "He finished the year with 44 receptions, 791 receiving yards and 9 TDs. ", "He was thought to be Temple’s #1 receiver for 2014 and could be a 2nd day NFL draft pick if he did well. ", "That did not happen as Robby as he was dismissed from Temple shortly after the season because of poor grades.", "\n\nRobby went back to Florida figuring his dream of playing in the NFL was over. ", "It took some time and prodding from his mom, but Anderson went back to school and studied like he never studied before. ", "He needed nearly straight “A’s” to make it back to Temple. ", "It took the entire 2014 year, but Anderson made his grades and played the 2015 season at Temple. ", "He played 14 games with 70 receptions, 939 yards and 7 TDs despite having a poor QB. ", "He was not invited to the NFL Combine which hurt his chances at being drafted.", "\n\nRobby did well at his pro day and ran a 4.36/40 which would have been 3rd best at the Combine. ", "The fact that he left school twice, problems in the past, and the Combine snub left Robby as a UDFA after the draft. ", "He signed with the Jets, went to rookie minicamp, and then home to Florida. ", "While in Florida he met with Chad (Ocho Cinco) Johnson who grew up in the same neighborhood as Robby. ", "Chad worked with Robby some and gave him advice (like only Chad can give) which gave Robby confidence going into training camp.", "\n\nAnderson entered training camp far down on the list of WR with a chance to make the roster. ", "Brandon Marshall, Eric Decker and Quincy Enunwa were certain locks to make the team so that left just a few spots. ", "Robby worked hard in camp and excelled in preseason games. ", "He finished with 13 receptions, 264 yards and 3 TDs. ", "He made the team then received a great chance to play in meaningful games when Eric Decker went down for the year with an injury. ", "He showed he can compete with plays like this.", "\n\nThat was Fitzpatrick just throwing it up (into coverage) giving the new guy a chance, and he came through. ", "Robby started by playing most of his snaps in the slot but moved outside to the boundary later in the year. ", "Robby at the time was one of those players who was faster than quick so the move outside was a welcome change. ", "It allowed him to feel more comfortable, running routes he had run many times in college. ", "At the same time he was developing trust from his QB along with the rest of his teammates.", "\n\nHere he catches his 1st TD in a home game on Monday Night Football from Bryce Petty, a memory he will have for a lifetime. ", "It had to be very gratifying for Robby after his life’s journey to actually have an NFL receiving TD on his resume. ", "He came a long way as a skinny kid from South Florida. ", "You know he had to be teased unmercifully by peers when he said he wanted to play in the NFL. ", "No one knows all he went through except some family and a few close friends. ", "Those two high school coaches who believed in Robby. ", "They set him on a path to success instead of an unknown future in his neighborhood. ", "They have to sit back and smile knowing they probably set a kid on a path to this type of success.", "\n\nAs a rookie UDFA WR this play showed he had the kind of speed that can beat a defense deep no matter what the coverage. ", "The Jets were getting destroyed in this game, but still it is a huge stage for an unknown player to step up and make a play. ", "It put him on the map as a possible playmaker.", "\n\nAfter that Robby showed he had big play capabilities with a play later in the year against the same Dolphins team. ", "This now showed that Robby could succeed on routes other than “9” routes or other “go” type routes. ", "He also will venture across the middle on in breaking routes as well as run intermediate type routes. ", "It is a nice step in the route progression for a player who might have been known as only a player who can take the top off a defense.", "\n\nAs he progressed as a receiver he had a hard time convincing people he was an all around good receiver. ", "He went back to Florida and worked with Chad (Ocho Cinco) Johnson on his footwork. ", "Being a lean boundary receiver he needs to be able to beat press coverage in order to get into his routes in a timely fashion. ", "Chad Johnson always had very quick feet so Robby worked on footwork since he wasn’t going to break press coverage with his strength.", "\n\nHere he is working against one of the better cover corners in the NFL in Desmond Trufant. ", "You can see the quick feet as Robby feigns a move to the left only to come back to the right breaking free easily and then just dusting Trufant.", "\n\nTrufant makes the mistake of playing man coverage but not pressing Robby off the line. ", "With Robby’s speed and footwork there was no way Trufant could keep up. ", "He is beaten soundly in the first 5 yards. ", "Once Robby got even with Trufant, with nothing but open space and an end zone in front of him, it was over. ", "Trufant is considered an excellent defender.", "\n\nThis next clip is basically the same play although it is a sight adjustment by McCown and Robby. ", "The Dolphins are playing a single high safety on the hash farthest away from Robby’s side. ", "You see McCown look left at the snap fully knowing that he was coming back right, he is trying to hold the safety to the left side of the field as long as possible.", "\n\nAlterraun Verner #42 was a Pro Bowl zone corner who just gets roasted in man coverage. ", "You had to know the Jets were just waiting for this matchup. ", "Robby gives a cursory lean to the outside then takes off on an inside release, and again this race was over after the first 5 yards. ", "This is an easy 69 yard TD. ", "This is the skill set Robby brings to the Jets that no other player on the squad has. ", "Robby is 26 with his prime years ahead of him.", "\n\nThis next clip is again that same sight adjustment play, but this time McCown doesn’t hold the safety long enough with his look left. ", "With corner Logan Ryan playing off Robby’s outside shoulder all Robby had to do is take one step right and blast back inside to gain the advantage. ", "You can see again that after 5 yards the race is over.", "\n\nKevin Byard #31 is an All Pro safety and reads the play fairly quickly. ", "He gets over just in time to stop a TD. ", "Even with that Robby had to slow down just a bit to wait on the ball then adjust to the throw that is wisely lead to the outside, away from the defenders. ", "By allowing Robby inside leverage the defense was giving him room away from the sideline. ", "This way when McCown’s throw is towards the sideline it is away from the defenders on the play.", "\n\nWhen you can make plays like these long passes, it will force the defense play a more passive coverage with a lots of deep zone which will open up plays underneath. ", "Robby has stated openly that he believes he can be a 10 target a game player. ", "and to do that you have to be able to run all kinds of different routes. ", "It’s a good thing he can do that. ", "Here against the evil empire he shows some speed and versatility.", "\n\nThis was a 2nd and 10 play that is just a simple swing pass to a motion man but turns into much more with Robby’s speed. ", "This is s safe pass that gets Robby to the edge of the defense and allows him to gain 16 yards from the 14 all the way to the 30.", "\n\nLater in the same game it is 4th and 12 ,and Robby is able to get inside the middle of the zone and gain 32 yards on just a simple in route. ", "The speed that Robby has is not just useful on go routes. ", "It also makes defenders give a wider angles when trying to tackle him in the open field. ", "Robby is not real elusive but he is able to exploit seams and gaps in the defense by racing through them before they close.", "\n\nThe Patriots are blitzing on the play so it left Devin McCourty alone in coverage. ", "Because of Robby’s speed he plays outside influence coverage so he squeezes Robby inside towards the middle of the defense where he can get help. ", "This works to perfection, but he is so far off in coverage it allows an easy catc. ", "Although McCourty didn’t allow a TD he did allow a huge play on a 4th and 12 situation.", "\n\nWhen a defense plays against speed you will often see them playing two high safeties to have more deep coverage, use a safety over the top of a receiver (high and low coverage), or play quarters coverage with the DBs. ", "Here the Dolphins are playing two high safeties when the safety from Robby’s side of the field decides to help out on the TE who is running an out route to the sideline. ", "You see McCown just read this, excitedly it is as if he couldn’t get the ball out of his hand fast enough on a gift TD pass.", "\n\nYou can see Cordrea Tankersley get totally toasted on the play then turn around and look at the safety in astonishment. ", "He thought he had deep inside help but the safety deserted him for an out route. ", "In the NFL speed kills so when you make a mistake in coverage it doesn’t take long to find the other team in the end zone.", "\n\nYou see a lot of teams who play man coverage using the outside shoulder technique against Robby to force him inside and towards possible help from safeties. ", "The reasoning is sound because when you play heads up on Robby you give him a choice of directions to go. ", "This makes it very difficult to guard him one on one. ", "This is against the #13 PFF graded corner from the 2017 season, and Robby beats him like a drum.", "\n\nTre’Davious White gives Robby that free release, so when Robby takes the outside route White has no help over the top. ", "This is also great route running as Robby leaves himself a good 5 yards between him and the sideline to allow McCown room to fit a pass in. ", "Once again White is beaten badly in the first 5 yards so if the throw is on the money he cannot prevent a TD. ", "The throw in a little behind, but Robby reaches back, hauls in the throw, puts it away snugly then falls harmlessly out of bounds. ", "You can see two other defenders came over to lend assistance but they are just spectators on the play.", "\n\nEven when you do play the correct coverage with the defense is spot on in their assignments it doesn’t guarantee success. ", "Here the Carolina Panthers are playing over coverage on Robby with James Bradberry in coverage and Mike Adams over top. ", "Bradberry graded out as the #26 PFF ranked corner of that season.", "\n\nWith the Panthers up by 9 with less than 90 seconds to go in the first half this is the correct coverage to play. ", "You make the Jets try a field goal instead of getting a TD. ", "Yet great players can make great plays, knowing Robby is 6’ 3” and Mike Adams is 5’ 11’ with Bradberry at 6’ 1” so McCown throw a high arching ball that Robby can reach up to grab.", "\n\nThe next clip is a drag route where Robby comes back inside the TE in sort of a rub play which helps Robby to get into open space. ", "Again it is Trufant on the coverage. ", "He gets held up by the ILB who is tangled the TE so he is late coming over. ", "By the time he makes it back to the play Robby is downfield for a 25 yard play.", "\n\nThis is a safe, huge play in the 4th quarter of a close game in the rain. ", "It once again shows the versatility of Robby in his route running and how far he has come from that first TD he scored on a Monday night.", "\n\nThis next play is just idiotic coverage against Robby for the situation. ", "With less than a minute to go in the game and the Jets down by 12 you would think you would double cover the fastest man on the field or that least give over the top help. ", "Brent Grimes was the #30 PFF ranked corner in that year, and he probably would have been ranked higher if he was given some help on this play.", "\n\nYou know you have great speed when the entire stadium knows your team has to try and throw the ball deep yet the do it successfully anyway. ", "This ball is even underthrown by a few yards, but Robby is able to reach back to haul it in anyway. ", "Robby’s not given the credit deserved for his hand especially in contested situations.", "\n\nThis next clip is of Robby against Bradley Roby who I was very high on as a DB coming out in the Draft in 2014. ", "Here he is playing an off coverage plus he is giving ground at the snap. ", "The problem with that is if you stop your feet for even a split second your a dead duck; which is exactly what happens.", "\n\nThis is a 76 yard TD pass from Sam Darnold to a wide open Robby Anderson because of a double move. ", "It’s the one area where Bradley Roby struggles in coverage so it was wise to try the move on him.", "\n\nYou can see it much clearer in another view.", "\n\nBradley Roby ran 4.39/40 at the Combine, but Anderson makes him look slow. ", "You can see Bradley stop his backpedal for just a second but that is all it took. ", "With no help over the top it was an easy pitch and catch for Sam and Robby. ", "We will look for much more of this in the years to come in the coach Gase offense. ", "The new offense is said to be much more vertical than Bates’ offense, and the players (WR’s) seem to like it as does Robby. “", "That’s what I’ve been waiting for.” ", "Robby is quoted as saying, ”Now I have a coach that’s going to utilize me as a player, not just make me run straight down the field. ", "Look at my second year [2017]. ", "I wasn’t just running straight down the field. ", "I was getting the ball in my hands, getting touches, getting momentum, opening things up for other people, and we were putting up points. ", "Last year was hard. ", "I was kind of put into a box. ", "But I know this year, it’s going to be lights out.” “", "Nothing else should be our goal besides the Super Bowl,”\n\nThis last play has Sam and Robby hooking up against the Texans near the goal line. ", "The more these two work together the better it should be. ", "As earlier when a veteran like McCown was in the game; both Robby and he were able to see the same opportunities in coverage and make a sight adjustment with just a look. ", "Sam and Robby should develop the same QB/WR coordination that McCown and Robby had, only better. ", "It will take some time but the trust factor between players is vital to a dynamic offense.", "\n\nPlays like this can be effective in opening up space for the receivers, TE’s and RB’s. ", "Intermediate routes that find holes in zones are crucial plus this is no short route. ", "It gained 26 yards. ", "When Sam came back from injury last year Robby caught 23 balls for 336 yard and 3 TDs in the last 4 games. ", "This bodes well for the future and these two leading the charge on offense with a lot more weapons to use this year.", "\n\nAs most of you know Robby has been his own worst enemy at times. ", "He has made some poor decisions but he is older now. ", "He has had time to think and change his mindset. ", "His uncle Daniel Harris (who has been like a father to Robby) said, “I think that last arrest pretty much woke him up. ", "It got his attention as far as what could’ve happened as far as playing in the NFL, I think he realized his career could be over in an instant.”", "\n\nRobby can be an unrestricted free agent in 2020. ", "As a restricted free agent he signed a 1 year $3.11 million contract this year. ", "He has worked hard to get to this point in his life so it seems he finally realized what was at stake for him.", "\n\n“I worked hard my whole life to be something and see new things and better places, and be great to build a legacy and do amazing things. ", "I don’t want to be stuck or slowed down and fall into bad situations. ", "I don’t want to be known or remembered as somebody that went to jail and his career went downhill. ", "I refuse to let that happen to me ever again. ", "Period.”", "\n\nRobby says he understands now how important his decisions in life are now, “I really think about things before I do them. ", "No discredit from where I come from, but I made it out of certain things. ", "Not everything deserves your attention. ", "Sometimes it’s the simple thing of involving yourself in other people’s problems. ", "It kind of pulls you back. ", "As a man, I don’t deserve to keep getting tugged and pulled. ", "But I don’t want to blame anything on anybody.”", "\n\nThe additions to the Jets offense has also gotten the attention of Robby, plus all of it’s possible ramifications. “", "Well, they can’t double team me no more and they can’t stay over the top in coverage because if they do, they’re going to get loose underneath too. ", "It’s going to be explosive. ", "It’s what we’ve been waiting for. ", "I think everybody is excited to see that we’re a legitimate team now.”", "\n\nBeing in a contract year has not escaped Robby’s mind either. ", "With the possibility of a huge year and with it a huge contract, he is as fixated on the future like never before, “It’s huge, my eyes are on the prize more than ever. ", "I’m just more focused than ever.”", "\n\nRobby Anderson has been the lone deep threat for the Jets the past 3 seasons in a non-vertical offense. ", "In this variable offense, the chance to take more shots downfield should be much higher than in recent memory. ", "Robby’s 155 receptions and 15 TDs in the last 3 years is decent considering he was not even close to the focal point of the passing game. ", "As I have shown you here, Robby Anderson has the chance to be a wildly dynamic player for the Jets if given the opportunity.", "\n\nI have felt the same sort of frustration with Robby’s usage as he has, well maybe not near as much as he has, “It’s frustrating when you know your capabilities and you know that you should be an at least 10 targets a game receiver, because you know the impact you have on the game. ", "And that’s not given to you. ", "And you’ve proven that.”", "\n\nHe came to camp focused this year with the hopes of this new Gase offense, but also because he felt a little cheated the way last years Bates run offense allowed him a mere 50 receptions on the year. “", "I ended the season frustrated, I didn’t take a vacation this year, so far this offseason, because I didn’t feel like I had anything to really kick my feet up about. ", "That was a down year for me last year. ", "I know my capabilities. ", "I know what I want to become, and that’s the best receiver in the NFL,” Anderson said.", "\n\nI am hoping that the Jets do something smart and lock Robby Anderson up with a new contract this offseason. ", "I have heard nothing about an extension even though Joe Douglas is on record as saying he was impressed by Anderson. ", "He watched 6 games of tape on the Jets before he interviewed for the GM job.", "\n\n“He’s a tough weapon for defenses to match up with, he can get behind you and he can challenge the defense vertically” Douglas said. ", "As the only player on the roster who can do that he is a very valuable weapon. ", "I think if the Jets wanted to they could sign an extension (with language to protect the team should Robby get in trouble again) before training camp fairly easily.", "\n\nThe Jets have a little over $28 million in cap space minus about 4 to 5 million for Quinnen Williams so if the Jets take the 3 million Anderson is earning this year and add on 4 years at $7 million a year with 40% guaranteed and a $8 million signing bonus I think that could get it done. ", "That would take Robby until he is 31 give him $11 million this year total it would be about a $6.6 million cap hit for the next 5 years which is a bargain for a dynamic receiver like Robby. ", "Plus he is entering his prime years so you are getting a discount in his prime years which is better than overpaying for a veteran or having a rookie who is learning his craft. ", "It also gives Sam a receiver he is very familiar with for the next 5 years.", "\n\nThat would leave the Jets about $18 million in cap space for the year, there will be roster moves but none that will drastically alter the salary cap. ", "That is a decent amount to roll over to next year where the Jets would have around $64 million to start the year with. ", "We can add some key pieces and extend Jamal Adams before he starts his 4th year with that money and still save enough cash to extend Sam Darnold in 2022.", "\n\nBottom line I think when you have a player who can get you easy TDs in a game where most games go down to the wire; it is essential we lock up some of our key building block players for the years ahead and bring in other like them to build upon. ", "I think Robby Anderson is a building block kind of player.", "\n\nWhat do you think?" ]
{ "pile_set_name": "OpenWebText2" }
[ 0.009900990099009901, 0, 0.010638297872340425, 0, 0.01282051282051282, 0.0273972602739726, 0.057692307692307696, 0, 0, 0, 0.026845637583892617, 0, 0.046511627906976744, 0, 0.013513513513513514, 0, 0.009523809523809525, 0, 0.02631578947368421, 0.021739130434782608, 0.02702702702702703, 0, 0.007936507936507936, 0, 0.00909090909090909, 0, 0.011904761904761904, 0.02127659574468085, 0.018867924528301886, 0, 0.01904761904761905, 0.009174311926605505, 0.025, 0.008333333333333333, 0, 0.020618556701030927, 0, 0.02564102564102564, 0.020618556701030927, 0.008547008547008548, 0.013157894736842105, 0.0196078431372549, 0.015748031496062992, 0.02127659574468085, 0.02608695652173913, 0.01694915254237288, 0, 0.007692307692307693, 0, 0.009174311926605505, 0.009259259259259259, 0.009009009009009009, 0, 0, 0.008, 0.02586206896551724, 0, 0.010638297872340425, 0, 0.018867924528301886, 0, 0, 0, 0, 0, 0.008547008547008548, 0.01, 0, 0, 0, 0.012048192771084338, 0, 0.015151515151515152, 0.021739130434782608, 0.013888888888888888, 0.011235955056179775, 0.027777777777777776, 0, 0.018518518518518517, 0, 0.020202020202020204, 0.02197802197802198, 0.006097560975609756, 0.011235955056179775, 0, 0.007518796992481203, 0.03571428571428571, 0.011627906976744186, 0.021739130434782608, 0.007352941176470588, 0.02027027027027027, 0, 0.013513513513513514, 0.025, 0.0064516129032258064, 0.011111111111111112, 0.010526315789473684, 0, 0.01282051282051282, 0, 0, 0, 0.008130081300813009, 0.007751937984496124, 0.006993006993006993, 0.017241379310344827, 0, 0.008130081300813009, 0.023529411764705882, 0.0136986301369863, 0, 0.022988505747126436, 0, 0.0058823529411764705, 0.016129032258064516, 0.00819672131147541, 0, 0.00819672131147541, 0.006289308176100629, 0.009433962264150943, 0, 0.020833333333333332, 0.01652892561983471, 0.014285714285714285, 0.00909090909090909, 0.007633587786259542, 0, 0, 0.025, 0.03076923076923077, 0, 0.03333333333333333, 0.027777777777777776, 0.015037593984962405, 0, 0, 0.012658227848101266, 0, 0.014598540145985401, 0.013333333333333334, 0, 0.007042253521126761, 0, 0.01, 0.011627906976744186, 0.017543859649122806, 0, 0, 0.0297029702970297, 0, 0, 0.025974025974025976, 0, 0.02631578947368421, 0.012048192771084338, 0.024, 0, 0.007518796992481203, 0, 0, 0, 0, 0, 0, 0.014184397163120567, 0, 0.011695906432748537, 0.05154639175257732, 0, 0.02247191011235955, 0, 0, 0.018691588785046728, 0, 0.029850746268656716, 0, 0, 0.01680672268907563, 0.006944444444444444, 0.0196078431372549, 0, 0, 0, 0, 0, 0, 0, 0.008064516129032258, 0, 0, 0, 0, 0, 0, 0.01694915254237288, 0, 0, 0, 0, 0.015625, 0, 0, 0.009433962264150943, 0, 0.007246376811594203, 0.016129032258064516, 0.0035211267605633804, 0, 0, 0.009852216748768473, 0, 0, 0, 0.023255813953488372, 0.00909090909090909, 0.017094017094017096, 0.013157894736842105, 0.007407407407407408, 0, 0.006097560975609756, 0.006896551724137931, 0.010526315789473684, 0, 0.013333333333333334, 0, 0.008403361344537815, 0.013071895424836602, 0, 0.017241379310344827, 0 ]
0.009082
5
[ "Hawaii\n\nHurricane Hector gained strength as it headed across the Pacific, threatening to hit Hawaii's Big Island and possibly its active volcano, officials said early Sunday.", "\nHector grew to a Category 4 hurricane late Saturday with sustained…\n\nPAHOA, HAWAII: Approximately 600 homes have been swallowed by lava flows from Kilauea Volcano on Hawaii's Big Island since early last month, marking its most destructive eruption in modern times, Hawaii County Mayor Harry Kim said on…\n\nHONOLULU: The Hawaii community hardest hit by the Kilauea Volcano was ordered sealed off under a strict new mandatory evacuation on Thursday as the eruption marked its fourth week with no end in sight.", "\nThe Big Island’s mayor, Harry Kim,…\n\nA tide of molten rock turned a Hawaii street into a volcanic wasteland as the number of homes destroyed by the erupting Kilauea volcano soared and authorities told residents to flee a surge of lava heading towards them.", "\n\nPAHOA, Hawaii: A rising tide of lava turned a Hawaii street into a smoking volcanic wasteland on Friday, destroying at least eight homes as residents stood on the road and watched their houses burn.", "\nThe destructive fury of the erupting…\n\nHAWAII: The Kilauea Volcano on Hawaii’s Big Island came to life on Thursday, belching ash into the sky and spewing fountains of lava in a residential area where people were ordered to leave their homes, officials said.", "\nThe eruption…\n\nHawaii is quite known for beautiful, pristine beaches. ", "But, do you know about the archipelago that is famous for its volcanoes in the US?", "\nSurprisingly, while many go to the islands to have fun on the beach, there are hundreds of travel…\n\nWASHINGTON: An alert warning of an incoming ballistic missile aimed at Hawaii was sent in error Saturday, sowing panic and confusion across the US state, which is already on edge over the risk of attack, before officials dubbed it a \"false…\n\nA tour guide in Hawaii managed to recover video of his GoPro camera being slowly enveloped by lava which continued to record video.", "\nErik Storm, owner and lead guide shared the amazing video after managing to dig the camera out of the…" ]
{ "pile_set_name": "Pile-CC" }
[ 0, 0.005917159763313609, 0.007782101167315175, 0, 0, 0, 0, 0.002109704641350211, 0.009708737864077669 ]
0.002835
5
[ "Commercial aircraft are used to transport passengers between various locations. ", "A commercial aircraft generally flies according to a predetermined flight plan between a departure airport and a destination airport. ", "The flight plan includes a path from the departure airport to the destination airport, and may also include a flight time between the locations.", "\nFor various reasons, commercial, business, and general aviation aircraft may be diverted from a flight plan. ", "For example, inclement weather may cause an air traffic controller to divert an aircraft from a flight plan. ", "Due to inclement weather (such as rain or snow), visibility at a destination airport may be limited. ", "Accordingly, an air traffic controller may then determine that separation times between landing aircraft need to be increased. ", "As another example, flight congestion at a destination airport may also cause the air traffic controller to divert an aircraft from a flight plan into a holding pattern.", "\nAn aircraft may be diverted into a holding pattern, which deviates from the flight plan, in order to accommodate landing delays at a particular destination airport, whether due to inclement weather, flight congestion, and/or the like. ", "Typically, an air traffic controller verbally communicates with a pilot onboard an aircraft to inform the pilot of a required landing delay, and directs the pilot to fly the aircraft in a specified holding pattern until further notice. ", "The air traffic controller communicates the particulars of the holding pattern to the pilot, who then files the aircraft according to the holding pattern. ", "Once the aircraft is diverted into the holding pattern, the pilot is typically unaware as to how long the holding pattern will last. ", "As such, the pilot may periodically contact the air traffic controller to inquire as to when the aircraft will be cleared for landing.", "\nA holding pattern is typically a fixed pattern over which an aircraft is flown. ", "The holding pattern may include a route having multiple legs and turns, each of which provides a preset distance and time of flight. ", "For example, a holding pattern may resemble a racetrack having two straight legs connected by two 180 degree turns. ", "An aircraft in the holding pattern flies each preset leg and turn over a defined time period.", "\nAs can be appreciated, the aircraft consumes fuel as it flies from a current position within the holding pattern to an exit waypoint. ", "Moreover, flying along a set holding pattern to a designated exit waypoint increases an overall time of flight." ]
{ "pile_set_name": "USPTO Backgrounds" }
[ 0, 0, 0, 0, 0, 0, 0, 0, 0, 0, 0, 0, 0, 0, 0, 0, 0, 0, 0 ]
0
5
[ "#!", "/usr/bin/env python\n\n# Copyright 2016 The Chromium Authors. ", "All rights reserved.", "\n# Use of this source code is governed by a BSD-style license that can be\n# found in the LICENSE file.", "\n\n'''Tracing controller class. ", "This class manages\nmultiple tracing agents and collects data from all of them. ", "It also\nmanages the clock sync process.", "\n'''\n\nimport ast\nimport json\nimport sys\nimport tempfile\nimport uuid\n\nimport py_utils\n\nfrom systrace import trace_result\nfrom systrace import tracing_agents\nfrom py_trace_event import trace_event\n\n\nTRACE_DATA_CONTROLLER_NAME = 'systraceController'\n\n\ndef ControllerAgentClockSync(issue_ts, name):\n \"\"\"Record the clock sync marker for controller tracing agent.", "\n\n Unlike with the other tracing agents, the tracing controller should not\n call this directly. ", "Rather, it is called via callback from the other\n tracing agents when they write a trace.", "\n \"\"\"\n trace_event.clock_sync(name, issue_ts=issue_ts)\n\n\nclass TracingControllerAgent(tracing_agents.", "TracingAgent):\n def __init__(self):\n super(TracingControllerAgent, self).__init__()\n self._log_path = None\n\n @py_utils.", "Timeout(tracing_agents.", "START_STOP_TIMEOUT)\n def StartAgentTracing(self, config, timeout=None):\n \"\"\"Start tracing for the controller tracing agent.", "\n\n Start tracing for the controller tracing agent. ", "Note that\n the tracing controller records the \"controller side\"\n of the clock sync records, and nothing else.", "\n \"\"\"\n del config\n if not trace_event.trace_can_enable():\n raise RuntimeError, ('Cannot enable trace_event;'\n ' ensure py_utils is in PYTHONPATH')\n\n controller_log_file = tempfile.", "NamedTemporaryFile(delete=False)\n self._log_path = controller_log_file.name\n controller_log_file.close()\n trace_event.trace_enable(self._log_path)\n return True\n\n @py_utils.", "Timeout(tracing_agents.", "START_STOP_TIMEOUT)\n def StopAgentTracing(self, timeout=None):\n \"\"\"Stops tracing for the controller tracing agent.", "\n \"\"\"\n # pylint: disable=no-self-use\n # This function doesn't use self, but making it a member function\n # for consistency with the other TracingAgents\n trace_event.trace_disable()\n return True\n\n @py_utils.", "Timeout(tracing_agents.", "GET_RESULTS_TIMEOUT)\n def GetResults(self, timeout=None):\n \"\"\"Gets the log output from the controller tracing agent.", "\n\n This output only contains the \"controller side\" of the clock sync records.", "\n \"\"\"\n with open(self._log_path, 'r') as outfile:\n data = ast.literal_eval(outfile.read() + ']')\n # Explicitly set its own clock domain. ", "This will stop the Systrace clock\n # domain from incorrectly being collapsed into the on device clock domain.", "\n formatted_data = {\n 'traceEvents': data,\n 'metadata': {\n 'clock-domain': 'SYSTRACE',\n }\n }\n return trace_result.", "TraceResult(TRACE_DATA_CONTROLLER_NAME,\n json.dumps(formatted_data))\n\n def SupportsExplicitClockSync(self):\n \"\"\"Returns whether this supports explicit clock sync.", "\n Although the tracing controller conceptually supports explicit clock\n sync, it is not an agent controlled by other controllers so it does not\n define RecordClockSyncMarker (rather, the recording of the \"controller\n side\" of the clock sync marker is done in _IssueClockSyncMarker). ", "Thus,\n SupportsExplicitClockSync must return false.", "\n \"\"\"\n return False\n\n # pylint: disable=unused-argument\n def RecordClockSyncMarker(self, sync_id, callback):\n raise NotImplementedError\n\nclass TracingController(object):\n def __init__(self, agents_with_config, controller_config):\n \"\"\"Create tracing controller.", "\n\n Create a tracing controller object. ", "Note that the tracing\n controller is also a tracing agent.", "\n\n Args:\n agents_with_config: List of tracing agents for this controller with the\n corresponding tracing configuration objects.", "\n controller_config: Configuration options for the tracing controller.", "\n \"\"\"\n self._child_agents = None\n self._child_agents_with_config = agents_with_config\n self._controller_agent = TracingControllerAgent()\n self._controller_config = controller_config\n self._trace_in_progress = False\n self.all_results = None\n\n @property\n def get_child_agents(self):\n return self._child_agents\n\n def StartTracing(self):\n \"\"\"Start tracing for all tracing agents.", "\n\n This function starts tracing for both the controller tracing agent\n and the child tracing agents.", "\n\n Returns:\n Boolean indicating whether or not the start tracing succeeded.", "\n Start tracing is considered successful if at least the\n controller tracing agent was started.", "\n \"\"\"\n assert not self._trace_in_progress, 'Trace already in progress.'", "\n self._trace_in_progress = True\n\n # Start the controller tracing agents. ", "Controller tracing agent\n # must be started successfully to proceed.", "\n if not self._controller_agent.", "StartAgentTracing(\n self._controller_config,\n timeout=self._controller_config.timeout):\n print 'Unable to start controller tracing agent.'", "\n return False\n\n # Start the child tracing agents.", "\n succ_agents = []\n for agent_and_config in self._child_agents_with_config:\n agent = agent_and_config.agent\n config = agent_and_config.config\n if agent.", "StartAgentTracing(config,\n timeout=self._controller_config.timeout):\n succ_agents.append(agent)\n else:\n print 'Agent %s not started.' % ", "str(agent)\n\n # Print warning if all agents not started.", "\n na = len(self._child_agents_with_config)\n ns = len(succ_agents)\n if ns < na:\n print 'Warning: Only %d of %d tracing agents started.' % (", "ns, na)\n self._child_agents = succ_agents\n return True\n\n def StopTracing(self):\n \"\"\"Issue clock sync marker and stop tracing for all tracing agents.", "\n\n This function stops both the controller tracing agent\n and the child tracing agents. ", "It issues a clock sync marker prior\n to stopping tracing.", "\n\n Returns:\n Boolean indicating whether or not the stop tracing succeeded\n for all agents.", "\n \"\"\"\n assert self._trace_in_progress, 'No trace in progress.'", "\n self._trace_in_progress = False\n\n # Issue the clock sync marker and stop the child tracing agents.", "\n self._IssueClockSyncMarker()\n succ_agents = []\n for agent in self._child_agents:\n if agent.", "StopAgentTracing(timeout=self._controller_config.timeout):\n succ_agents.append(agent)\n else:\n print 'Agent %s not stopped.' % ", "str(agent)\n\n # Stop the controller tracing agent. ", "Controller tracing agent\n # must be stopped successfully to proceed.", "\n if not self._controller_agent.", "StopAgentTracing(\n timeout=self._controller_config.timeout):\n print 'Unable to stop controller tracing agent.'", "\n return False\n\n # Print warning if all agents not stopped.", "\n na = len(self._child_agents)\n ns = len(succ_agents)\n if ns < na:\n print 'Warning: Only %d of %d tracing agents stopped.' % (", "ns, na)\n self._child_agents = succ_agents\n\n # Collect the results from all the stopped tracing agents.", "\n all_results = []\n for agent in self._child_agents + [self._controller_agent]:\n try:\n result = agent.", "GetResults(\n timeout=self._controller_config.collection_timeout)\n if not result:\n print 'Warning: Timeout when getting results from %s.' % str(agent)\n continue\n if result.source_name in [r.source_name for r in all_results]:\n print ('Warning: Duplicate tracing agents named %s.' %\n result.source_name)\n all_results.append(result)\n # Check for exceptions. ", "If any exceptions are seen, reraise and abort.", "\n # Note that a timeout exception will be swalloed by the timeout\n # mechanism and will not get to that point (it will return False instead\n # of the trace result, which will be dealt with above)\n except:\n print 'Warning: Exception getting results from %s:' % str(agent)\n print sys.exc_info()[0]\n raise\n self.all_results = all_results\n return all_results\n\n def GetTraceType(self):\n \"\"\"Return a string representing the child agents that are being traced.\"\"\"", "\n sorted_agents = sorted(map(str, self._child_agents))\n return ' + '.join(sorted_agents)\n\n def _IssueClockSyncMarker(self):\n \"\"\"Issue clock sync markers to all the child tracing agents.\"\"\"", "\n for agent in self._child_agents:\n if agent.", "SupportsExplicitClockSync():\n sync_id = GetUniqueSyncID()\n agent.", "RecordClockSyncMarker(sync_id, ControllerAgentClockSync)\n\ndef GetUniqueSyncID():\n \"\"\"Get a unique sync ID.", "\n\n Gets a unique sync ID by generating a UUID and converting it to a string\n (since UUIDs are not JSON serializable)\n \"\"\"\n return str(uuid.uuid4())\n\n\nclass AgentWithConfig(object):\n def __init__(self, agent, config):\n self.agent = agent\n self.config = config\n\n\ndef CreateAgentsWithConfig(options, modules):\n \"\"\"Create tracing agents.", "\n\n This function will determine which tracing agents are valid given the\n options and create those agents along with their corresponding configuration\n object.", "\n Args:\n options: The command-line options.", "\n modules: The modules for either Systrace or profile_chrome.", "\n TODO(washingtonp): After all profile_chrome agents are in\n Systrace, this parameter will no longer be valid.", "\n Returns:\n A list of AgentWithConfig options containing agents and their corresponding\n configuration object.", "\n \"\"\"\n result = []\n for module in modules:\n config = module.get_config(options)\n agent = module.try_create_agent(config)\n if agent and config:\n result.append(AgentWithConfig(agent, config))\n return [x for x in result if x and x.agent]\n\n\nclass TracingControllerConfig(tracing_agents.", "TracingConfig):\n def __init__(self, output_file, trace_time, write_json,\n link_assets, asset_dir, timeout, collection_timeout,\n device_serial_number, target):\n tracing_agents.", "TracingConfig.__init__(self)\n self.output_file = output_file\n self.trace_time = trace_time\n self.write_json = write_json\n self.link_assets = link_assets\n self.asset_dir = asset_dir\n self.timeout = timeout\n self.collection_timeout = collection_timeout\n self.device_serial_number = device_serial_number\n self.target = target\n\n\ndef GetControllerConfig(options):\n return TracingControllerConfig(options.output_file, options.trace_time,\n options.write_json,\n options.link_assets, options.asset_dir,\n options.timeout, options.collection_timeout,\n options.device_serial_number, options.target)\n\ndef GetChromeStartupControllerConfig(options):\n return TracingControllerConfig(None, options.trace_time,\n options.write_json, None, None, None, None,\n None, None)\n" ]
{ "pile_set_name": "Github" }
[ 0, 0.03333333333333333, 0, 0.0196078431372549, 0, 0, 0, 0, 0, 0, 0, 0.015748031496062992, 0, 0.007874015748031496, 0, 0, 0.008968609865470852, 0.005405405405405406, 0, 0.00847457627118644, 0.008888888888888889, 0, 0.008333333333333333, 0, 0.006622516556291391, 0.008928571428571428, 0.0064516129032258064, 0.004975124378109453, 0, 0, 0.014545454545454545, 0, 0, 0.006172839506172839, 0, 0.007407407407407408, 0, 0, 0, 0.012987012987012988, 0, 0.014084507042253521, 0, 0, 0.017241379310344827, 0, 0.0053475935828877, 0, 0, 0, 0, 0, 0, 0, 0, 0, 0, 0, 0.014084507042253521, 0, 0, 0.014925373134328358, 0, 0, 0, 0, 0, 0, 0, 0, 0.012658227848101266, 0.009345794392523364, 0.008695652173913044, 0, 0, 0, 0.007462686567164179, 0, 0.0033333333333333335, 0.004830917874396135, 0.00720164609053498 ]
0.003752
5
[ "Q:\n\nSignificance of Contents.xml file in eclipse plugin jar\n\nCan anyone explain me the importance of contents.xml file for eclipse plug-in development?", "\n\nA:\n\nThere is a contexts.xml file.", "\nThis contains context sensitive help used in the UI.", "\nhttp://help.eclipse.org/luna/topic/org.eclipse.platform.doc.isv/guide/ua_help_context_xml.htm?cp=46_0_19_1_2_1\n\n" ]
{ "pile_set_name": "StackExchange" }
[ 0.006622516556291391, 0, 0.018867924528301886, 0.008849557522123894 ]
0.008585
5
[ "Olivos: hizo explotar un camión porque lo habían estacionado en la puerta de su casa\n\nParece una escena de la película \"Relatos Salvajes\". ", "Harto de que le estacionen en la puerta de su casa, un hombre hizo explotar un camión que impedía su salida. ", "Ocurrió en Olivos y el dueño del vehículo incendiado denuncia al agresor.", "\n\nEl hecho ocurrió el domingo por la madrugada en la calle General Bernardo Ohiggins al 3200, Olivos. ", "Allí, Juan Carlos Rodríguez había esacionado su camión de mudanzas sin saber con qué se encontraría horas más tarde.", "\n\n\"A él le molestó que yo deje el camión ahí. ", "En la cuedra no hay mucho lugar para estacionar y esa noche había un cumpleaños cerca. ", "Él ya me había dicho una vez que no deje el camión ahí. ", "Un vez había llamado all patrullero para que lo corra\", relató Juan Carlos a C5N.\n\nY la promesa se cumplió. ", "Cerca de las 3 de la mañana del domingo, y según el relato de algunos testigos, el hombre sacó una garrafa de gas de su casa, la depositó debajo del camión, abrió la llave y lanzó un fósforo.", "\n\n\"Yo me había ido a dormir y me despertó la explosión. ", "Cuando salí no se venía nada en toda la cuadra. ", "Era impresionante el fuego y el humo\", relató Rodríguez, quien denunció a su vecino por lo ocurrido.", "\n\n\"Me dejó en la calle. ", "Yo tenía todos mis ahorros, todos mis pesos puestos uno por uno en mi camión. ", "El seguro no me cubre nada\", se lamentó el hombre." ]
{ "pile_set_name": "OpenWebText2" }
[ 0.02158273381294964, 0.045871559633027525, 0.0136986301369863, 0.00980392156862745, 0.02586206896551724, 0, 0.011494252873563218, 0, 0.018518518518518517, 0.020942408376963352, 0, 0, 0.03, 0, 0.01282051282051282, 0.02 ]
0.014412
5
[ "Gustave Francq\n\nGustave Francq (March 1871 – January 2, 1952) was a typographer, printer, trade unionist and journalist in Canada.", "\n\nBiography \n\nBorn in Brussels, Belgium, the son of Benoît Francq and Henriette-Julie-Marie-Anne Cruks, Francq arrived in Québec in 1886 and worked as an apprentice typographer. ", " He lived for a time in Lowell, Massachusetts and in Brussels before settling in Montréal in 1900.", "\n\nIn 1902, Francq founded the Imprimerie Mercantile and was its director until 1949. ", "In 1902, he became a member of l'Union typographique Jacques-Cartier.", "\n\nHe was the secretary of the Parti ouvrier (Labour Party) and, in the 1908 Quebec general election, he was candidate for this party in the district of Hochelaga.", "\n\nIn 1909, he became president of the Conseil des métiers et du travail de Montréal. ", "From 1909 until 1911, he was vice-president of the Trades and Labour Congress of Canada.", "\n\nIn 1916, he founded the publication Le Monde ouvrier / The Labour World, which published articles about trade unionism and social issues and promoted reforms of the electoral system and the creation of programs of unemployment insurance, old age pensions and health insurance.", "\n\nHe was close to the progressive wing of the Liberal Party.", "\n\nFrom 1925 until 1937, he was president of the Quebec Commission of minimum wage of women (Commission du salaire minimum des femmes du Québec). ", "From 1939 until 1944, he was vice-president of the Commission of minimum wage (Commission du salaire minimum).", "\n\nIn 1937, he participated in the creation of the Fédération provinciale du travail du Québec.", "\n\nHe was a member of the masonic Lodge l'Émancipation from 1908 until 1910 and he founded the Lodge Force et courage.", "\n\nPersonal life \nHe married Léda Fournier in 1891 in Québec.", "\n\nHonours \nHe was designated a Person of National Historic Significance by the government of Canada.", "\n\nReferences\n\nBibliography \nGeoffrey Ewen, The ideas of Gustave Francq as expressed in Le Monde ouvrier / The Labour World, MA thesis, University of Ottawa, 1981.", "\nAndré E. Leblanc, Gustave Francq : un pionnier du mouvement syndical au Québec, Fédération des travailleurs et travailleuses du Québec, Montréal, 1991.", "\nÉric Leroux, La carrière polyvalente de Gustave Francq, figure marquante du syndicalisme international au Québec (1871–1952), Ph.D. thesis, Université de Montréal, Montréal, 1999.", "\nÉric Leroux, Gustave Francq. ", "Figure marquante du syndicalisme et précurseur de la FTQ, VLB Éditeur, 2001, 371 p.\nSelected texts of Gustave Franck, most of them originally published in Le Monde ouvrier, republished in Éric Leroux, La pensée de Gustave Francq, syndicaliste et réformateur social. ", "Textes choisis, 1905-1948, collection « Études et documents », Regroupement des chercheurs en histoire des travailleurs québécois, 2001.", "\nGeorges Massé, Gustave Francq, Fédération des travailleurs et travailleuses du Québec, 2002.", "\nJacques Rouillard, Histoire du syndicalisme au Québec, Boréal, 1989.", "\n\nExternal links\n \n\nCategory:1871 births\nCategory:1952 deaths\nCategory:Belgian emigrants to Canada\nCategory:Canadian Freemasons\nCategory:Canadian typographers\nCategory:Journalists from Quebec\nCategory:Persons of National Historic Significance (Canada)\nCategory:People from Brussels\nCategory:People from Montreal\nCategory:Trade unionists from Quebec\nCategory:Trades and Labor Congress of Canada people" ]
{ "pile_set_name": "Wikipedia (en)" }
[ 0, 0.011235955056179775, 0, 0.011764705882352941, 0.014492753623188406, 0.018518518518518517, 0.011764705882352941, 0.011363636363636364, 0, 0.016666666666666666, 0.013793103448275862, 0.00909090909090909, 0.010638297872340425, 0.017094017094017096, 0.03333333333333333, 0, 0.018518518518518517, 0.02631578947368421, 0.027777777777777776, 0.06666666666666667, 0.011278195488721804, 0.007352941176470588, 0.043010752688172046, 0.028985507246376812, 0 ]
0.016387
5
[ "Modulation of phosphatidylcholine synthesis in vitro. ", "Inhibition of diacylglycerol cholinephosphotransferase and lysophosphatidylcholine acyltransferase by centrophenoxine and neophenoxine.", "\n1,2-Diacyl-sn-glycerol : CDPcholine cholinephosphotransferase (EC 2.7.8.2) and acyl-CoA : 1-acyl-sn-glycero-3-phosphocholine acyltransferase (EC 2.3.1.23) activities of rat liver microsomes can be inhibited by centrophenoxine (N,N-dimethylaminoethyl p-chlorophenoxyacetate). ", "This inhibition is brought about by the intact centrophenoxine molecule rather than by the products of hydrolysis. ", "A nonhydrolyzable ether analog of centrophenoxine was synthesized (neophenoxine; N,N-dimethylaminoethyl p-chlorophenoxyethyl ether) and proved most effective in inhibiting the two routes of phosphatidylcholine biosynthesis. ", "While 50% inhibition of the cholinephosphotransferase was attained at 5 mM neophenoxine, 50% inhibition of the acyltransferase required 0.6 mM neophenoxine levels only. ", "Inhibition of the cholinephosphotransferase (Ki approximately 1.5 mM) and the acyltransferase (Ki approximately 1 mM) by neophenoxine was shown to be noncompetitive. ", "Other membrane-bound enzymes, such as glucose-6-phosphatase, monoacylglycerol lipase, alkaline phosphatase or phospholipase A2 were not affected by the inhibitors. ", "Because of this specificity, and because of the high affinity of the microsomal membrane for such agents, centrophenoxine and neophenoxine should prove useful for controlling phosphatidylcholine synthesis and for modulating the phosphatidylcholine deacylation-reacylation cycle." ]
{ "pile_set_name": "PubMed Abstracts" }
[ 0, 0, 0.010869565217391304, 0, 0, 0.005917159763313609, 0.018072289156626505, 0.006097560975609756, 0 ]
0.004551
5
[ "Over the past decade in the Premier League a huge influx of foreign owners has helped transform many of England’s biggest clubs.", "\n\nSome purists argue that foreign owners don’t understand the ethos of certain teams and are not sensitive to the needs of the fans, others believe the endless millions that wealthy benefactors, who hail from outside the UK, pour into the teams and the country itself is of huge benefit to the English people.", "\n\nOne of the largest shareholders in Arsenal Football Club, Alisher Usmanov, is of the latter opinion.", "\n\nMr. Usmanov has a personal wealth of over $20 billion and has tried to increase his 30 percent stake in Arsenal only for American majority shareholder Stan Kroenke to brush him aside. ", "However, with the growing band of foreign owners in the PL set to increase in the future, Usmanov believes it is benefiting the league.", "\n\n“Everyone who has legal right to buy something can perform this right,” Usmanov told BBC Sport. “", "We have Kroenke, [Aston Villa’s Randy] Lerner, many, many people and now many Asian owners. ", "I think everyone is comfortable if the club are successful. ", "If the club are not successful then, of course, everybody wants to find somebody guilty.”", "\n\nRussian billionaire Roman Abramovich bought Chelsea in 2003 and that started a wave of foreign investment in the PL as American Malcolm Glazer took over at Manchester United, Sheikh Mansour from Abu Dhabi bought Manchester City, Aston Villa were bought by American Randy Lerner and now the list goes on and on.", "\n\nUsmanov wants that to continue and doesn’t think it’s doing anybody any harm as more money floods in from overseas.", "\n\nCardiff’s owner Vincent Tan changed the clubs color from blue to red to help sales in his homeland of Malaysia. ", "Many loyal fans walked away.", "\n\n“I’m sure it is good [for English football],” Usmanov said. “", "When investors come and invest in the economy, in British football, it is a big part of the entertainment economy in the United Kingdom. ", "What’s bad?”", "\n\nAs it stands ten (Manchester United, Manchester City, Liverpool, Arsenal, Aston Villa, Sunderland, Southampton, Cardiff City, Chelsea and Fulham) of the PL’s 20 teams are owned by non-British companies or individuals. ", "Many fans of those clubs have claimed struggles with their identity as new owners try new ideas, re-brand the team in different ways and try to make their new ‘business’ as profitable as possible.", "\n\nBut can fans really complain about changes in color or traditions? ", "After all, these wealthy individuals are allowing the side they support to thrive and compete in the most lucrative and most watched league in the world.", "\n\nSomething has to give and ultimately, money talks.", "\n\nWhether that’s with finances from England or finances from foreign countries, the Premier League clubs involved are benefiting tremendously." ]
{ "pile_set_name": "Pile-CC" }
[ 0.0078125, 0, 0.0196078431372549, 0.016129032258064516, 0.007407407407407408, 0.020202020202020204, 0.03260869565217391, 0, 0, 0.02564102564102564, 0, 0.008771929824561403, 0, 0, 0, 0, 0.022727272727272728, 0, 0, 0, 0, 0.007042253521126761 ]
0.007634
5
[ "Luffs from Somnia\n\nHai there you special snowflakes,\n\n♥ Some more new releases!", "\n\n\n\nSuper cute belted mini skirts!", "\n\n\n\nI Love these high waisted pants!", "\n\n\n\nThis was also the gift for reachign 2000 members but I was asked if I would put it out for sale because some people missed it!", "\n\nEach set only 200L\n\nOnly at Somnia Main Store\n\n♥ Somnia is part of Lazy Sunday this week!! ", "So you can grab this new set for only 75L until Monday!", "\n\n\n\nThis might be my fav set of the week! ", "You can wear the Cardigan separately but not the shirt as it is just a shell. ", "But they are not linked together for maximum mixability!", "\n\nOnly at Somnia Main Store\n\n♥ There are MORE lucky boards!!! ", "Every style is in its own board with all the colors of that style. ", "They are set on a 2 minute timer with frequent wildcards as well to help you collect them all! ", "Below are a few colors of each style please check here for all the colors!", "\n\n\n\nStarting next week I will be retiring some of the lucky boards. ", "So each board will have 30 days before I retire them. ", "I am debating what I may do with them afterwards if you have any sugguestions please feel free to drop them in the sugguestion box on the table in the center of the store.", "\n\nOnly at Somnia Main Store\n\nAll my love and thanks ❤\n\nSanura" ]
{ "pile_set_name": "OpenWebText2" }
[ 0.012658227848101266, 0, 0, 0, 0.010752688172043012, 0, 0, 0.01282051282051282, 0, 0, 0, 0, 0, 0, 0, 0, 0 ]
0.002131
5
[ "Space Exploration News\n\nThe latest news about space exploration and technologies,\nastrophysics, cosmology, the universe...\n\nPosted: Aug 06, 2015\n\nFrom a million miles away, satellite camera shows moon crossing face of Earth (w/video)\n\n(Nanowerk News) A NASA camera aboard the Deep Space Climate Observatory (DSCOVR) satellite captured a unique view of the moon as it moved in front of the sunlit side of Earth last month. ", "The series of test images shows the fully illuminated \"dark side\" of the moon that is never visible from Earth.", "\n\nThis image shows the far side of the moon, illuminated by the sun, as it crosses between the DSCOVR spacecraft's Earth Polychromatic Imaging Camera (EPIC) camera and telescope, and the Earth - one million miles away. (", "Image: NASA/NOAA) (click on image to enlarge)\n\nThe images were captured by NASA's Earth Polychromatic Imaging Camera (EPIC), a four megapixel CCD camera and telescope on the DSCOVR satellite orbiting 1 million miles from Earth. ", "From its position between the sun and Earth, DSCOVR conducts its primary mission of real-time solar wind monitoring for the National Oceanic and Atmospheric Administration (NOAA).", "\n\nEPIC maintains a constant view of the fully illuminated Earth as it rotates, providing scientific observations of ozone, vegetation, cloud height and aerosols in the atmosphere. ", "Once EPIC begins regular observations next month, the camera will provide a series of Earth images allowing study of daily variations over the entire globe. ", "About twice a year the camera will capture the moon and Earth together as the orbit of DSCOVR crosses the orbital plane of the moon.", "\n\nThese images were taken between 3:50 p.m. and 8:45 p.m. EDT on July 16, showing the moon moving over the Pacific Ocean near North America. ", "The North Pole is in the upper left corner of the image, reflecting the orbital tilt of Earth from the vantage point of the spacecraft.", "\n\nThe far side of the moon was not seen until 1959 when the Soviet Luna 3 spacecraft returned the first images. ", "Since then, several NASA missions have imaged the lunar far side in great detail. ", "The same side of the moon always faces an earthbound observer because the moon is tidally locked to Earth. ", "That means its orbital period is the same as its rotation around its axis.", "\n\nIn May 2008 NASA's Deep Impact spacecraft captured a similar view of Earth and the moon from a distance of 31 million miles away. ", "The series of images showed the moon passing in front of our home planet when it was only partially illuminated by the sun.", "\n\nEPIC's \"natural color\" images of Earth are generated by combining three separate monochrome exposures taken by the camera in quick succession. ", "EPIC takes a series of 10 images using different narrowband spectral filters -- from ultraviolet to near infrared -- to produce a variety of science products. ", "The red, green and blue channel images are used in these color images.", "\n\nCombining three images taken about 30 seconds apart as the moon moves produces a slight but noticeable camera artifact on the right side of the moon. ", "Because the moon has moved in relation to the Earth between the time the first (red) and last (green) exposures were made, a thin green offset appears on the right side of the moon when the three exposures are combined. ", "This natural lunar movement also produces a slight red and blue offset on the left side of the moon in these unaltered images.", "\n\nThis animation features actual satellite images of the far side of the moon, illuminated by the sun, as it crosses between the DSCOVR spacecraft's Earth Polychromatic Imaging Camera (EPIC) and telescope, and the Earth - one million miles away.", "\n\nThe lunar far side lacks the large, dark, basaltic plains, or maria, that are so prominent on the Earth-facing side. ", "The largest far side features are Mare Moscoviense in the upper left and Tsiolkovskiy crater in the lower left. ", "A thin sliver of shadowed area of moon is visible on its right side.", "\n\n\"It is surprising how much brighter Earth is than the moon,\" said Adam Szabo, DSCOVR project scientist at NASA's Goddard Space Flight Center in Greenbelt, Maryland. \"", "Our planet is a truly brilliant object in dark space compared to the lunar surface.\"", "\n\nOnce EPIC begins regular observations next month, NASA will post daily color images of Earth to a dedicated public website. ", "These images, showing different views of the planet as it rotates through the day, will be available 12 to 36 hours after they are acquired.", "\n\nDSCOVR is a partnership between NASA, NOAA and the U.S. Air Force with the primary objective of maintaining the nation's real-time solar wind monitoring capabilities, which are critical to the accuracy and lead time of space weather alerts and forecasts from NOAA." ]
{ "pile_set_name": "Pile-CC" }
[ 0.009478672985781991, 0, 0.013636363636363636, 0.021929824561403508, 0.0111731843575419, 0.005555555555555556, 0.006369426751592357, 0.007575757575757576, 0, 0, 0, 0.012195121951219513, 0, 0, 0.015151515151515152, 0, 0.006896551724137931, 0.006289308176100629, 0, 0, 0, 0, 0.012244897959183673, 0, 0.008928571428571428, 0, 0.017857142857142856, 0, 0.015873015873015872, 0, 0.015037593984962405 ]
0.006006
5
[ "This feels like a simple enough question but answering it requires a check on how scientists define ‘species,’ a look at the phylogeny of crows and ravens (that’s the study of the evolutionary development and diversification of species i.e. the study of the tree of life), as well as an understanding of how their biology determines whether “Can they hybridize?” ", "turns into “Do they hybridize?”", "\n\nFor decades, the textbook standard for defining the term ‘species’ in introductory biology classes has been the biological species concept. ", "First coined in 1942 by ornithologist Ernst Mayr, this is the idea that a species is a population of animals that share a gene pool, and can successfully mate with each other, but not with animals outside their gene pool. ", "Almost always, the pudding offered as proof is a mule; the sterile offspring of a breeding attempt between a horse and an ass. “", "See,” your teacher would say, “The offspring are sterile because it’s the result of two different species,” and then they would slap their hands together like a satisfied chef. ", "But of course, nature is rarely so inclined to follow such rules.", "\n\nIn fact, lots of animals we consider distinct species can and do interbreed. ", "Possibly even more problematic is that there are plenty of living things that don’t reproduce sexually at all. ", "So how do you define them under this concept? ", "These issues have left scientists jaded and unsatisfied with the biological species concept and given rise to new classification schemes that don’t consider sex. ", "One of the most robust rivals to the biological concept is the phylogenetic species concept, where species are defined by their evolutionary histories. ", "The problem with this concept though is that sometimes the amount of hair splitting that goes on divides formally unified species beyond what is perhaps biologically appropriate. ", "Meaning, if two species have a distinct evolutionary history, but fulfill the same ecological role, look the same, and freely interbreed, are they really distinct species? ", "Without going too far further down this rabbit hole suffice it to say that there still remains no universally accepted scientific definition of a species, and that more than likely the most comprehensive approach is one that takes into consideration an each organism’s unique situation and leans on multiple concepts as appropriate.", "\n\nThis is relevant to American crows and common ravens because knowing that they are two different species might incline some people to believe that the answer to the question “Can they interbreed?” ", "is, by definition, “No.” ", "However, hopefully now it’s clear that such dogmatism will only hamper our understanding of the situation. ", "To advance further on this question, we instead need turn our attention to the tree of life.", "\n\nFor animals to freely interbreed, they generally need to share a very recent relative. ", "Given their geographical overlap and how similar American crows and common ravens look, it might be tempting to assume they must be quite closely related, but of course by now you know what assuming will get you. ", "As you can see from the phylogentic tree below, you need to go back four ancestors (or about 7 million years) to find the relative shared by both birds.1 American crows are actually more closely related to the collared crows of China than they are to common ravens. ", "So, while we haven’t ruled on our original question one way or another just yet, this suggests it shouldn’t happen extremely freely.", "\n\nFor the last piece we need to consider the real life, biological relationship between these birds. ", "Most crucially, we need to ask if, when in close proximity, they generally tolerate each other and socialize.", "\n\nA paper out earlier this year by Freeman and Miller answers this question with a resounding, “No.", "”2 Thanks to thousands of observations provided by citizen scientists, the authors were able to show that most interactions between the two are aggressive, and that crows are almost always the aggressor, particularly during the reproductive period. ", "This is thanks in no small part to the fact that ravens will depredate crow nests. ", "Therefore, come breeding time, crows will be most anxious to evict ravens, not bed them.", "\n\nWith all this in mind, it seems we can finally conclude that the most informed answer would be, “Ravens and crows do not hybridize…\n\n…most of the time.”", "\n\nOk time for me to come clean. ", "The truth is that I’ve buried the lead a little bit here, because in fact we know the answer to the question “can they hybridize.” ", "In 1990’s Beth Jefferson documented a successful breeding attempt between a wild American crow and common raven in Toronto, Canada.3 In 1990, the single raven started showing up in the area; a surprise to the local birders since they didn’t usually encounter ravens for another 145km north. ", "Over the course of the next three years, the raven appeared to be buddying up to a crow in the area. ", "It wasn’t until 1993, however, that Beth and other locals were able to document the pair nest building, attending to nestlings, and successfully fledgling two young.", "\n\nSo why on earth did I drag this out so much? ", "Because your take home message should not be that crows and ravens are going buck wild making little cravens. ", "Just as one swallow does not a summer make, one or two craven babies does not a habit make. ", "By and large, American crows and common ravens are reproductively isolated and do not hybridize. ", "But under the strangest of circumstances there’s no questioning that…\n\nLiterature cited\n\nJønsson K.A., Fabre P.H., and Irestedt, M. 2012. ", "Brains, tools innovations and biogeography in crows and ravens. ", "BCM Evolutionary Biology 12\n\nhttps://bmcevolbiol.biomedcentral.com/articles/10.1186/1471-2148-12-72 Freeman B.G. and Miller, E.T. 2018. ", "Why do crows attack ravens? ", "The roles of predation threat, resource competition, and social behavior. ", "The Auk 135: 857-867\n\nhttp://www.bioone.org/doi/full/10.1642/AUK-18-36.1 Jefferson E.A. 1994. ", "Successful hybridization of common raven and American crow. ", "Ontario Birds. ", "https://sora.unm.edu/sites/default/files/27-35%20notes%20OB%20Vol12%231%20Apr1994.pdf" ]
{ "pile_set_name": "OpenWebText2" }
[ 0, 0, 0, 0.0045045045045045045, 0, 0, 0, 0, 0.018018018018018018, 0, 0, 0, 0, 0, 0, 0, 0, 0, 0, 0, 0, 0, 0, 0, 0, 0.020202020202020204, 0, 0, 0, 0, 0, 0, 0.003436426116838488, 0, 0.006060606060606061, 0, 0, 0, 0, 0.021739130434782608, 0, 0.029411764705882353, 0, 0, 0.02127659574468085, 0, 0.06666666666666667, 0.011764705882352941 ]
0.004231
5
[ "Q:\n\nHow to determine if a method in a particular class is being called from a JAR file\n\nI have a jar file, it has its own source and object code, but is also bundled with its dependent jars within the master jar file\nI want to find out if a given method in a given Class is being called by the jar, and which of the dependent jars bundled within the master jar contains that class.", "\nSpecifically, I want to find out if the code is calling the getProviderVersion from either MQCOnnectionData or ConnectionData class\nSo I want to see if the code uses which or either of these, and which of the dependent jars within the master jar contains the methods \n\nA:\n\nStrange question, but you can set breakpoint on method getProviderVersion() and run debugger. ", "If the program will stop on the method, check whether the call was from interests you jar.", "\n\n" ]
{ "pile_set_name": "StackExchange" }
[ 0.0026246719160104987, 0.002717391304347826, 0, 0 ]
0.001336
5
[ "Cell output, cell cycle duration and neuronal specification: a model of integrated mechanisms of the neocortical proliferative process.", "\nThe neurons of the neocortex are generated over a 6 day neuronogenetic interval that comprises 11 cell cycles. ", "During these 11 cell cycles, the length of cell cycle increases and the proportion of cells that exits (Q) versus re-enters (P) the cell cycle changes systematically. ", "At the same time, the fate of the neurons produced at each of the 11 cell cycles appears to be specified at least in terms of their laminar destination. ", "As a first step towards determining the causal interrelationships of the proliferative process with the process of laminar specification, we present a two-pronged approach. ", "This consists of (i) a mathematical model that integrates the output of the proliferative process with the laminar fate of the output and predicts the effects of induced changes in Q and P during the neuronogenetic interval on the developing and mature cortex and (ii) an experimental system that allows the manipulation of Q and P in vivo. ", "Here we show that the predictions of the model and the results of the experiments agree. ", "The results indicate that events affecting the output of the proliferative population affect both the number of neurons produced and their specification with regard to their laminar fate." ]
{ "pile_set_name": "PubMed Abstracts" }
[ 0, 0, 0, 0, 0, 0, 0, 0 ]
0
5
[ "Q:\n\nWhat encoding is used by functions such as glGetString and glGetShaderInfoLog\n\nOpenGL has a few functions like glGetString and glGetShaderInfoLog that return strings. ", "What form of text encoding is used for these?", "\nI assume, given that they're returned as a GLchar*, that it is ASCII encoded text contained in the return value but is this specified anywhere?", "\nAs a second and related point, what text encoding is expected by functions such as glShaderSource and glBindAttribLocation. ", "Do GLSL programs have to be encoded in ASCII or can it be UTF-8?", "\n\nA:\n\nOpenGL 4.3 Compatibility Profile Specification, section 22.2, page 627:\n\nString queries return pointers to UTF-8 encoded, null-terminated static strings describing properties of the current GL context.", "\n\nAs far as I can see the spec doesn't mention the encoding of glGetShaderInfoLog().", "\n\n" ]
{ "pile_set_name": "StackExchange" }
[ 0, 0, 0.006944444444444444, 0, 0.015625, 0.004830917874396135, 0, 0 ]
0.003425
5
[ "The Dot – You are More Amazing Than You Know – Part 3 One day, as the first dot leaps up off the paper, he pauses for a moment and looks down.", "\n\nThe Dot – You are More Amazing Than You Know – Part 3\n\nOne day, as the first dot leaps up off the paper, he pauses for a moment and looks down. ", "He realizes he’s not just jumping up and down – he is up and down. ", "He has height! ", "He has height and depth and an inside and an outside. ", "He’s not just a flat two-dimensional being. ", "He’s not just a dot on a piece of paper jumping into a three-dimensional world. ", "He is a three-dimensional being that exists in a three-dimensional world. ", "He realizes that he had always been that and he had only been experiencing a portion of who he truly was. ", "He now had a whole new world to explore as a liberated three-dimensional being.", "\n\n***\n\nToday, we perceive ourselves as three-dimensional beings. ", "We’ve been taught that reality is three-dimensional—and in fact, this fits with what our physical senses tell us so usually we have no reason to question it. ", "Everything we see in our world appears three-dimensional: with height, width, and depth. ", "We can move back and forth, side to side, up and down, in and out. ", "But . . . ", "are we actually more than this? ", "Do we have a limited perception of reality?", "\n\nThe astonishing truth is that you are not just a three-dimensional being." ]
{ "pile_set_name": "Pile-CC" }
[ 0, 0, 0, 0, 0, 0, 0, 0, 0, 0, 0, 0, 0, 0, 0, 0, 0, 0 ]
0
5
[ "Rejecting speculations over the emergence of a hung Assembly after May 12 elections, Amit Shah said that the BJP will form next government on its own and there was no question of either taking or giving support to anybody.", "\n\nBJP President Amit Shah on Thursday claimed that his party will form the next government in Karnataka by winning more than 130 seats in upcoming Assembly elections in the state.", "\n\nRejecting speculations over the emergence of a hung Assembly after May 12 elections, Amit Shah said that the BJP will form the next government in the state on its own and there was no question of either taking or giving support to anybody.", "\n\nThe BJP president accused the Congress of taking support of traitors for winning elections.", "\n\n\" We can lose elections but can't sit with Social Democratic Party of India (SDPI) and Popular Front of India (PFI). ", "Congress takes support of SDPI and PFI to win elections and that's the difference between them and BJP. ", "Congress doesn't refrain from taking support of traitors,\" Amit Shah told media persons In Bengaluru.", "\n\nHe also alleged that the Siddaramaiah government of Karnataka is the most unsuccessful government in India since independence.", "\n\nRefering to the killing of BJP and RSS workers in Karnataka, Shah alleged that Law and Order has seen a downfall in the state.", "\n\n\"Law and order have seen a downfall in Karnataka. ", "More than 24 workers of BJP & RSS were brutally killed in the state and Congress considers it a part of politics. ", "No efforts were made to nab the culprits,\" Shah said.", "\n\nWatch video: BJP president Amit Shah attack Congress over law and order situation in Karnataka" ]
{ "pile_set_name": "Pile-CC" }
[ 0.009009009009009009, 0.01675977653631285, 0.008298755186721992, 0.021505376344086023, 0.025210084033613446, 0.028846153846153848, 0.009900990099009901, 0, 0.0234375, 0, 0.017543859649122806, 0.018867924528301886, 0.03125 ]
0.016202
5
[ "Australia urgent plea to shear overgrown sheep Published duration 2 September 2015\n\nimage copyright AFP image caption The merino sheep is \"four-to-five times its normal size\", animal welfare officials say\n\nAustralia's national shearing champion has been urgently called in to help after a heavily overgrown sheep was found near the capital Canberra.", "\n\nIan Elkins responded after animal welfare officials warned the life of the merino sheep could be in danger because it was so woolly.", "\n\nSheep can develop serious health issues if they are not regularly shorn.", "\n\nThe animal - believed to be male - was \"four-to-five times its normal size,\" welfare official Tammy Ven Dange said.", "\n\n\"It's definitely one of the biggest sheep we've ever seen,\" Ms Ven Dange, head of the RSPCA in the Australian Capital Territory, was quoted as saying by the AFP news agency.", "\n\nShe added that the animal was \"pretty stressed out\" around human beings after what was probably years of solitude.", "\n\nimage copyright AFP image caption The animal is believed to have spent a number of years on the loose\n\nRSPCA officials had initially wanted to shear the animal themselves, but later decided to wait for an expert.", "\n\nMr Elkins, a four-time Australian Shearing Championship winner, said removing the fleece \"could be one of my biggest challenges yet\".", "\n\nIn 2004, Shrek the New Zealand sheep was found after six years on the loose.", "\n\nThe animal - also a merino - later lost his giant 27kg (60lb) fleece in a televised shearing broadcast live around the world.", "\n\nThe country's most famous sheep died in June 2011." ]
{ "pile_set_name": "OpenWebText2" }
[ 0.0028653295128939827, 0.014925373134328358, 0, 0.008547008547008548, 0.011428571428571429, 0, 0.004672897196261682, 0, 0, 0, 0 ]
0.003858
5
[ "Ming Yang — who in the past has been described as a “fearsome tiger mother” and reportedly instructed her son, Sun, to smash open a box containing his blood — has provided a spicy twist in the doping scandal that’s rocked swimming.", "\n\nAccording to reports in the Chinese media picked up by Swimming World Magazine, Ming Yang has levelled serious accusations at the Chinese Swimming Association (CSA) in a since-deleted social media post.", "\n\nSun Yang, his family and his enormous supporter base in China have been devastated by the eight-year ban handed to the 28-year-old freestyle star that will effectively force him into retirement unless he can have it overturned on appeal.", "\n\nAnd it appears his mother, a former volleyball player who has been intricately involved in Sun’s career, is directing her frustrations at her home nation.", "\n\nMing has backed long-held suspicions the CSA deliberately covered up Sun’s positive test for a banned substance in 2014 so he could compete in that year’s Asian Games.", "\n\nNo one outside of China was told about trimetazidine being found in Sun’s system until November, 2014, two months after he’d won three gold medals at the event in South Korea.", "\n\nChina’s anti-doping authority (CHINADA) revealed the banned stimulant had been discovered at its national championships in May that year, but was “not very serious” and had only warranted a three-month ban, which Sun had served.", "\n\nCHINADA said it had not announced the suspension until after the Asian Games because it only reveals positive tests every three months.", "\n\n“Sun is the most famous athlete in China and is known in the world, which means we need to handle his case very cautiously. ", "This is huge bad news but we will not cover it up,” CHINADA said. “", "We announce positive cases and test statistics in our quarterly reports just as WADA requires.”", "\n\nIt smelled bad — and now Ming claims it was a deliberate attempt to ensure Sun could shine on the continental stage.", "\n\nA translation of her social media post includes her version of the message the Sun camp received from the CSA at the time.", "\n\n“If we report this offence in the way we left it, it will not be approved,” Ming alleges. “", "Now the Asian Games are over, your result will not be affected and the final outcome in this case will not be announced to the public either. ", "We can say that the penalty was for three months sometime between May and October.”", "\n\nSwimming World Magazine reported “Sun is said to have rejected the strategy but the plan was agreed to by his mother”.", "\n\nShe would later be accused of taking a leading role in the incident at Sun’s home in September, 2018, that has brought down his career after The Court of Arbitration for Sport found Sun’s camp destroyed containers holding his blood sample after a disagreement with drug testers.", "\n\n“This foolish operation, carried out in full view of drug testers, was directed by Ming Yang, a fearsome tiger mother who, since Sun was a boy, has managed every aspect of his development into the greatest freestyle swimmer of his era and a poster boy for China’s emergence as a global sporting power,” a Sydney Morning Herald report of the incident detailed.", "\n\n“It is Ming Yang who decides where her 27-year-old son trains and with whom, what he eats and when, who he sees and what he can do outside swimming.”", "\n\nIf his appeal fails, deciding what he does outside swimming will take up more time than it used to.", "\n\nWith the Tokyo Olympics just five months away, the reigning 1500m freestyle world record-holder is fighting to save his career and reputation.", "\n\nA lawyer for Sun, a hugely popular figure in China, issued a fiery statement on Saturday reiterating that he will appeal, based on “a series of procedural errors”.", "\n\n“February 28, 2020 was a dark day. ", "It shows the scene where evil defeats justice and power replaces self-evident truths,” Beijing lawyer Zhang Qihuai said in a statement.", "\n\n“On this day, CAS listened to prejudice, turned a blind eye to rules and procedures, turned a blind eye to facts and evidence, and accepted all lies and false evidence.”", "\n\nThe statement reasserted Sun’s defence that doping officials who came to his home were not qualified or authorised, and it was they who decided not to pursue testing.", "\n\nSun will sue a doping inspector who gave “false evidence”, said the lawyer, also accusing the World Anti-Doping Agency (WADA) of “distorting facts and abuse of power”.", "\n\n— with AFP" ]
{ "pile_set_name": "OpenWebText2" }
[ 0.008658008658008658, 0.014705882352941176, 0.0041841004184100415, 0.00641025641025641, 0.011834319526627219, 0.005649717514124294, 0.008695652173913044, 0.0072992700729927005, 0, 0.014925373134328358, 0.010526315789473684, 0.00847457627118644, 0.008064516129032258, 0, 0, 0, 0.008333333333333333, 0.007142857142857143, 0.008310249307479225, 0.006622516556291391, 0, 0, 0.006060606060606061, 0, 0.007407407407407408, 0.005847953216374269, 0.005952380952380952, 0.005917159763313609, 0.08333333333333333 ]
0.008771
5
[ "The youngest ever entrant was 12-year-old Jean Jacques from Monaco. ", "He represented Monaco in Eurovision 1969 with the song \"Maman, Maman\". ", "The oldest was 95-year-old Emil Ramsauer from the Swiss 2013-band \"Takasa\"" ]
{ "pile_set_name": "Pile-CC" }
[ 0.014705882352941176, 0, 0.02702702702702703 ]
0.013911
5
[ "Le Premier ministre a évoqué dans son discours la \"défiance\" des Français face à certaines mesures, dont celle-ci, vivement contestée.", "\n\nUn aparté dans le discours de restitution du grand débat national. ", "Le Premier ministre, Edouard Philippe, a regretté, lundi 8 avril, la \"défiance\" manifestée au moment de l'instauration de la limitation de vitesse à 80 km/h sur les routes secondaires à double sens. \"", "La bonne foi, c'est celle qui m'a par exemple conduit à instaurer le 80 km/h parce que je ne pouvais pas accepter l'hécatombe quotidienne qui brise des vies, des familles, des existences\", a expliqué Edouard Philippe. \"", "Je voulais sauver des vies, on m'a accusé de vouloir remplir les caisses\" de l'Etat, a-t-il regretté.", "\n\nLa limitation de vitesse à 80 km/h, entrée en vigueur le 1er juillet 2018 sur les routes secondaires à double sens sans séparateur central, a été parmi les mesures les plus contestées dans la crise des \"gilets jaunes\". \"", "Nous ne sommes pas parvenus à concilier les Français avec ceux qui les dirigent\", a poursuivi le Premier ministre, évoquant \"les mots très durs, très violents\" utilisés à l'égard du \"gouvernement, des élus ou des fonctionnaires\".", "\n\nJe ne me résigne pas à abandonner cette ambition en matière de sécurité routière. ", "C'est une grande politique publique au service de nos concitoyens, elle est indispensable.", "Edouard Philippe, Premier ministre\n\n\"J'ai dû moi-même apprendre à gouverner avec cette défiance\", a-t-il ajouté en évoquant cette mesure phare relative à la sécurité routière. \"", "Je dois apprendre à composer avec l'incompréhension ou le rejet de certains de nos concitoyens.\" ", "En janvier, lors d'un débat dans l'Eure, le président Emmanuel Macron s'était dit pour la première fois ouvert à des aménagements locaux de cette mesure, au risque de désavouer le chef du gouvernement." ]
{ "pile_set_name": "OpenWebText2" }
[ 0.007462686567164179, 0, 0.01, 0.0182648401826484, 0.019801980198019802, 0.0045045045045045045, 0.008733624454148471, 0, 0, 0.01694915254237288, 0, 0.014925373134328358 ]
0.008387
5
[ "The effect of a brominated flame retardant, tetrabromobisphenol-A, on free radical formation in human neutrophil granulocytes: the involvement of the MAP kinase pathway and protein kinase C.\nThis study investigates the effects of one of the most frequently used brominated flame-retardants (BFR), tetrabromobisphenol-A (TBBPA), on formation of reactive oxygen species (ROS) and calcium levels in human neutrophil granulocytes. ", "TBBPA enhanced ROS production in a concentration-depended manner (1-12 microM), measured as 2,7-dichlorofluorescein diacetate amplified (DCF) fluorescence. ", "The results on ROS production by TBBPA was confirmed by lucigenin-amplified chemiluminescence. ", "The TBBPA induced formation of ROS was due to activation of respiratory burst, as shown by the NADPH oxidase inhibitor DPI (10 microM). ", "TBBPA induced activation of respiratory burst was also inhibited by the MEK 1/2 inhibitor U0126 (10 microM), the PKC inhibitor BIM (0.25 microM), and the tyrosine kinase inhibitor erbstatin-A (25 microM). ", "We also found a small reduction in ROS formation in the absence of extracellular calcium and when verapamil was added. ", "The phosphorylation of ERK 1/2 was confirmed by Western blotting. ", "TBBPA also induced a concentration dependent increase in intracellular free calcium measured with Fura-2/AM. ", "We suggest that exposure of human neutrophil granulocytes to the brominated flame retardant TBBPA leads to an activation of the NADPH oxidase primarily by an ERK 1/2 stimulated pathway. ", "The data also show that PKC, calcium, and tyrosine kinases may be involved in the activation." ]
{ "pile_set_name": "PubMed Abstracts" }
[ 0.00702576112412178, 0.01282051282051282, 0, 0.014705882352941176, 0.004878048780487805, 0, 0.015151515151515152, 0.009174311926605505, 0.010752688172043012, 0 ]
0.007451
5
[ "Bone-marrow relapse in paediatric acute lymphoblastic leukaemia.", "\nMarrow relapse is the major obstacle to cure for 10-15% of young patients with acute lymphoblastic leukaemia (ALL). ", "Recent investigations into the biology of minimal residual disease indicate that many early relapses derive from residual cells present at first diagnosis, but some late relapses might represent new mutations in leukaemic cells not eliminated by conventional therapy. ", "Treatment of marrow relapse involves higher doses and more intensive schedules of the drugs used for initial therapy with or without haemopoietic stem cell transplantation. ", "In most reports, transplantation is better than continuation chemotherapy in early marrow relapse, but its role in later relapse is less clear. ", "Current therapy cures 10% of patients with early marrow relapses and 50% of those with late relapses, but outcomes have changed little in the past two decades. ", "Understanding the molecular biology of ALL underlies development of improved risk stratification and new therapies. ", "Although better drugs are needed, introduction of new agents into clinical trials in paediatric disease has been difficult. ", "Innovative trial designs and use of valid surrogate endpoints may expedite this process." ]
{ "pile_set_name": "PubMed Abstracts" }
[ 0, 0, 0, 0, 0, 0, 0, 0, 0 ]
0
5
[ "A member of an anti-Muslim group seen outside a mosque in Texas on Saturday.", "\n\nAn armed anti-Muslim protest has been held in the US state of Texas with protesters gathering outside the Nation of Islam Mosque in South Dallas.", "\n\nThe anti-Muslim demonstration by a mostly white group was held on Saturday, but the group outnumbered by counter-protesters who gathered outside the mosque.", "\n\nA few hundred South Dallas residents, mostly black, marched along Martin Luther King Jr. Boulevard to oppose the anti-Muslim demonstrators.", "\n\nDozens of police officers stood in between the two groups, trying to maintain security. ", "Some officers were also stationed on rooftops. ", "No violence or arrests were reported.", "\n\nThe anti-mosque group, that routinely protests outside mosques, showed up in camouflage, while carrying guns and an American flag.", "\n\nThe anti-Muslim demonstrators soon left and the protests ended without incident.", "\n\n“It’s a people’s victory here in South Dallas today,” said Yafeuh Balogun of the Huey P. Newton Gun Club, named for the founder of the original Black Panther Party.", "\n\nArmed with a shotgun, Krystal Muhammad stood guard with other members of the New Black Panther Party outside a mosque Saturday in South Dallas on Saturday.", "\n\n“This is what they fear — the black man,” said activist Olinka Green. “", "This is what America fears.”", "\n\nLast week, Krystal Muhammad of the New Black Panther Party told Fox4 that they \"won’t allow anybody to come in and try to intimidate our brothers and sisters.”", "\n\nSource: presstv" ]
{ "pile_set_name": "OpenWebText2" }
[ 0, 0.006802721088435374, 0, 0.0070921985815602835, 0, 0, 0, 0, 0, 0.006024096385542169, 0.012738853503184714, 0.0136986301369863, 0, 0.012422360248447204, 0 ]
0.003919
5
[ "The effects of temperature on service employees' customer orientation: an experimental approach.", "\nNumerous studies have demonstrated how temperature can affect perceptual, cognitive and psychomotor performance (e.g. Hancock, P.A., Ross, J., and Szalma, J., 2007. ", "A meta-analysis of performance response under thermal stressors. ", "Human Factors: The Journal of the Human Factors and Ergonomics Society, 49 (5), 851-877). ", "We extend this research to interpersonal aspects of performance, namely service employees' and salespeople's customer orientation. ", "We combine ergonomics with recent research on social cognition linking physical with interpersonal warmth/coldness. ", "In Experiment 1, a scenario study in the lab, we demonstrate that student participants in rooms with a low temperature showed more customer-oriented behaviour and gave higher customer discounts than participants in rooms with a high temperature - even in zones of thermal comfort. ", "In Experiment 2, we show the existence of alternative possibilities to evoke positive temperature effects on customer orientation in a sample of 126 service and sales employees using a semantic priming procedure. ", "Overall, our results confirm the existence of temperature effects on customer orientation. ", "Furthermore, important implications for services, retail and other settings of interpersonal interactions are discussed. ", "Practitioner Summary: Temperature effects on performance have emerged as a vital research topic. ", "Owing to services' increasing economic importance, we transferred this research to the construct of customer orientation, focusing on performance in service and retail settings. ", "The demonstrated temperature effects are transferable to services, retail and other settings of interpersonal interactions." ]
{ "pile_set_name": "PubMed Abstracts" }
[ 0, 0.024096385542168676, 0, 0.011111111111111112, 0, 0, 0, 0, 0, 0, 0, 0, 0 ]
0.002708
5
[ "Search\n\nSpring Allergies 101\n\nBrought to you by FLONASE®\n\nAbout Spring Allergies\n\nWhile the start of spring is pretty much the same every year (March 19th, 20th, or 21st), the start of spring allergy season is much harder to anticipate. ", "In some parts of the country, spring allergy season can start as early as February and may last well into summer!", "\n\nSo even though it may not technically be \"spring\", your sneezing, nasal congestion, itchy, watery eyes and runny nose could very well show up earlier than expected, and stick around long past their welcome. ", "But that doesn't mean you have to let spring allergy season catch you off guard!", "\n\nGet the inside scoop on spring allergies—what causes them, what they do, and more importantly, what you can do about them.", "\n\nCauses of Spring Allergies\n\nTree Pollen\n\nDuring late winter and early spring, dormant trees bounce back to life and start releasing allergy-inducing pollen into the air. ", "Some common culprits include birch, cedar, hickory and walnut.", "\n\nGrass Pollen\n\nOf all the various kinds of grasses blanketing lawns, fields and gardens, only a small number can be blamed for causing allergy symptoms. ", "Timothy, Bermuda and Kentucky Blue are some of the most common allergy-provoking grasses.", "\n\nMold\n\nMold thrives in damp indoor and outdoor environments such as leaf piles, rotten logs, basements and bathrooms. ", "Unlike pollen, mold doesn't die with the first frost; instead, it stops growing and lays dormant until the weather starts to warm.", "\n\nDust Mites\n\nDust mites, or more specifically, dust mite waste, is a common indoor allergen that can be found in many places around the house—furniture, carpets, bedding and stuffed toys.", "\n\nSymptoms\n\nOne of the most common, and most bothersome, symptoms reported for allergies is nasal congestion.", "\n\nAlso on the list are:\n\nsneezing\n\nitchy, watery eyes\n\nitchy nose\n\nrunny nose\n\nPollen Count\n\nNo matter what you're up to this spring, checking the pollen count for wherever you’re headed can make a real difference.", "\n\nSpring Cleaning\n\nThere's nothing like a fresh, clean house to mark the beginning of spring. ", "Before you get out the dust cloth and vacuum, make sure you have a strategy for how to tackle your cleaning to reduce indoor allergens.", "\n\nTreat Your Allergy Symptoms\n\nNo matter how vigilant you are, you can't get rid of spring allergens. ", "That's why it's important to treat your symptoms every day. ", "Unlike most allergy pills, once-daily FLONASE® Nasal Sprays treat nasal congestion, sneezing, runny nose, itchy nose and itchy, watery eyes. ", "With FLONASE®, you can leave your symptoms behind and get the most from the spring season." ]
{ "pile_set_name": "Pile-CC" }
[ 0, 0, 0, 0, 0, 0, 0, 0, 0.02247191011235955, 0.008403361344537815, 0, 0, 0, 0.004672897196261682, 0, 0, 0, 0, 0, 0 ]
0.001777
5
[ "Q:\n\nWhy am I getting \"Insecure operation ‘write’ at level 4\" when sandboxing code?", "\n\nI have the following Ruby code:\nTestcase.rb:\nfilename = \"/absolute/path/to/Untrusted.rb\"\nthread = Thread.new do\n $SAFE = 4\n Kernel::load(filename, true)\nend\nthread.join\n\nUntrusted.rb\n#Untrusted code\nputs \"Untrusted!\"", "\n\nHowever, I get an error when I try to run Testcase.rb:\n/Volumes/Data/Users/mike/Desktop/Testcase.rb:4:in `write': Insecure operation `write' at level 4 (SecurityError)\n from /Volumes/Data/Users/mike/Desktop/Testcase.rb:7:in `join'\n from /Volumes/Data/Users/mike/Desktop/Testcase.rb:7\n\nRemoving $SAFE=4 solves the issue, but I want to be able to safely run untrusted code. ", "Any ideas about why this isn't working?", "\n\nA:\n\nI tried your code and got the same result as you. ", " Then I changed the $SAFE level to 3, and got this warning:\nInsecure world writable dir /tmp in LOAD_PATH, mode 041777\n\nI moved the file being loaded from /tmp to a directory that isn't world-writable and the warning went away. ", " Changing the $SAFE level to 4 then worked.", "\nSo, try making sure that the loaded file's directory isn't world writable. ", " Also try a lower safe level and see if you get a useful warning.", "\n\n" ]
{ "pile_set_name": "StackExchange" }
[ 0, 0.01818181818181818, 0.007894736842105263, 0, 0, 0, 0, 0, 0, 0 ]
0.002608
5
[ "The Impressions\n\nBiography\n\nLed by the brilliant Curtis Mayfield, the Impressions spear-headed the transformation of doo-wop into Soul music. ", "Mayfield looked to the smooth strains of Gospel music rather than to other street corner vocal groups and wrote deceptively simple, stripped-down arrangements on such classics as \"It's All Right.\" ", "Always an elegant, harmony-rich outfit, the Impressions started referencing the civil rights struggle on such hits as \"People Get Ready\" and \"This is My Country\" without ever getting heavy-handed or forgetting about crafting great songs. ", "Mayfield became an R&B mover and shaker, writing and producing for other acts before going on to an impressive solo career in 1970. ", "The Impressions carried on without their leader after 1970 and their later work, while not groundbreaking, remained top-shelf Soul. ", "The Impressions left an R&B legacy equal to that of Ray Charles, Sam Cooke, Marvin Gaye or Stevie Wonder." ]
{ "pile_set_name": "Pile-CC" }
[ 0.02112676056338028, 0, 0.004201680672268907, 0.015151515151515152, 0.007575757575757576, 0.05714285714285714 ]
0.017533
5
[ "A more modern photo of the church prior to the restoration of 2017.", "\n\nHoly Family Parish in Latrobe, Pennsylvania, within the Diocese of Greensburg, recently completed a very nice restoration and de-wreckovation. ", "Under the leadership of Fr Daniel Mahoney, V.F., the parish put back the ornately patterned ceiling, a high altar, murals of varies “modern” Saints around the nave, as well as all new lighting and sound system, etc. ", "The decorative work, murals, and painting were done by EverGreene Architectural Arts; the restored churched was blessed with the dedication of the new altar by Bishop Edward Malesic on June 25, 2017. ", "Our thanks to Mr Christopher Pujol, a seminarian of the diocese of Greensburg, for sharing these photos with us, and our congratulations to Fr Mahoney and Bishop Malesic for bringing beauty back to this church. ", "Ad multos annos!The church prior to the renovations of 1967. ", "Notice the murals, ornamented ceiling, and the similarity to the newly restored high altar seen below. ", "Images of the Holy Family crown the arch.", "This is the 1967 renovation of the church as pictured in the commemorative booklet from the consecration; the high altar, pulpit, and all decorative paintings have been removed.", "The altar installed in 1967 was granite, and consecrated with the rite in the revised Pontifical of 1961, according to the commemorative booklet of the day. ", "The relic chamber can be seen in the front of the altar where the relics from the original altar were placed; these have now been moved to the new high altar. ", "The booklet from 1967 notes “This restoration observes the prescriptions given in the tradition of the Roman Church, assigning dignity to the altar by due attention to essentials and not to temporary decorations.", "”The restored church played off the original design by returning the Holy Family to the arch; Christ the Divine Teacher takes the center position, as the parish school is claimed in His name. ", "The focus again becomes the great window of the crucifixion in the apse, as well as the restored high altar with tabernacle.", "The new high altar comes from a closed church in the Archdiocese of Baltimore; it is strikingly similar to the original altar of Holy Family. ", "The relics deposited within the mensa are the same from the original high altar.", "The beginning of a celebration of Low Mass at the new High Altar, by Fr Daniel Mahoney, assisted by Mr Christopher Pujol, June 2018." ]
{ "pile_set_name": "OpenWebText2" }
[ 0, 0, 0.004629629629629629, 0.01, 0.009478672985781991, 0, 0, 0.024390243902439025, 0, 0, 0, 0.0047169811320754715, 0, 0, 0.007042253521126761, 0, 0.015151515151515152 ]
0.004436
5
[ "Tag Archives :Sullivan County\n\nThe summer may be all about the perfect beach getaway for you, and we are certainly with you on that! ", "But maybe you’re looking for something a little different, and/or want to plan ahead for the perfect place to enjoy the fall foliage? ", "If so, get in your car and take a drive to Sullivan County, NY – just two hours from New York City. ", "Gorgeous mountains, picturesque lakes, lush forest and The Delaware River are all right there for your enjoyment.", "\n\nWe’ve owned a lake house in Sullivan County for close to a decade and cannot express how much fun and enjoyment it’s brought us and our family over the years. ", "We’ve also discovered some great area attractions that made us love the area even more. ", "This is our round-up of favorite things to do, and is by no means exhaustive! ", "One of the delights of the area is taking a country drive and discovering hidden treasures: antique shops, art galleries, restaurants and more can all be happened upon…not to mention wildlife! ", "As this area of New York is right across the river from Pennsylvania, our area explorations have taken us to PA. ", "Some of the attractions I mention are in PA, but still just a short drive from Sullivan County. ", "And believe me, if I’ve included them, they are worth checking out!", "\n\nWhere To Stay\n\nOver the years, we have seen many new businesses pop up in the area. ", "Since we are city people at heart, we do enjoy having more of our creature comforts close at hand while we’re upstate. ", "One such place was opened by our dear friends, but we can say, objectively, that this place rocks. ", "Stickett Inn and Stickett Inn: The Store and Annu Juice in Barryville is a four suite bed and breakfast with a gift shop and juice bar in front. ", "The suites are huge and incredibly chic, from the Warhol on the walls to the steam shower and deep soaking tub in the bathrooms.", "\n\nIf you’re coming up with a family and need a bit more space, Home Away has many beautiful homes for rent in the area. ", "Many of the homes are on lakes and can accommodate ten or more people! ", "If you are looking for a truly special location, perhaps to host an event, you must, MUST, check out The Floating Farmhouse…just stunning!", "\n\nWhere To Eat\n\nAs a New Yawker, I’d be remiss if I didn’t mention my favorite pizza joint in the area! ", "Sitting right on the edge of big, beautiful Kauneonga Lake is Benji & Jakes. ", "Though they don’t make your traditional New York style pizza pie, their creations are delicious nonetheless. ", "They make over a dozen specialty pies, one more delicious than the next (think pesto, roasted eggplant, smoked mozzarella and more). ", "Don’t stop at the pizza, though. ", "The appetizers and specialty cocktails are not to be missed…nor is the homemade gelato!", "\n\nHenning’s Local is a recent addition to the Sullivan County scene but has already become a favorite. ", "Using locally grown ingredients, Henning’s offers a casual atmosphere with delicious and super-fresh comfort food. ", "All the fish dishes are superb, and be sure to order the quinoa cake as a side. ", "Great place for a family meal.", "\n\nIf you are looking for something more chic or a fine dining experience, cross the river to Milford, PA and visit Hotel Fauchere. ", "There are two distinct dining options at the hotel, both delicious and special in their own way. ", "Upstairs, The Delmonico Room offers fine dining, while Bar Louis is a stylish, ultra-modern environment for casual and late-night dining, live entertainment and great cocktails. ", "Never heard of sushi pizza? ", "Dig into some at Bar Louis!", "\n\nMost Importantly: What To Do\n\nPerhaps your entire itinerary consists of relaxing lakeside with a good book. ", "If you’re interested in doing some sightseeing, shopping or physical activity, read on!", "\n\nOutdoor Sporting\n\nIf you want some outdoor sporting, and if the weather is warm enough, spend the day rafting on The Delaware River. ", "Kittatinny is just one of the many places from which you can rent a raft, canoe, tube or kayak to take out on the river. ", "You can choose between a long trip or a short one (I’d stick with the short; it feels long enough). ", "Kittatinny also offers paintball and two ziplines, if you’re into that sort of thing.", "\n\nCulture & Entertainment\n\nDid you know that Woodstock didn’t happen in Woodstock, NY? ", "It happened in Bethel, NY…in Sullivan County! ", "The lush, hilly grounds of Bethel Woods are still a haven for great music, festivals and there is even a Woodstock museum on site. ", "Also of note in Bethel is Rocking Horse Ranch (literally next door to Bethel Woods) and Dancing Cat Distillery.", "\n\nShopping\n\nThere are several other towns to check out all over Sullivan County for good food and shopping. ", "Two of my favorite:\n\nSpa Retreat\n\nLast but not least, there’s the spa…and a five star one at that! ", "The Lodge at Woodloch was created by former Canyon Ranch employees and feels every bit as luxurious. ", "Woodloch is consistently chosen as one of the top ten spas in the world by Travel & Leisure and Condé Nast Traveler. ", "You can, of course, choose to stay there, but day passes and à la carte services are available to non-guests as well. ", "The facilities are top notch: imagine sitting in an outdoor hot tub as you watch deer running through the snowy forest around you. ", "This actually happened to me. ", "As a side note, soaking in an outdoor hot tub in the middle of winter only sounds crazy. ", "It’s glorious! ", "There is also an indoor pool complete with waterfall, steam room, sauna and so much more. ", "The body treatments are where the spa really shines…spring for the scrub and wrap combination treatments. ", "They are worth every penny." ]
{ "pile_set_name": "Pile-CC" }
[ 0, 0, 0.01, 0, 0, 0, 0, 0, 0, 0, 0, 0, 0, 0, 0.020689655172413793, 0.0078125, 0.008333333333333333, 0, 0.007246376811594203, 0, 0.012987012987012988, 0, 0, 0, 0, 0, 0, 0, 0, 0.007633587786259542, 0, 0.0056179775280898875, 0, 0.037037037037037035, 0, 0, 0, 0.008264462809917356, 0, 0, 0.034482758620689655, 0.021739130434782608, 0.015267175572519083, 0.018018018018018018, 0, 0, 0.009900990099009901, 0.02564102564102564, 0, 0, 0, 0, 0, 0, 0, 0 ]
0.004476
5
[ "FC Tosno\n\nFC Tosno () was a professional Russian football club based in Tosno. ", "On 6 May 2017, Tosno won promotion to the 2017–18 Russian Premier League for the first time after securing second place in the Russian National Football League. ", "They were relegated after one season at the top level. ", "On 9 June 2018 it was announced that FC Tosno was dissolved because of financial problems.", "\n\nHistory\nFC Tosno was officially registered in 2008 by Leonid Khomenko, then director of 'FC Era'. ", "In 2009, FC Tosno and another amateur club, 'Ruan', were merged. ", "The name 'Ruan' was carried by the club till late 2013.", "\n\nFoundation \nThe decision to found a professional club was taken in March 2013, with the support of the club's main sponsor at the time, a holding company named 'Fort Group', and the Regional Public Organization, 'FC Tosno'. ", "Since 2013, FC Tosno have represented the Leningrad Oblast in Russian competitions. ", "In Tosno's first professional season, the club were declared to compete in the West Zone of the Russian Professional Football League. ", "Due to Tosno's then-current home stadium (Tosno Stadium)'s failing to satisfy PFL's minimum requirements, it was required that they move to a different, more capacious stadium for them to be eligible to compete in the division. ", "In the meantime, Tosno were offered to play their home matches at the Petrovskiy Stadium.", "\n\nThe club's first official match took place on 10 July 2013, when they defeated FC Dynamo Vologda in the Round of 512 of the 2013–14 Russian Cup. ", "This victory marked the start of the club's 22-game 'unbeaten run' in all competitions (18 league games and 4 cup games), which ended on the 27th of October. ", "The following day, head coach Viktor Demidov's contact with the club had terminated. ", "The remaining games of the season had keepers' coach Kirill Gashichev in charge.", "\n\nPFL Years (2013–14)\n\nRussian Cup \n\nAfter signing a contract with Ukrainian manager Oleh Leshchynskyi on 4 March 2014, FC Tosno have defeated FC Spartak Moscow in the 2013–14 Russian Cup Round of 16, with the only goal scored by Valentin Filatov on the 114th minute. ", "It was the fourth time in Russian Cup's history, when a team from the Second Division had participated in the Russian Cup's Quarter-finals.", "\n\nOn 26 March 2014, 'Tosno' were eliminated from the cup in an away game against Krasnodar, which ended 3–0 to the hosts.", "\n\nOn 15 May, Leshchynskyi was fired due to a conflict with the club's players, so Vyacheslav Matyushenko, the team's chairman, had to head the team. ", "On 27 May, after a 1–0 win against FC Tekstilshchik Ivanovo in PFL, FC Tosno had secured a place in the Russian National Football League of the upcoming season.", "\n\nFNL Years (2014–17) \nPrior to the season's start, Tosno had signed a contract with Bulgarian manager Nikolay Kostov. ", "The first four games were headed by Kirill Gashichev, however, on 30 July Kostov had been officially included in the squad. ", "Tosno had topped the league for the time of Nikolay Kostov's leadership, however, on 5 November the contract with Kostov had terminated and Kirill Gashichev had to head Tosno again. ", "In 2014, under the leadership of Gashichev, the club had finished third with four wins and one loss. ", "\n\nOn 4 December 2014, Aleksandr Grigoryan was appointed the club's new manager on a two-year contract, however Grigoryan left the club on 28 February 2015 without taking charge of first team game for the club due to family circumstances. ", "\n\nThe manager post was then passed to Yevgeni Perevertailo, who guided Tosno to a third-place finish in the league. ", "However, Tosno lost 1:5 on aggregate to Rostov in the play-offs and did not earn promotion to the Russian Premier League.", "\n\nOn 12 August 2015, Dmytro Parfenov was appointed as Tosno's new manager on a two-year contract. ", "Tosno finished the season in 7th position and reached the Round of 16 in the Russian Cup, where the club lost only to future champions Zenit.", "\n\nAt the end of the 2016–17 season, Tosno have earned promotion to the 2017–18 Russian Premier League for the first time.", "\n\nRFPL Year (2017–2018) \nThe 2017–18 season was Tosno's first ever top-tier campaign in Russian football. ", "The club spent much of the league stage of the season struggling in relegation or relegation-playoff places. ", "The cup, on the other hand, saw the team enjoy vast success, reaching the final after defeating reigning champions FC Spartak Moscow in a match that saw them fight back from a one goal deficit in regular time to seal a close victory in penalty shoot-outs. ", "They went on to win the final on 9 May 2018, netting two goals and conceding once in a FC Avangard Kursk triumph.", "\n\nAs a consequence of winning the cup, Tosno were qualified to represent Russia in the following season's Europa League group stage, pending UEFA licensing, which would have marked the club's first ever appearance in a major European competition. ", "However, immediately following the cup victory, Russian Football Union president Aleksandr Alayev announced that Tosno had missed the deadlines for UEFA licensing applications and would therefore not be eligible to participate in the competition. ", "They were relegated from the Russian Premier League on the last day of the 2017–18 season after losing 0–5 to FC Ufa.", "\n\nOn 30 May 2018, Russian National Football League announced that Tosno failed on its appeal to obtain the FNL license for the 2018–19 season and will be forced to apply for the third-tier Russian Professional Football League license, sealing a two-level relegation. ", "On 9 June 2018, the founder of FORT Group (the company that owned the team), Maxim Levchenko, announced that the team is officially dissolved.", "\n\nDomestic history\n\nStadium\nIn November 2014, it was announced that FC Tosno are planning to build a new home stadium in Tosno. ", "Its capacity was expected to be over 10,000 seats and it was to be completed before the start of the 2015–16 season. ", "Those plans were not realised.", "\n\nIn the 2017–18 season, the club played their home games at the Petrovsky stadium in St. Petersburg, which previously hosted the home matches of FC Zenit. ", "The stadium has a capacity of 20,985.", "\n\nHonours\n Russian Cup (1): 2017–18\nRussian Professional Football League (1): 2013–14, West Zone\n\nTimeline of head coaches \nNumber represents chronological order. ", "\nInformation correct as of May 2018.", "\n\nNotable players\nHad international caps for their respective countries. ", "Players whose name is listed in bold represented their countries while playing for Tosno.", "\n\nRussia\n Vladimir Bystrov\n Arseny Logashov\n Pavel Pogrebnyak\n Yegor Sorokin\n Anton Zabolotny\n\nFormer USSR countries\n Dzyanis Laptsew\n Otar Martsvaladze\n Giorgi Navalovski\n Nukri Revishvili\n Artem Milevskiy\n Vagiz Galiulin\n\nEurope\n Ante Vukušić\n Mladen Kašćelan\n Nemanja Mijušković\n Marcin Kowalczyk\n Marko Poletanović\nAfrica\n Nuno Rocha\n\nReferences\n\nExternal links\n Official website\n\n \nCategory:Defunct football clubs in Russia\nCategory:Sport in Leningrad Oblast\nCategory:Association football clubs established in 2013\nCategory:2013 establishments in Russia\nCategory:Association football clubs disestablished in 2018\nCategory:2018 disestablishments in Russia" ]
{ "pile_set_name": "Wikipedia (en)" }
[ 0.012658227848101266, 0.006211180124223602, 0, 0, 0.02, 0.015384615384615385, 0.01818181818181818, 0.008849557522123894, 0, 0.007462686567164179, 0.0043859649122807015, 0, 0.006802721088435374, 0, 0.011764705882352941, 0.0125, 0.007462686567164179, 0.007194244604316547, 0.01652892561983471, 0.013422818791946308, 0.01875, 0.008403361344537815, 0.008064516129032258, 0.016483516483516484, 0.009900990099009901, 0.008403361344537815, 0.008620689655172414, 0.008264462809917356, 0.02040816326530612, 0, 0, 0, 0, 0, 0.008849557522123894, 0.004048582995951417, 0.008097165991902834, 0.008547008547008548, 0.00749063670411985, 0.014084507042253521, 0, 0, 0, 0.01282051282051282, 0, 0, 0, 0, 0, 0.0015151515151515152 ]
0.006831
5
[ "Q:\n\nBootstrap DIVs sliding in JQuery\n\nI am using Bootstrap to manage my UI. ", "I have three Divs -\nFirst div : col-lg-10.", "\nSecond div : col-lg-36.", "\nThird div : col-lg-1. ", "Third div has another hidden div (Inner_Third div) that has my data to display.", "\nFirst and Second Divs are always visible. ", "Third Div is collapsible and it expands (and show() Inner_Third div) based on a hyperlink click from Second div.", "\nI am able to expand the Third div (and show() the Inner_Third div) with ease. ", "But I am stuck when I am again collapsing the Third div (and hide() Inner_Third div) again. ", "The hide() of Inner_Third div happens before the collapsing starts and I want it to hide() after the collapse.", "\nCollapsed State :\nFirst div : col-lg-10.", "\nSecond div : col-lg-36.", "\nThird div : col-lg-1 (with Inner_Third div hidden).", "\nExpanded State :\nFirst div : col-lg-10.", "\nSecond div : col-lg-19.", "\nThird div : col-lg-19 (with Inner_Third div visible).", "\nBelow is my One Page code -\n<!", "DOCTYPE html>\n<html xmlns=\"http://www.w3.org/1999/xhtml\">\n<head>\n <title></title>\n <script src=\"Scripts/jquery-1.9.0.js\"></script>\n <script src=\"Scripts/bootstrap.js\"></script>\n <script src=\"Scripts/jquery-ui-1.10.4.js\"></script>\n <link href=\"Content/bootstrap.css\" rel=\"stylesheet\" />\n\n</head>\n<body>\n <div class=\"row\">\n <div class=\"col-lg-10\" style=\"height:300px;background-color: #FA5D4E\" id=\"content\">\n <h2>Getting started</h2>\n <p>\n ASP.NET MVC gives you a powerful, patterns-based way to build dynamic websites that\n enables a clean separation of concerns and gives you full control over markup\n for enjoyable, agile development.", "\n </p>\n <p><a class=\"btn btn-default\" href=\"http://go.microsoft.com/fwlink/?LinkId=301865\">Learn more &raquo;</a></p>\n </div>\n <div class=\"col-lg-36\" style=\"background-color:yellow;height:300px\" id=\"menu1\"> \n <h2>Get more libraries</h2>\n <p>NuGet is a free Visual Studio extension that makes it easy to add, remove, and update libraries and tools in Visual Studio projects.</p>\n <p><a class=\"btn btn-default\" href=\"http://go.microsoft.com/fwlink/?LinkId=301866\">Learn more &raquo;</a></p>\n <a href=\"#\" id=\"showDetailsSection\">Details</a>\n </div>\n <div class=\"col-lg-1\" style=\"background-color:blue;height:300px;padding:0px\" id=\"menu2\">\n <div id=\"menu2Div\" style=\"display:none;background-color:green;overflow:auto;margin:0px;height:300px;padding:0px\">\n <h2>Web Hosting</h2>\n <p>You can easily find a web hosting company that offers the right mix of features and price for your applications.</p>\n <p><a class=\"btn btn-default\" href=\"http://go.microsoft.com/fwlink/?LinkId=301867\">Learn more &raquo;</a></p> \n </div>\n </div>\n </div>\n</body>\n</html>\n\n<script type=\"text/javascript\"> \n $('#showDetailsSection').click(function () {\n\n var $col1 = $('#menu1');\n var $col2 = $('#menu2');\n\n if ($col1.hasClass('col-lg-36')) {\n $col1.switchClass('col-lg-36', 'col-lg-19');\n $col2.switchClass('col-lg-1', 'col-lg-19');\n $('#menu2Div').show(0);\n }\n else { \n $col1.switchClass('col-lg-19', 'col-lg-36');\n $col2.switchClass('col-lg-19', 'col-lg-1');\n $('#menu2Div').hide();\n }\n });\n</script>\n\nA:\n\nBootstrap has an event that fires after the collapse is complete. ", "Instead of on click for the hide, you could use hidden.bs.collapse:\n$('#showDetailsSection').on('hidden.bs.collapse', function () {\n $('#menu2Div').hide();\n});\n\n" ]
{ "pile_set_name": "StackExchange" }
[ 0.02631578947368421, 0, 0, 0, 0, 0, 0, 0, 0, 0, 0, 0, 0, 0, 0, 0, 0, 0.002751031636863824, 0.0031762837480148226, 0 ]
0.001612
5
[ "Perturbation of Trp 138 in T4 lysozyme by mutations at Gln 105 used to correlate changes in structure, stability, solvation, and spectroscopic properties.", "\nIn order to correlate between spectroscopic and structural changes in a protein, the environment of Trp 135 in T4 lysozyme was deliberately perturbed by the replacement of Gln 105 with alanine (Q105A), glycine (Q105G), and glutamic acid (Q105E). ", "In wild-type lysozyme, Trp 135 is buried, but the indole nitrogen is hydrogen-bonded to the side-chain of Gln 105. ", "In the Q105G and Q105A mutant structures, the indole nitrogen becomes accessible to solvent. ", "Crystallographic analysis shows that the structures of all of the mutants are similar to wild-type. ", "There are, however, distinct rearrangements of the local solvent structure in response to the new side-chains. ", "There are also small but significant changes in the relative orientations of the two domains of the protein that appear to result from a series of small, concerted movements of side-chains adjacent to residue 105. ", "Evaluation of the fluorescence and phosphorescence of the mutant proteins in terms of their observed three-dimensional structures shows that large spectral changes do not necessarily imply large changes in structure or in static solvent accessibility. ", "Increases in polar relaxation about the excited state of tryptophan may be the result of only small increases in local dynamics or solvent exposure. ", "1H-NMR was also used to monitor the effects of the substitutions on Trp 138. ", "In Q105E, but not in Q105G, Q105A and WT, the H epsilon chemical shift of Trp 138 is very pH-dependent, apparently reflecting the titration of Glu 105 which has a spectroscopically determined pKa of 6.0. ", "The elevation of the pKa of Glu 105 in Q105E is also reflected in the pH dependence of the stability of this mutant." ]
{ "pile_set_name": "PubMed Abstracts" }
[ 0, 0, 0, 0, 0, 0, 0, 0, 0, 0, 0.00980392156862745, 0 ]
0.000817
5
[ "New Bond Author Boyd Interviewed\n\nThe title of the new James Bond adventure written by author and screenwriter William Boyd will be announced on Monday, April 15, in central London, during a special talk given by the award-winning novelist, and he has also given a new interview to help promote the event.", "\n\nThe announcement of the title to Boyd’s eagerly anticipated new 007 book will be made at 11.30am at the London Book Fair (LBF), held at Earl’s Court in London, on April 15. ", "Boyd will make a personal appearance at the Book Fair to deliver his talk (which he has termed an ‘interview/conversation’), and will be available to speak to journalists. ", "A special James Bond drinks reception will also take place later on in the day, during Monday evening. ", "It will be Boyd’s third visit to the famous Book Fair, which is a key event for the publishing industry, and this year runs from April 15-17.", "\n\nIn a short press release, Boyd said: ‘I’m both delighted and honoured to be one of the Authors of the Day at the London Book Fair. ", "It’s a rare and rather wonderful pleasure to find oneself for a few hours at the very centre of the international publishing industry and LBF always provides the warmest of welcomes to writers’.", "\n\nA special press release officially announcing the new book title will also be made available to the media from midday on Monday, April 15.", "\n\nBoyd on Bond\n\nWilliam Boyd was also interviewed for a special LBF tie-in podcast, where he talked about his general literary career and also spoke briefly about his involvement with the world of James Bond.", "\n\nBoyd said that he said ‘yes instantly’ when he was asked whether he would write the next James Bond novel. ", "He said it came along at ‘just the right time’, as he had just written two spy novels and was steeped in the world of spying. ", "He said he also knew a lot about author Ian Fleming, and had made use of Fleming in one of his previous novels. ", "However, even though he knew much about the James Bond author, as part of the preparation for his own Bond novel he said he had re-read all the Fleming novels himself, as a form of ‘interesting re-education’.", "\n\nBoyd was also keen to stress the differences between the book Bond and the screen Bond. ", "Boyd said the literary Bond ‘remains true to itself’, whereas most people only know Bond from the films and interpret him via the screen version. ", "The movies have ‘diluted’ and changed people’s perceptions of Bond, and had taken Bond off in directions that he (Boyd) was not always comfortable with. ", "But, he said, the literary Bond ‘is a fascinating creation’, and ‘is a distinct and clear-cut creation’.", "\n\nWhen asked whether he had a personal favourite Ian Fleming novel, Boyd said he liked the ones with their ‘feet firmly on the ground’. ", "He said the stories that get ‘gimmicky’ and ‘extravagant’ were his least favourite ones. ", "His favourite one, he said, is From Russia With Love, followed closely by the ‘much maligned’ later Bond book The Man With The Golden Gun.", "\n\nHe said he liked From Russia With Love because it is like ‘a real spy novel’, with a ‘honey-trap’ plot, and (in his view) was Fleming’s ‘most interesting novel’. ", "As with From Russia With Love, Boyd said, The Man With The Golden Gun was also ‘more true’ to the real world of spying, as it was about an assassination in Jamaica, with a straightforward story. ", "These were the books he favoured because his own Bond book is also ‘a straightforward mission’.", "\n\nIndeed, this more realistic approach had influenced his own upcoming Bond novel, which, he said, will follow Fleming’s chronology. ", "In Boyd’s new book, set in 1969, Bond will be aged 45, and will be a more experienced man, ‘an older and wiser Bond’, not a young agent. ", "Asked to sum up his James Bond novel, Boyd said he told the Fleming family that his Bond book is about ‘a real spy on a real mission’.", "\n\nTowards the end of the interview, Boyd revealed that he had ‘great fun’ writing the new 007 novel, but said he had also taken it ‘very seriously’, too, with a lot of research. ", "Boyd also said that he recognised that James Bond has become ‘global’, and he said it was already ‘quite an experience to suddenly be part of that’.", "\n\nBoyd’s new James Bond book will be published in the UK by Jonathan Cape on September 26, 2013, and by Harper Collins in the USA on October 8, 2013." ]
{ "pile_set_name": "Pile-CC" }
[ 0.009836065573770493, 0.022857142857142857, 0.005813953488372093, 0.009708737864077669, 0.0070921985815602835, 0.007518796992481203, 0.005154639175257732, 0, 0.019230769230769232, 0.01834862385321101, 0, 0.008928571428571428, 0.009615384615384616, 0.011111111111111112, 0.00684931506849315, 0.013071895424836602, 0.009615384615384616, 0.014705882352941176, 0, 0, 0, 0.005128205128205128, 0, 0.007518796992481203, 0.014598540145985401, 0.022388059701492536, 0.0056179775280898875, 0.013513513513513514, 0.026845637583892617 ]
0.009485
5
[ "Introduction\n\nWelcome back to the Banlist series! ", "There have been quite a few changes to the list from when I first released the original series. ", "For this article, I will detail the updates to the list and, same as before, talk about why those cards got banned, and whether or not they should come back.", "\n\nLet us begin!", "\n\nBanned Monsters\n\nFor this section, I will talk about the monsters in chronological order. ", "Like before, I will split them into Main Deck and Extra Deck monsters.", "\n\nMain Deck\n\nKonami has banned only a few monsters from the main deck since around July of last year.", "\n\nEclipse Wyvern\n\nEclipse Wyvern is a level 4 LIGHT Dragon monster. ", "If sent to the GY, you can banish a level 7 or higher LIGHT or DARK Dragon from your deck. ", "If banished from the GY, you can add that banished monster to your hand.", "\n\nThunder Dragon decks, as well as other Dragon Link decks, utilized this card to add Chaos Dragon Levianeer to the hand. ", "This would allow the opponent to either destroy cards on the field or shuffle a card from their opponent’s hand to the deck. ", "They could also Special Summon a monster from their GY.", "\n\nGuardragon Elpy easily summoned Wyvern from the deck and helped further Dragon Deck combos.", "\n\nKonami banned it on the July 2019 list.", "\n\nCan It Come Back?", "\n\nAt the moment, I think it is fine coming back. ", "The Guardragon engine is slightly less powerful because of the banning of two important cards, which I will detail later on. ", "However, I feel that it would be able to come back to 1, and not more. ", "It would give a small boost to dragon decks, especially since they all need it.", "\n\nOrcust Harp Horror\n\nOrcust Harp Horror is a level 4 DARK Machine that, while in the GY, can banish itself to Special Summon an Orcust from the Deck. ", "If this effect is used, you cannot Special Summon monsters, except for DARK monsters, for the rest of the turn.", "\n\nFrom the release of Knightmare Orcust in SOFU, Orcusts have been a very good deck in the metagame. ", "In fact, it was considered one of the best decks in the format, especially with the Sky Striker engine. ", "One of the main reasons that the Orcust strategy was so resilient and consistent was due to the utility of Harp Horror. ", "It served as a starter for nearly any deck that wanted a consistent counter-trap negate with Orcust Crescendo. ", "So much extension was possible, making OTKs with Borrelsword Dragon extremely easy. ", "Harp Horror provided excellent interaction during your opponent’s turn with Orcustrated Babel on the field as well.", "\n\nKonami banned it on the January 2020 banlist.", "\n\nCan It Come Back?", "\n\nIf you have read any of my past articles, you would obviously know what my answer would be. ", "However, as much as I would want it back, we need to be a bit more realistic.", "\n\nBy this point, a lot of players were tired of having to play with or against this deck. ", "Because it was so popular, it had extremely high representation in the metagame. ", "Although there might have been other cards to hit in the Orcust strategy, I think that Harp to 0 was the correct choice. ", "It stopped the deck’s ability to be put into nearly any deck, but did not hinder the deck too much.", "\n\nExtra Deck\n\nThe Phantom Knights of Rusty Bardiche\n\nThe Phantom Knights of Rusty Bardiche is a LINK-3 DARK monster with 2100 ATK that requires at least 2 DARK monsters. ", "It allowed you to send a “The Phantom Knights” monster from your deck to the GY in order to set a “Phantom Knights” S/T from your deck, an excellent boost to the PK deck.", "\n\nKonami banned this monster on the July 2019 banlist.", "\n\nCan It Come Back?", "\n\nUnfortunately, it cannot. ", "DARK is one of the best attributes in the game (arguably THE best), and as such, there are so many decks that would be able to abuse this card. ", "Not only that, but on the January 2020 list, the PK RuM was brought back to 3. ", "This card is very good, and would be broken beyond belief if Bardiche were to return.", "\n\nLong story short, it provides too much advantage for too little investment.", "\n\nGuardragon Agarpain\n\nReleased in SOFU as part of the Guardragon engine, Guardragon Agarpain summoned Dragon monsters from the Extra Deck. ", "Usually, this would include cards like Hot Red Dragon Archfiend Abyss or Crystal Wing Synchro Dragon for negates. ", "Pendulum decks would summon Odd-Eyes Absolute Dragon and Link it away to get an Odd-Eyes Vortex Dragon.", "\n\nThe Guardragon Engine in general was extremely consistent. ", "Any number of 1 or 2-card starters would end on boards that included many negates, especially because Agarpain provided so much power to this strategy.", "\n\nKonami banned it on the October 2019 banlist.", "\n\nCan It Come Back?", "\n\nMost likely not. ", "Despite Agarpain being gone, the other 2 Guardragons are still very good cards. ", "Dragon decks are still very powerful, and with things like Ib, the World Chalice Justiciar being legal during this card’s time, this strategy was extremely strong and consistent. ", "It should stay banned for now.", "\n\nKnightmare Mermaid\n\nReleased in FLOD, Knightmare Mermaid is a LINK-1 that requires any “Knightmare” monster to summon. ", "If Link Summoned, it allows you to discard a card to Special Summon a Knightmare monster from your deck.", "\n\nGouki utilized this well by creating extra links. ", "Knightmare provided an extra material with Knightmare Corruptor Iblee. ", "Iblee would then summon itself to the opponent’s side of the field, stopping them from Special Summoning.", "\n\nHowever, it was the Orcust deck that caused Konami to ban this card on the October 2019 banlist. ", "With the release of Orcust Knightmare, any two monsters were able to make the full Orcust Combo. ", "Thus, literally every single deck had the capability of splashing in a free negation and an extremely resilient monster engine.", "\n\nCan it Come Back?", "\n\nNot as long as Orcust Knightmare is still in the game. ", "Unbanning Orcust Mermaid again would greatly increase its consistency and allow many decks to run the Orcust engine in their strategies again.", "\n\nThunder Dragon Colossus\n\nWith SOFU came the release of a new deck for a beloved card: Thunder Dragon. ", "This new archetype focused on utilizing different forms of Thunder Dragons to combo off and end turns with its new boss monsters: Thunder Dragon Colossus and Thunder Dragon Titan. ", "Colossus prevented the opponent from adding cards from their deck to their hand. ", "Titan allowed the player to destroy cards on their opponent’s field whenever a “Thunder Dragon” effect is activated in the hand.", "\n\nThis deck went through a few iterations, including Brilliant Fusion and Dragon Combo variants. ", "It was a popular choice to play since it was able to deal with Sky Strikers relatively well.", "\n\nKonami banned it on the January 2020 list.", "\n\nCan it Come Back?", "\n\nLike many people, I was surprised that this card was hit so severely on the list. ", "Thunder Dragons, although being a strong deck, were not unbeatable. ", "There were a few ways in which strategies were able to deal with it, and many felt that it was a fair deck.", "\n\nHowever, I will admit that the Dragon combo variant was a bit oppressive, especially with Colossus being able to be summoned to hinder the opponent. ", "Not only that, but based on the trends seen on the January 2020 list, it made sense that it was hit.", "\n\nHeavymetalfoes Electrumite\n\nReleased way back in EXFO, Heavymetalfoes Electrumite is a LINK-2 that requires two Pendulum monsters to summon. ", "It was the best card that strategy could ask for. ", "Not only was it extremely easy to summon, but it helped search out those missing scales, and helped maintain advantage with its draws.", "\n\nThe Pendulum deck has gone through a lot of versions ever since the Link Era started, going from Pendulum FTK to Negation Turbo to Dragon Link, to the Endymion strategy of today. ", "At the center of all of it, was Electrumite.", "\n\nKonami banned it in the January 2020 banlist.", "\n\nCan It Come Back?", "\n\nThe way the January 2020 list made it necessary to ban an important card of the Pendulum archetype, even if it was not as active in the metagame as the other decks that were hit. ", "Electrumite is too good for the Pendulum playstyle, and keeping it in the game might not be a good idea.", "\n\nWith new strategies coming out every day, I think that it is best for this card to stay on the list.", "\n\nIb, the World Chalice Justiciar\n\nAfter being defeated by Longirsu’s lance, Knightmare Incarnation Idlee was reborn into Ib, the World Chalice Justiciar. ", "This is a lvl 5 Synchro Monster that, when Synchro Summoned, allows you to add a “World Legacy” card from the deck to your hand. ", "If sent to the GY, it Special Summons a “World Chalice” monster from the deck or GY, except itself.", "\n\nAll Dragon Link decks utilized this card in their combos. ", "In these cases, Ib would search out a World Legacy Guardragon, which would summon a Dragon monster from the GY. ", "There were a number of one or two card combos that led to boards of many negates. ", "Mathematician or Souleating Oviraptor alone would lead to end boards that included many negates.", "\n\nKonami banned it on the January 2020 list.", "\n\nCan It Come Back?", "\n\nI do not think that this card will return. ", "It is an extremely important engine piece for an extremely powerful strategy. ", "Having one-card combos that can lead to so much board building is a bit scary to think about.", "\n\nTempest Magician\n\nTempest Magician is a bit of an old card. ", "Konami first released it in Crossroads of Chaos back in 2008. ", "It is a lvl 6 Synchro Monster that, when Synchro Summoned, allows you to place a Spell Counter on it. ", "During your turn, you can discard cards and place Spell Counters on face-up cards on your field, up to the number of cards discarded. ", "Finally, you can remove all Spell Counters on the field to inflict 500 damage to the opponent for each one.", "\n\nObviously, the only way to use this card is with an FTK. ", "As there was no consistent way to naturally generate a large amount of Spell Counters through other cards, there were some gimmicky decks that utilized Tempest Magician’s effect. ", "These strategies included cards like Tuningware, Formula Synchron, and T.G. Hyper Librarian to draw a whole bunch of cards. ", "You would then be able to discard 16 cards to burn the opponent for 8000 LP.", "\n\nHowever, with the introduction of the Endymion Structure Deck, generating Spell counters became even easier than before. ", "Tempest Magician quickly became a relatively consistent FTK for Pendulum.", "\n\nKonami banned in on the January 2020 list.", "\n\nCan it Come Back?", "\n\nI do not see it coming back. ", "Cards like this tend to promote degenerate strategies and should be kept on the list. ", "Not only that, but Endymion Magicians are a thing.", "\n\nOuter Entity Azathot\n\nOuter Entity Azathot is a RANK 5 Xyz monster that, if Special Summoned, prevents the opponent from using Monster effects. ", "When Rusty Bardiche was legal, decks would use RANK 4 Time Thief Redoer for the PK RuM, summoning it on their opponent’s turn. ", "After both the RuM and Bardiche were banned, combo decks would summon first Outer Entity Nyarla, then overlay the Azathot to stymie hand trap usage. ", "Lunalights were utilizing this card quite a lot, especially once Nibiru, the Primal Being was released.", "\n\nQuite a lot of players disliked this card. ", "Yugioh has reached a point where many decks need to run hand traps to counteract other strategies. ", "For a deck to have easy access to a card that effectively makes the opponent unable to do anything to fight back is a bit disheartening.", "\n\nKonami banned it on the January 2020 list.", "\n\nCan it Come Back?", "\n\nFor the reasons mentioned above, I do not think so.", "\n\nSalamangreat Miragestallio\n\nFirst released with the Salamangreat Structure deck, Salamangreat Miragestallio is a RANK 3 Xyz monster. ", "You can detach a material so Special Summon a “Salamangreat” monster from your deck. ", "If used as material for a Link Summon of a Salamangreat monster, it can return a monster on the opponent’s field to their hand.", "\n\nWith the limiting of both Salamangreat Gazelle and Salamangreat Circle, Miragestallio became even more important for the deck. ", "With any two lvl 3 monsters, you were still able to consistently find your Gazelle and start the engine. ", "Not only that, but it provides a quick way to get rid of problems on the field without destruction.", "\n\nKonami banned it on the January 2020 list.", "\n\nCan it Come Back?", "\n\nI do not think that this card can come back. ", "It provides the Salamangreat deck with too much consistency. ", "With other decks at a lower power level due to the list, it also makes sense that the card stays banned.", "\n\nBanned Spells\n\nOnly two spells were banned within the last six months or so, but both are important in the metagame.", "\n\nBrilliant Fusion\n\nBrilliant Fusion is a Continuous Spell that allows you to Fusion Summon a “Gem-Knight” monster by using materials in your deck.", "\n\nEver since its release, any deck that utilized Normal Summons ran the Brilliant Fusion engine. ", "Gem-Knight Seraphinite was the target for Brilliant Fusion because it provided an additional Normal Summon. ", "With the advent of the Link Era, Seraphinite provided an excellent way to spam materials for link summoning.", "\n\nFor Brilliant Fusion to work as intended, you need to have a Gem-Knight in the deck. ", "Because of this, if you drew the target, the Brilliant Fusion was dead. ", "Most decks ran Gem-Knight Garnet because it had the highest attack of all other Gem-Knight monsters. ", "Its use was so widely spread that if a card needed to remain in the Main Deck for an engine to work, it was called a “Garnet”.", "\n\nUnfortunately, the main reason that Konami banned it on the January 2020 list was that it facilitated the Gem-Knight FTK.", "\n\nCan it Come Back?", "\n\nUnfortunately, this card cannot come back. ", "Giving decks an easy way to Normal Summon an additional time is an excellent effect. ", "The fact that this card also happens to make an FTK more consistent is not a good thing, either.", "\n\nSky Strike Mobilize – Engage!", "\n\nAnd with this, we have finally reached the last card that was forbidden on a list since the last time I released my articles. ", "This is arguably the one that required a ban the most.", "\n\nSky Striker Mobilize – Engage! ", "is a Spell card that adds a “Sky Striker” card from the deck to the hand. ", "If there are three or more Spells in the GY, you can also draw a card. ", "This alone should be enough to explain why the card needed to go.", "\n\nSpells are the most diverse cards in Yugioh. ", "Decks that ran Sky Striker engines utilized a lot of one-time-use spells in order to have at least three Spells in the GY. ", "This included things like Upstart Goblin, Foolish Burial Goods, and Metalfoes Fusion. ", "Prior to the January 2020 list, Orcust Sky Striker was considered as the best deck of the format.", "\n\nNo matter what deck it was used in, this card provided too much advantage off of so little.", "\n\nKonami banned it on the January 2020 list.", "\n\nCan it Come Back?", "\n\nOf all cards in this article, Engage deserves a return to the game the least. ", "As explained above, it provided too much advantage for so little. ", "The feeling of topdecking an Engage with three spells in the GY while you’re behind is sublime. ", "Not only that but Sky Striker Ace – Kagari was brought back to three on this same list.", "\n\nBanned Traps\n\nInterestingly enough, there weren’t any traps banned. ", "However, there is one that I am sure many players agree should go: Imperial Order. ", "Just because it got an errata that makes it so you have to pay LP twice as fast does not make it any less degenerate.", "\n\nConclusion\n\nWith this, we are now caught up on the current state of the Banlist. ", "However, we are not done with the ban list quite yet. ", "As the meta keeps shaping, more lists will be released, and Konami will likely ban more cards. ", "Not only that, but a few cards on here have been released. ", "I will likely make another article talking about those cards that are finally free.", "\n\nUntil then, we shall keep waiting!", "\n\nThis should have been gone by now, but it is still in the game for some reason." ]
{ "pile_set_name": "OpenWebText2" }
[ 0, 0, 0, 0, 0, 0, 0, 0, 0.01098901098901099, 0.013888888888888888, 0.01639344262295082, 0, 0.01818181818181818, 0.03225806451612903, 0.024390243902439025, 0, 0, 0.008, 0, 0, 0.006622516556291391, 0, 0.019801980198019802, 0.009615384615384616, 0, 0.009009009009009009, 0.011904761904761904, 0.017391304347826087, 0.02127659574468085, 0, 0, 0, 0, 0, 0, 0, 0.01764705882352941, 0.0058823529411764705, 0.018518518518518517, 0, 0, 0, 0.012658227848101266, 0.011764705882352941, 0, 0.02857142857142857, 0.008771929824561403, 0.009708737864077669, 0, 0, 0.02127659574468085, 0, 0, 0, 0.0111731843575419, 0, 0.01652892561983471, 0.009615384615384616, 0, 0.028169014084507043, 0.009523809523809525, 0.010101010101010102, 0.020618556701030927, 0, 0, 0.017543859649122806, 0, 0.019230769230769232, 0.011111111111111112, 0, 0, 0.010309278350515464, 0.010869565217391304, 0.022727272727272728, 0, 0, 0.014705882352941176, 0, 0.006622516556291391, 0, 0.006993006993006993, 0, 0, 0.016574585635359115, 0.022727272727272728, 0.02127659574468085, 0, 0.0055248618784530384, 0.019230769230769232, 0, 0.025806451612903226, 0.007751937984496124, 0.020202020202020204, 0, 0.026785714285714284, 0, 0.010416666666666666, 0.022727272727272728, 0, 0, 0, 0, 0, 0.016129032258064516, 0.0196078431372549, 0.007462686567164179, 0.009345794392523364, 0, 0.0111731843575419, 0.016129032258064516, 0, 0.016260162601626018, 0.0136986301369863, 0.022727272727272728, 0, 0, 0, 0.02, 0.00684931506849315, 0.023622047244094488, 0.026845637583892617, 0.009708737864077669, 0, 0.010101010101010102, 0, 0.022727272727272728, 0, 0, 0.007407407407407408, 0, 0.007874015748031496, 0.023255813953488372, 0.009523809523809525, 0, 0.022727272727272728, 0, 0, 0, 0, 0, 0.006802721088435374, 0.010309278350515464, 0.027777777777777776, 0.009259259259259259, 0.011494252873563218, 0.013888888888888888, 0.019801980198019802, 0, 0.008130081300813009, 0, 0, 0.011764705882352941, 0, 0, 0, 0, 0.030303030303030304, 0, 0.028169014084507043, 0, 0, 0.024390243902439025, 0.023255813953488372, 0.010309278350515464, 0, 0.022727272727272728, 0, 0, 0, 0.010416666666666666, 0.011494252873563218, 0, 0.012048192771084338, 0, 0.012048192771084338, 0, 0.010526315789473684, 0, 0, 0, 0 ]
0.007831
5
[ "Q:\n\nFlow can't assign property: assignment of computed property/element\n\nI'm trying to add property in the object like this:\nrequest(options:HttpRequestData): Promise<any> {\n options.headers = options.headers || {};\n\n options.headers['Accept'] = 'application/json';\n options.headers['Content-Type'] = 'application/json'; // Error\n\n return this._request(options);\n}\n\nBut it has error:\noptions.headers['Content-Type'] = 'application/json';\n ^^^^^^^^^^^^^^^^^^^^^^^^^^^^^^^ \nassignment of computed property/element. ", "Computed property/element cannot be assigned on possibly undefined value\n\nHttpRequestData is a object type:\ntype HttpRequestData = {\n url:string,\n method:string,\n headers?:Object,\n body?:string,\n};\n\nAssiging just one value into options.header(in this case, 'Accept') works but it fails when I try two or more.", "\nWhat the difference between single assigning and multiple assigning in Flow, and how to assign those properties into options.headers object?", "\nAny advice will very appreciate it.", "\n\nA:\n\nI solved this problem like this:\nrequest(options:HttpRequestData): Promise<any> {\n options.headers = options.headers || {};\n\n const modifiedHeaders = options.headers;\n modifiedHeaders['Accept'] = 'application/json';\n modifiedHeaders['Content-Type'] = 'application/json';\n\n options.headers = modifiedHeaders;\n\n return this._request(options);\n}\n\nNow no more error message.", "\n\n" ]
{ "pile_set_name": "StackExchange" }
[ 0.00375234521575985, 0, 0, 0, 0, 0 ]
0.000625
5
[ "Your favorite post of the week, month, year or even your latest post will all do. ", "Whatever you are blogging about, we want to kn...\n\n16 Nutritious Breakfast Options with Extras\n\nEating breakfast is so important, however if you are like most families, mornings may be quite hectic. ", "Do you ever end up skipping breakfast because you are in a rush? ", "It happens at least once a week here. ", "Getting up at 6:00am for the day and our day typically starts at 7:30am. ", "I make sure we get up earlier so we can have everything done to start the day right. ", "You can always prepare your diced veggies at night time so your morning goes that much quicker. ", "There is also great protein shake options for those of you that like that too. ", "Stay tuned for a whole list of tasty smoothies and shakes in a future post. ", "For now, we are just going to go over a bunch of nutritious meals that we have had on our meal plan before. ", "What are some of you favorite meals for breakfast?", "\n\nThe portions for these meals looked a little like this:\n\n1/2 cup ww oats, splash milk, 1 packet sw n low, 1/2 cup blueberries, 2 sliced strawberries you can serve this with boiled eggs with 1 or none yolks and just the whites or you can even pan fry an 2 egg whites with 1 egg and so on.", "\n\nAs you can see I love some spinach..well lots of spinach, it really is my favorite thing to eat. ", "It is so nutritious and besides it makes Popeye happy to see you eat your greens. ", "It can get expensive with a wide range of ingredients, so we tend to stick to just a few key ingredients when doing the meal planning so we do not break the bank just on ingredients for different recipes. ", "We did quite a few replacements and often times I was making breakfast for myself and my kids would eat something slightly altered from mine. ", "Those are just a handful of great options you can have for breakfast and surely start your day off right. ", "Please enjoy a couple recipe cards from me to you! ", "A print friendly button is at the end of the post or you can save the image in your digital recipe files and print it later. ", "What are some of your favorite quick and nutritious recipes? ", "What does your breakfasts normally look like?", "\n\nI am not sure on the calorie count, but I wouldn't imagine the calorie count is very high with all of the healthy options." ]
{ "pile_set_name": "Pile-CC" }
[ 0, 0, 0, 0, 0, 0, 0, 0, 0, 0, 0, 0, 0, 0, 0, 0, 0, 0, 0, 0, 0, 0 ]
0
5
[ "skyline.vista package\n=====================\n\nSubmodules\n----------\n\nskyline.vista.agent module\n--------------------------\n\n.. automodule:: vista.agent\n :members:\n :undoc-members:\n :show-inheritance:\n\nskyline.vista.fetcher module\n----------------------------\n\n.. automodule:: vista.fetcher\n :members:\n :undoc-members:\n :show-inheritance:\n\nskyline.vista.worker module\n---------------------------\n\n.. automodule:: vista.worker\n :members:\n :undoc-members:\n :show-inheritance:\n\n\nModule contents\n---------------\n\n.. automodule:: vista\n :members:\n :undoc-members:\n :show-inheritance:\n" ]
{ "pile_set_name": "Github" }
[ 0.0016313213703099511 ]
0.001631
5
[ "Electronic circuits, particularly digital logic circuits used in computers, have been extensively miniaturized, and continue to be miniaturized. ", "In digital computer logic circuits, multi-layer wiring systems are frequently employed. ", "The multi-layer wiring systems include integrated circuits containing active components which perform required logic functions. ", "The integrated circuits are mounted on the surface of a ceramic plate, generically termed a substrate, carrying conductors that establish different interconnections between access terminals of different integrated circuits. ", "The substrate, in addition to carrying integrated circuits, frequently carries other components, particular capacitors, having dimensions larger or smaller than those of the integrated circuits. ", "The conductors are generally referred to as printed circuit conductors and are deposited on the substrate by a serigraphy, i.e., silk screen, process. ", "Occasionally, a substrate must undergo wiring modifications, either during or after production.", "\nThe substrate structure is so devised that any wiring modification may be made by acting only on external connection layers of the substrate, as the external layer is the only layer which is readily available. ", "Thus, it is not possible to modify the internal substrate layer of a multi-layer wiring system. ", "To enable the wiring to be modified, all components mounted on the substrate have access terminals connected only to the external layer. ", "Thus, it is possible to modify the wiring of the multi-layer wiring by isolating particular parts of a previously existing interconnection circuit of the multi-layer wiring. ", "To sever a conductor from the external interconnection layer of the substrate in such a manner that one of the access terminals of an integrated circuit is completely isolated without deteriorating the remainder of the wiring system and/or active components implanted on the substrate, the apparatus disclosed in the copending, commonly assigned application of Maurice Devoille, entitled \"Method of And Apparatus For Severing Printed Circuit Conductors\" was devised. ", "The present invention is directed to an apparatus for and method of placing complementary circuit elements on the surface of a substrate, to modify the wiring system carried by the substrate.", "\nWith existing machines, it is possible to solder a stripped end of an insulated wire to a surface of a conventional printed circuit board. ", "With the prior art machines, the wire is applied to the board in a path between two terminals which are to be electrically connected and are firmly attached to the board. ", "The electro-mechanical machines of the prior art have been found unsuitable and too imprecise to be adaptable, without fundamental alteration, to produce an analogous wiring system on a multi-layer substrate of a few square centimeters. ", "One of the problems with the prior art devices is that they are incapable of handling wire of the order of a few hundredths of a millimeter in diameter, as employed in complementary multi-layer wiring systems. ", "The machine in accordance with the present invention has been developed to handle such wiring chores.", "\nIt is, accordingly, an object of the present invention to provide a new and improved apparatus for modifying and/or completing printed circuit wiring applied to printed circuit boards which carry substrates for integrated circuits and other micro-miniaturized components." ]
{ "pile_set_name": "USPTO Backgrounds" }
[ 0, 0, 0, 0, 0, 0, 0, 0, 0, 0, 0, 0.0021413276231263384, 0, 0, 0, 0, 0, 0, 0 ]
0.000113
5
[ "Q:\n\nIs it possible to override document.cookie in WebKit?", "\n\nIn Firefox it's possible using the following...\nHTMLDocument.prototype.__defineGetter__(\"cookie\",function (){return \"foo=bar\";});\nHTMLDocument.prototype.__defineSetter__(\"cookie\",function (){});\n\nThis doesn't cause any errors in WebKit, and WebKit definitely supports __defineGetter__ and __defineSetter__, but it doesn't work. ", "Guessing WebKit is protecting that property somehow.", "\nSo, any ideas of how to achieve the same effect in WebKit?", "\n\nA:\n\nHave you tried do define the getter/setter pair on the document object itself, instead on the prototype?", "\ndocument.__defineGetter__(\"cookie\", function() {} );\ndocument.__defineSetter__(\"cookie\", function() {} );\n\nI know it shouldn't matter, but I don't underestimate browser quirks. ", "Even with WebKit.", "\nUpdate\nI've done some tests in Chrome 2 and it appears that it only allows defining a setter. ", "Anyway, I'm not sure how valid this observation is to WebKit, as Google Chrome uses a different JavaScript interpreter than WebKit.", "\n\n" ]
{ "pile_set_name": "StackExchange" }
[ 0, 0, 0, 0, 0, 0.0056179775280898875, 0, 0, 0, 0 ]
0.000562
5
[ "SH2 and SH3 domains as molecular adhesives: the interactions of Crk and Abl.", "\nThe Src homology domains SH2 and SH3 are modular components present in many signal transduction proteins. ", "They allow rapid formation of stable protein complexes and may also regulate protein function through intramolecular binding events. ", "SH2 domains recognize phosphotyrosyl residues in a specific sequence context, while SH3 domains recognize a PxxP motif and additional residues that mediate binding specificity." ]
{ "pile_set_name": "PubMed Abstracts" }
[ 0, 0, 0, 0 ]
0
5
[ "Vietnam’s government is cracking down on Internet freedom (or whatever little there was of it in the first place) in the country — after it introduced a fine of 100 million dong ($4,740) for anyone who criticizes the government on social media.", "\n\nThe new law was announced this week, Reuters reports. ", "Some types of comments are classified as a criminal offense, which can see the person responsible slapped with a jail sentence. ", "Even if people make comments that don’t constitute criminal offenses, they may be fined if these comments are determined to be “propaganda against the state”, or spreading “reactionary ideology,” according to the law, which was reportedly signed off by Prime Minister Nguyen Tan Dung.", "\n\nThis follows up from a decree that the government rolled out in September this year, which stipulates that blogs and social media profiles belonging to individuals and businesses should contain personal information only — effectively banning people from broadcasting opinions around current affairs on the likes of Facebook and Twitter.", "\n\nOther than the fine being introduced for such social media-related comments, Vietnam’s government is also laying out new regulations for operating an e-commerce site in the country, Tech in Asia reports.", "\n\nIf a company doesn’t have a license to carry out e-commerce and fails to report service changes on its site, it could be fined anything from $200 to $1000. ", "If the site operator posts incorrect and/or falsified information, a $1,000 to $1,400 fine looms.", "\n\nVietnam is already not known for perpetuating much freedom of expression, but these moves to get a hold on Internet activities are a cause for concern — as they could lead to a much more difficult online environment, and could turn off foreign businesses wanting to expand into the country.", "\n\nHeadline image via Shutterstock\n\nRead next: Google launches elections portal in India to give citizens one-stop access to political news" ]
{ "pile_set_name": "OpenWebText2" }
[ 0, 0.017857142857142856, 0, 0.0035211267605633804, 0, 0.004878048780487805, 0, 0, 0, 0.007246376811594203 ]
0.00335
5
[ "The European Union set out a road map for boosting capital markets across its 28 members Wednesday, in an effort to spur investment in the region and revive the bloc’s flagging economy.", "\n\nJonathan Hill, the EU's financial services chief, outlined a set of initiatives that flesh out the so-called Capital Markets Union, his much-anticipated project aimed at helping companies raise cash in Europe by boosting non-bank financing and tackling obstacles to investment." ]
{ "pile_set_name": "Pile-CC" }
[ 0.005405405405405406, 0.010752688172043012 ]
0.008079
5
[ "Lake Malombe\n\nLake Malombe is a lake in southern part of Malawi, on the Shire River, in the Southern Region. ", "It is located at around , about south of much larger Lake Malawi. ", "It has an area of about . ", "In recent years the number of fishermen on the lake rose substantially, and this led to local decline in some fish species, especially the chambo cichlids which is an important source of food throughout Malawi. ", " The lake is extremely shallow with an average depth of approximately eight feet, and during periods of dry weather the water level recedes and can even disappear.", "\n\nThe lake-bed was dry for several hundred years until it refilled in the middle of the 19th century.", "\n\nReferences \n Community-based fisheries management, Lake Malombe\n  \n\nMalombe\nCategory:Shire River\nCategory:Geography of Southern Region, Malawi" ]
{ "pile_set_name": "Wikipedia (en)" }
[ 0.009174311926605505, 0.014925373134328358, 0, 0.004739336492890996, 0, 0, 0.013888888888888888 ]
0.006104
5
[ "2 Cents\n\nVladTV Staff Writer\n\n1/2\n\nNelly has A LOT to say in response to the rape allegations that fell upon him Saturday.", "\n\nIn a fiery tweet storm, Nelly said “Let me say that I am beyond shocked that I have been targeted with this false allegation. ", "I am completely innocent. ", "I am confident that once the facts are looked at, it will be very clear that I am the victim of a false allegation…I also want to thank my fans for their unwavering support. ", "They know me. ", "I assure you I will be vindicated. ", "And I assure you, I will pursue every legal option to address this defaming claim. ", "Thank you”\n\nEarlier today, Nelly was arrested for second-degree rape when a woman said that Nelly raped her on his tour bus in Ridgefield, Washington. ", "He is on tour with Florida Georgia Line. ", "Auburn police spokesman Commander Steve Stocker said that the St. Louis rapper will have his first appearance before a judge “at some point.” ", "Nelly will not address the allegations on stage tonight as he will not perform.", "\n\nNelly also vows to take legal action against this “defaming claim.” ", "To clear the record, Nelly tweeted, “To be absolutely clear. ", "I have not been charged with a crime therefore no bail was required. ", "I was released , pending further investigation.”" ]
{ "pile_set_name": "Pile-CC" }
[ 0.00819672131147541, 0.0078125, 0, 0, 0, 0, 0, 0.013245033112582781, 0, 0.007042253521126761, 0.012658227848101266, 0, 0.01639344262295082, 0, 0 ]
0.004357
5
[ "Main menu\n\nPost navigation\n\nTrust delegation in Openstack using Keystone trusts\n\nIn one of our blog posts we presented a basic tool which extends the Openstack Nova client and supports executing API calls at some point in the future. ", "Much has evolved since then: the tool is not just a wrapper around Openstack clients anymore and instead we rebuilt it in the context of the Openstack Mistral project which provides very nice workflow as service capabilities – this will be elaborated a bit more in a future blog post. ", "During this process we came across a very interesting feature in Keystone which we were not aware of – Trusts. ", "Trusts is a mechanism in Keystone which enables delegation of roles and even impersonation of users from a trustor to a trustee; it has many uses but is particularly useful in an Openstack administration context. ", "In this blog post we will cover basic command line instructions to create and use trusts.", "\n\nSetting up\n\nFirst of all – trusts are only available in Keystone v3, v3 endpoints must be exposed for Keystone.", "\n\nCheck your openstack-client and keystone version. ", "We had issues where Keystone Trusts were not working due to an outdated client version. ", "Here we use openstack-client version 3.0.1 and keystone 10.0.0. ", "Versions can be checked as follows:\n\n:# openstack --version\n:# keystone-manage --version\n\nThe openstack client can be updated using pip as follows:\n\n:# sudo pip install --upgrade python-openstackclient\n\nExample of users and projects\n\nFor all the commands shown in this blog post we will use the following users and projects so it is easier for you to understand what each command does. ", "Each user has admin and member roles in their respective projects – the admin user only has access to the admin project and the alt_demo user only has access to alt_demo project.", "\n\nCreating trusts\n\nCreating a trust relationship between users is fairly straightforward. ", "The basic premise is that one user can authorize another user to act on her behalf by creating an Openstack trust object. ", "More concretely, the trustor creates a trust object which enables a trustee to act on her behalf: the trustee uses her own credentials and the trust_id and is then able to act on behalf of the trustor.", "\n\nThe basic syntax of the command is shown below; This command returns a trust_id which can be used to authenticate operations in any Openstack service.", "\n\nHere, we assume that credentials are configured as environment variables as is standard practice, so we don’t have to provide them as parameters in the command itself.", "\n\nIn this case a trustor, alt_demo, gives a specific role, member, in a project, alt_demo, for a trustee, admin. ", "The trustee, admin, then will be able to perform any Openstack operations in that project based on the role provided.", "\n\nNote that the trustor can only grant trusts of roles and projects for which it has authorization; e.g. if the trustor is only a member in a certain project he cannot provide delegated admin role for another user in that same project using trusts.", "\n\nThe impersonate flag changes the attribute user in the token created using the trust_id, the user will be the trustor itself in case the flag is present otherwise the user in the token is the trustee.", "\n\nActing on the trustor’s behalf\n\nIn order to use the trust_id provided by the command above first we need to remove any environment variables related to projects and domains. ", "This information is already included in the token created by the trust_id (an exception is thrown if there is a conflict between the environment variables and the trust_id).", "\n\nTo check for environment variables and remove them use the commands below:\n\nThis command, executed with the trustee credentials, creates a VM in the project specified by the trust_id, which is associated with the trustor: the trustor can log into the Openstack web interface and will see this VM as if she had created it herself.", "\n\nIn case you don’t use environment variables there are very few changes to make in the API call:\n\nAdding these parameters after the command openstack will be the same as using environment variables allowing the admin user to create a VM in the alt_demo project.", "\n\nConclusion\n\nIn this blog post we gave an introduction to Keystone Trust and an example of how it can be used. ", "Note that the trust mechanisms do not solve the general problem of Openstack admins being able to act on the user’s behalf (eg creating VMs for the user based on snapshots etc) as the users still need to authorize them to do so, although there known workarounds. ", "However, it is very useful capability which is used in more complex configurations such as those where Openstack services need to act on behalf of a user.", "\n\n1 thought on “Trust delegation in Openstack using Keystone trusts”\n\nGlad you found them useful . ", "Trusts were originally developed for Heat, to be able to perform operations on a users behalf long after the user is no longer available to authorize them. ", "Glad to find they are usable to a wider array of uses" ]
{ "pile_set_name": "Pile-CC" }
[ 0.008547008547008548, 0.0035087719298245615, 0, 0.004694835680751174, 0, 0.008849557522123894, 0, 0, 0, 0, 0, 0, 0, 0, 0.006578947368421052, 0, 0.008849557522123894, 0.008547008547008548, 0, 0, 0, 0, 0.0030211480362537764, 0.003816793893129771, 0.008928571428571428, 0.0038022813688212928, 0, 0.010101010101010102, 0, 0 ]
0.002642
5
[ "Greek football players of AEL Larissa F.C (L), who have been banned from wearing refugee crisis awareness messages on their shirts, and Acharnaikos F.C held a sit-in protest at the start of a match in favor of migrants and refugees January 29, 2016 (AFP Photo/Nichalis MPATZIOLAS)\n\nAthens (AFP) - Second-division football players in Greece held a sit-in protest at the start of a game over the spiralling death toll among migrants trying to reach the European country by sea.", "\n\nPlayers from rivals AEL Larissa and Acharnaikos sat on the field and observed two minutes of silence after kickoff Friday in a game in the central city of Larissa.", "\n\nAEL said in a statement that the protest was \"in memory of the hundreds of children that perish every day because of the heinous apathy of the EU and Turkey about what is happening in the Aegean Sea.\"", "\n\nDespite wintry conditions, thousands of people fleeing war and poverty in the Middle East, Asia and Africa continue to risk their lives crossing the Mediterranean to Europe in rickety boats.", "\n\nAt least 33 migrants drowned on Saturday when their boat sank in the Aegean Sea en route from Turkey to Greece, Turkey's Anatolia news agency reported.", "\n\nThe UN says more than 46,000 migrants have arrived in Greece so far this year, with 200 people dying during the voyage.", "\n\nLast year, Greece received over 850,000 arrivals.", "\n\nThe European Commission on Wednesday hit out at Athens, saying it had \"seriously neglected\" its duty to protect the bloc's frontiers." ]
{ "pile_set_name": "OpenWebText2" }
[ 0.00631578947368421, 0.006060606060606061, 0.009900990099009901, 0, 0.006535947712418301, 0.008264462809917356, 0, 0.007407407407407408 ]
0.005561
5
[ "Robert Dunkin\n\nRobert Dunkin (1761–1831), of Penzance, Cornwall, was a Quaker businessman and a mentor of the young Humphry Davy, a founder of the science of electrochemistry, in the practice of experimental science.", "\n\nAccording to a death notice in The West Briton 19 August 1831:\n\"At Penzance on Thursday the 11th instant, Mr Dunking, [died] aged 70 years. – ", "This most respectable man was a member of the Society of Friends; he was originally a saddler, which business he long carried on; but he was also an excellent mathematical instrument maker, and was well known to men of science by some valuable improvements in the barometer and the thermometer. ", "He was an able mathematician, and in natural philosophy, especially in electricity and magnetism, he was deeply skilled – His amiable disposition, and the unassuming manners so well suited to his religious opinions, secured him the respect of all who knew him and will long endear his memory to his numerous friends.\"", "\n\nPersonal life\nRobert Dunkin was the son of Robert and Anna Dunkin, born at Penzance and baptised on 14 July 1761. ", "On 20 January 1786 Robert Dunkin and Celia Bonello, a widow, daughter of William and Elizabeth Odgers of Helston, were married, after the manner of Quakers. ", "Their son, Robert, was born on 1 July 1787.<ref>Much of this article is based on June Z. Fullmer's book Young Humphry Davy. ", "For a full reference see Sources.</ref> Celia Dunkin was an active Quaker. ", "She died, aged 76, on 21 January 1824.", "\n\nRobert Dunkin died, aged 70, at Penzance, 10 August 1831.", "\n\nDunkin's business\nA trade card is on display at Penlee House, Penzance:.", "\n\nDUNKIN & JAMES/\nSADDLE AND HARNESS/\nManufacturers/\nPENZANCE/\nPHILOSOPHICAL INSTRUMENTS/\nMade and Repaired/THERMOMETERS, BAROMETERS, THEODOLITES, ANGLOMETERS,\nSHIPS COMPASSES &C/\nENGRAVED COPPERPLATE PRINTING/\nMEDICAL ELECTRICITY &C\n\nPigot's Directory of Cornwall—1823 shows Robert Dunkin operating from Market Place, Penzance. ", "Robert Dunkin is listed under \"Tradesmen\" as a saddlemaker in the Universal British Directory—1791.", "\n\nDunkin as a Quaker\nAs his marriage took place after the manner of Friends, both partners must have been Quakers at this date (1786). ", "He was listed as a member of Penzance Quaker Meeting, in a list written in 1828, his acquisition of membership being by \"convincement\". ", "He published a religious poem \"On God\" in the Imperial Magazine in 1820.", "\n\nDunkin and Davy\nHumphry Davy's first biographer, John Ayrton Paris, was an unreliable witness, according to June Z. Fullmer. ", "She contests the idea that Davy was a \"country bumpkin\", giving evidence that he and his family were of middling status in Penzance society: \"Intimated always, in Paris's descriptive flights, were notes of snobbish disdain\". ", "Paris caricatures Dunkin as \"Will Snaffle\" in his roman a clef – Philosophy in sport made science in earnest (1827).", "\n\nJune Z. Fullmer says:\"Instruction of Davy did not arise solely from his schooling and his apprenticeship. ", "As important as anyone for his training was Robert Dunkin .... Dunkin has been variously referred to as saddler, ironmonger, patentee and mathematical instrument maker. ", "These changing sobriquets reflect his versatility and technological expertise.\" ... \"", "Perhaps the best testimony to Dunkin's ability derives from his activities. ", "He received two patents, the first, in 1802, with Dr Henry Penneck as co-patentee, for the invention of 'Methods of improving Sailing and Navigation of certain ships and vessels'; the second, in 1813, for 'Methods of lessening the consumption of steam and fuel in working fire engines, and also methods for the improvement of certain instruments useful for mining and other purposes ' \".", "\n\nRobert Hunt, in his article in Dictionary of National Biography (1888) on Davy, says: \"....These conditions developed a love of poetry and the composition of verses and ballads. ", "At the same time he acquired a taste for experimental science. ", "This was mainly due to a member of the Society of Friends named Robert Dunkin, a saddler; a man of original mind and of the most varied acquirements. ", "Dunkin constructed for himself an electrical machine, voltaic piles, and Leyden jars, and made models illustrative of the principles of mechanics. ", "By the aid of these appliances he instructed Davy in the rudiments of science. ", "As professor at the Royal Institution, Davy repeated many of the ingenious experiments which he had learned from his quaker instructor.\"", "\n\nHarold Hartley in his Humphry Davy, discounted Ayrton Paris's tale (Life p. 31) about Davy's source of scientific instruments. ", "Paris claimed that they were a gift from a French doctor, who had been shipwrecked off Land's End. \" . . . ", "any doubt about the veracity of Davy's statements about his apparatus has now been removed, as Dunkin's help must have been of the greatest assistance to him in those early days\".", "\n\nDavy's study of chemistry started in 1897 and his contact with Dunkin ceased when he moved to Bristol on 20 December 1798. ", "Davy's initial contacts with Robert Dunkin may have been much earlier; they lived in the centre of the same town; Dunkin's son, also named Robert, was slightly older than Davy.", "\n\nFullmer cites the first documented contact was Davy's written attack on \"The pretended inspiration of Quakers and other sectaries\", the continuation of an oral debate. ", "Dunkin is reported as having responded \"I tell thee what, Humphry, thou art the most quibbling hand at a dispute as I ever met with in my life\". ", "The success of their chemical and electrical experiments must have been to some extent dependent in the skills of the Quaker instrument maker.", "\n\nReferences\n\nSources\nJune Zimmerman Fullmer: Young Humphry Davy: the making of an experimental chemist, Philadelphia: American Philosophical Society (2000), (APS Memoirs Volume 237) – \nGeorge Clement Boase and William Prideaux Courtney Bibliotheca Cornubiensis: a catalogue of the writings both written and printed of Cornishmen and of works relating to the County of Cornwall with biographical memoranda and copious literary references.'' (", "1874) \n\nCategory:1761 births\nCategory:1831 deaths\nCategory:18th-century English people\nCategory:19th-century English people\nCategory:Converts to Quakerism\nCategory:Cornish engineers\nCategory:Cornish inventors\nCategory:Cornish scientists\nCategory:English businesspeople\nCategory:English Quakers\nCategory:People from Penzance\nCategory:Scientific instrument makers" ]
{ "pile_set_name": "Wikipedia (en)" }
[ 0.018518518518518517, 0.006896551724137931, 0.003389830508474576, 0, 0.02586206896551724, 0.03821656050955414, 0.008064516129032258, 0.013333333333333334, 0, 0.03389830508474576, 0.02702702702702703, 0.015197568389057751, 0.010101010101010102, 0.02962962962962963, 0.007352941176470588, 0.013888888888888888, 0.023622047244094488, 0.0044444444444444444, 0.008620689655172414, 0, 0.011834319526627219, 0, 0.013157894736842105, 0.002583979328165375, 0.011111111111111112, 0, 0.013333333333333334, 0.013605442176870748, 0, 0.007352941176470588, 0.031007751937984496, 0, 0.0111731843575419, 0.016, 0.017045454545454544, 0.01764705882352941, 0.006896551724137931, 0.007042253521126761, 0.011286681715575621, 0 ]
0.011979
5
[ "Welcome to our Monthly Dev Post of June/July 2019. ", "Every month (ahem) one of these post will be created, to tell you about the latest developments in the world of OpenTTD.", "\n\nJune and July have been quiet months for changes in OpenTTD master, but there are a couple of nice development things to talk about, and as always, translations have been added and improved. ", "Thanks translators!", "\n\nIn this issue: Multi-docks, some tids and bits and the start of an exciting new series: Source Code Entrails!", "\n\nDevelopment\n\nMaster highlights\n\nThere is one big new feature in the master branch: Peter1138 has finished multiple-docks-per-station.", "\n\nThis is now merged to master and will show up in nightly builds (soon, if not already). ", "Players are no longer restricted to just one dock per station. ", "This is a great feature, which continues the recent theme of improving ship transport. ", "Find out more about it at the pull request page.", "\n\nThree new currencies have also been added: New Taiwan Dollar (NTD), Chinese Renminbi (CNY) and Hong Kong Dollar (HKD).", "\n\nAs usual a couple of bugs have been fixed, notably some improvements to music playback on Windows and a crash using the sprite aligner (#7609).", "\n\nFinally some other bugs and performance issues have had some attention, including\n\npossible framerate issues on Mac systems\n\nother possible performance issue\n\nan interesting bug where OpenTTD crashes when opening the industry directory window if a newgrf industry has 5 or more output cargos\n\nMaintenance mayhem\n\nThe current stable branch also received some love with the release of the 1.9.2 update.", "\n\nBesides the usual bugfixes, it contains one user visible change: A newly-installed OpenTTD will now default to showing internet server instead of LAN servers in the multiplayer lobby window.", "\n\nSource Code Entrails I: The Beginning\n\nThe Idea\n\nMany people have ideas and suggestions how OpenTTD could be made better. ", "In fact, there’s a whole sub-forum full of them. ", "Every so often, “I want to make this thing but I don’t know how” comes up.", "\n\nAnd indeed, looking at the source code of OpenTTD for the first time can be a daunting task. ", "The src folder alone contains 1314 files, and is accompanied by a plethora of other ancillary files. ", "To make it less daunting to get started with realising your idea, we will explore various parts of the OpenTTD source code and try to build some kind of index to the code. ", "We are going to assume that you have seen source code before and have some familiarity with the C++ language. ", "There are thousands of introductory tutorials on programming in general and specific to C++ on the internet, thus we don’t think it would make sense to replicate them all here. ", "Also, If you’ve never compiled OpenTTD before, we’ve got general tips and some specific to Windows, along with detailed notes on our wiki.", "\n\nThis is planned to be a multi-part series, to be continued as time permits.", "\n\nInspiration for this comes from an interesting treaty by MaiZure: Decoded: OpenTTD. ", "It’s a nice read if you are interested in the technical innards of the game and already somewhat familiar with the source code. ", "Unfortunately, if you are not, the parts MaiZure focuses on aren’t the parts you’ll predominantly need to be able to write your first working code patch. ", "Even if you read it again and again, you will know all about how memory pools manage memory in OpenTTD, but still have no idea where to look to make all trains run backwards. ", "Thus the idea of writing a similar exploration of OpenTTD, but more focused on practical things for the patch writer, was born.", "\n\nDiving In\n\nWith that out of the way, let’s start this series by trying to get some structure into those 1314 source files. ", "We are focusing on the src folder for now and will ignore all the other files and folders until some later part of this series.", "\n\nInside this folder you’ll find a lot of files and some more folders. ", "Some of the names speak for themselves, but don’t let yourself be fooled by some of the others. ", "Contrary to what the name might imply, the game folder is not the most important folder at all, but only contains code for running GameScripts; all the important game logic is contained elsewhere. ", "The core folder is a similar beast, supplying some low-level scaffolding, but not some mystical nugget controlling all objects in the game. ", "This should in no way imply that this is superfluous code; the scaffolding is essential part of almost all parts of the game. ", "But unless you are planning to rewrite whole swaths of the game, it is quite unlikely you’ll find any need to change these foundations.", "\n\nIndeed, most of the interesting files are the ones directly in the src folder. ", "If you just browse through the list of files, you might notice groups of similarly named files that refer to an entity or an concept. ", "There are, for example, groups of files for entities like aircraft , train or station , and concepts like economy . ", "The OpenTTD source code is structured along those entities; code related to the same thing is kept in files with a common base name. ", "If you want to modify the behaviour of trains, looking into the files whose names start with train is certainly not a bad idea. ", "Of course, such a strict division is impossible to keep in all cases. ", "The code that decides what should happen if an aircraft stops at a station conceptually belongs to both aircraft and station , even if it can exist in only one place in reality.", "\n\nYou might have also noticed that the file names follow a pattern and that you can find similarly named files in each group. ", "Many files are named after the following schema: <group>_<suffix>.<extension> . ", "The <group> part was already explained above. ", "The <suffix> tells you something about the contents of the file without having to open it inside an editor and can be viewed as a kind of file type. ", "The <extension> part is for the benefit of the C++ compiler and denotes if the file is a source code file that is passed to the compiler ( .cpp ) or a header file ( .h ) that is included in other source code files. ", "There’s also the hybrid of a header file containing source code (marked .hpp ) for C++ template implementations, as they have to be included everywhere they are used.", "\n\nLet’s use the station group as an example, as it contains almost all of the common file types you can find in the source code.", "\n\nstation.cpp The most obviously named file, but the one with the least defined contents. ", "The file contains the source code for all the station functionality that does not match any of the files described below. ", "It is a catch-all for everything without a better place. ", "station_cmd.cpp One of the most interesting file types in the OpenTTD source code. ", "Command handlers are the foundations of the user interaction with the game. ", "In this case, they for example encode the logic that determines where and if a station can be built, what happens when a vehicle stops at a station or how a station should be drawn on the screen. ", "station_gui.cpp This source file forms the second part of the user interaction by providing the windows visible on screen related to stations. ", "These are e.g. the station window itself, the station building window or the station list window. ", "saveload/station_sl.cpp A final source code file hides in a sub-folder. ", "The folder name already gives a hint, and this file indeed contains everything that is needed to read and write information about stations to and from savegame files. ", "station_base.h The first header file. ", "The data for a station is kept in a C++ class, like for many of the other game objects. ", "The base header contains the declaration of this class, which encapsulates and manages most of the information about a station, like for example the owner, the location or the name. ", "station_type.h While many other parts of the game need to know about stations, not all parts need to know about the innards of the station class. ", "To speed up compilation, basic type definitions about stations, like for example the type for station IDs or an enumeration of possible station types, are split into this separate header file. ", "station_func.h This file exists for the same reason as the previous header and contains function prototypes related to stations. ", "station_map.h The second main header file related to stations. ", "Stations are objects that occupy space on the game map and as such need to store some information in the map array. ", "This header file contains all the code related to storing and retrieving this information. ", "station_gui.h Type and class declarations for the user interface code are contained in this file. ", "widgets/station_widget.h This header is part of the user interface code. ", "It is split from the gui header to facilitate processing by automated build scripts. ", "table/station_land.h The final header file isn’t a header in the classical sense. ", "It is included by the code that draws the stations on the screen and was split purely to ease code navigation.", "\n\nNot all file groups have all the different files just described, but if a file type is present, it’s usage is generally like described. ", "Files are usually only split if they would contain a sufficient amount of code, there’s no advantage of splitting one or two code lines into a separate file.", "\n\nAnd with that said, we are going to conclude this very first installment of the Source Code Entrails series. ", "We hope this has helped you at least a bit with finding your way through the OpenTTD source code files, even if we’ve neither talked about all the files that don’t fit into the name scheme nor have taken a look inside one of these files yet.", "\n\nStay tuned for further installments where we are going the explore the various areas of the game in more detail.", "\n\nClosing Remarks\n\nWe hope this is something at least some of you find interesting. ", "The exact format of this series is still fluid and we haven’t decided yet how technical and specific we are going to get. ", "We’d love to hear your comments or questions, both regarding the contents of this installment and the idea for this series itself.", "\n\nThere is no roadmap for the topics of further installments yet, but if you tell us what topics you’d be most interested in, we’ll certainly take it into account when deciding about future topics. ", "If you are interested in contributing content for a topic, drop us a line.", "\n\nParticipate yourself\n\nDo you have an interesting Project you are currently working on in relation to OpenTTD? ", "These Monthly Dev Posts are prepared in a branch on our GitHub website project before they are made public on the website. ", "As soon as you are whitelisted as a contributor, it’s as simple as editing the file in the web interface. ", "If you are not a contributor yet, drop by on IRC to become one (make sure you have a GitHub account)." ]
{ "pile_set_name": "OpenWebText2" }
[ 0.0196078431372549, 0, 0, 0, 0, 0.007407407407407408, 0, 0, 0, 0, 0.025, 0, 0.0024875621890547263, 0.005208333333333333, 0, 0, 0, 0, 0, 0, 0, 0, 0.007246376811594203, 0, 0.011627906976744186, 0, 0.006493506493506494, 0, 0, 0, 0, 0, 0, 0.005076142131979695, 0, 0, 0, 0, 0, 0, 0, 0, 0, 0, 0, 0, 0, 0, 0, 0, 0, 0, 0, 0, 0, 0, 0, 0, 0, 0, 0, 0, 0, 0, 0, 0, 0, 0, 0, 0, 0, 0, 0, 0, 0, 0, 0.006369426751592357, 0, 0, 0, 0, 0, 0, 0, 0, 0, 0.016260162601626018, 0, 0.009900990099009901 ]
0.001378
5
[ "Introduction\n============\n\nIn December 2019, a severe public health emergency was induced by the outbreak of a novel coronavirus, which has since been named coronavirus disease (COVID-19) by the World Health Organization (WHO) \\[[@ref1]\\]. ", "Since the first-level response by government officials to COVID-19 across China's provinces and cities, governmentally imposed social isolation has provided the Chinese population with ample time to search online for the latest COVID-19--related news \\[[@ref2]\\]. ", "Baidu, as the most widely used Chinese search engine, accounts for two-thirds of China's search engine market share \\[[@ref3]\\]. ", "At the time of the COVID-19 outbreak, we found that the number of searches for COVID-19 had increased exponentially, despite the fact that its incidence and mortality rates were much lower than those of some noncommunicable diseases, such as cancer. ", "Hence, this phenomenon is worthy of further attention and discussion.", "\n\nMethods\n=======\n\nWe obtained web search data from the Baidu Index \\[[@ref4]\\]. ", "As of May 2020, Baidu accounts for 71.23% of the search engine market share in China \\[[@ref5]\\] and is the most widely used search engine in the country. ", "It is a well-known and extensive platform for information/resource sharing that Chinese internet users rely on.", "\n\nSearch data, dating to as early as 2004, were derived from search frequencies on Baidu. ", "Frequencies were calculated based on the search volumes of specific search terms entered by internet users, and data on a daily, monthly, and yearly basis were obtained for the search terms we chose from the Baidu Index.", "\n\nWe entered the formal Chinese names for \"lung cancer,\" \"liver cancer,\" \"esophageal cancer,\" \"colon and rectum cancer,\" and \"breast cancer,\" respectively, and acquired their search data history from January 1, 2020, to March 17, 2020. ", "The Baidu Index provides search term analysis; it also uses a process to scientifically determine related search terms based on the mode through which the searchers initiate a search request. ", "This means that other relevant search terms will be provided automatically once the internet users enter a search term. ", "Thus, we entered \"COVID-19\" into the Baidu Index and obtained its related search terms from February 11, 2020, to March 17, 2020 (search volumes for \"COVID-19\" can only be obtained from February 11, 2020 onward). ", "The search terms mainly included health literacy, social panic, and prevention and control measures relating to COVID-19. ", "All search data were downloaded free of charge on March 17, 2020.", "\n\nWe performed descriptive analyses to describe and compare the overall search situation under the context of the pandemic from different aspects. ", "Statistical analysis was conducted using Excel 2016 (Microsoft Corporation) and IBM SPSS (version 22.0, IBM Corporation). ", "We used Tableau (version 2018.3, Tableau Software) and Excel 2016 (Microsoft Corporation) to create heat maps.", "\n\nResults\n=======\n\nMapping of Health Literacy and Social Panic Via Web Search Data During a Public Health Emergency\n------------------------------------------------------------------------------------------------\n\nThe COVID-19 pandemic necessitates requirements for public health literacy. ", "Despite China's active response in combating this outbreak, a proportion of the Chinese population has remained afraid, which may seriously affect progress in controlling and preventing future outbreaks. ", "This phenomenon is particularly striking on the internet and within social media, and investigating COVID-19--related health literacy and social panic via these platforms may yield important insights.", "\n\nTaking the COVID-19 outbreak as an example, spurious information disseminated via social media by nonexpert individuals has included false claims on how to kill the COVID-19 virus (severe acute respiratory syndrome coronavirus 2 \\[SARS-CoV-2\\]), for example, by drinking alcohol, smoking, aromatherapy, essential balms, use of a hair dryer, or taking a hot bath \\[[@ref2]\\]. ", "The internet has figuratively become a double-edged sword in the context of the COVID-19 outbreak.", "\n\nConcerns of Internet Users During the COVID-19 Outbreak\n-------------------------------------------------------\n\nBy summarizing web searches for COVID-19, the most searched content has focused on the progression of the pandemic and tips on how to protect oneself from infection \\[[@ref6]\\]. ", "Web searches on COVID-19 have mainly included the following: the latest news on the pandemic; relevant knowledge about COVID-19 infection prevention and control; basic, as well as more comprehensive, information describing SARS-CoV-2; and rumors and suggested policies from unqualified sources \\[[@ref6]\\].", "\n\nCitizens in Wuhan, Hubei Province, have been more concerned about COVID-19 compared to citizens from other cities during the same period since the outbreak first occurred in Wuhan ([Figure 1](#figure1){ref-type=\"fig\"}). ", "Wuhan citizens searched for content related to \"medical help\" 22% of the time, followed by searches for \"protective materials\" and \"pandemic progress.\" ", "Medical help refers to special assistance and support provided to the citizens of Wuhan during the pandemic, which included the deployment of 42,000 trained medical workers from across the country to Wuhan and other cities in Hubei Province, and donations of medical supplies (medical masks, medical protective clothing, ventilators, etc).", "\n\n![", "The cumulative number of confirmed cases of coronavirus disease (COVID-19) (top row) and the change in COVID-19\\'s search index (bottom row) in China.](jmir_v22i7e18831_fig1){#figure1}\n\nSince the COVID-19 outbreak, web searches for wild animals have reached a historical peak---the Baidu topic \"refuse to eat wild animals\" has reached nearly 100 million views \\[[@ref7]\\]. ", "Content related to \"harm to wild animals\" has received considerable attention. ", "At the early stage of the pandemic, there were some speculation that wild animals may have spread the coronavirus to people in Wuhan, although there is no scientific conclusion regarding the origin of COVID-19 \\[[@ref8]-[@ref10]\\]. ", "In this context, on February 24, 2020, on the basis of the *Law of the People\\'s Republic of China on the Protection of Wildlife*, China established a comprehensive system to prohibit the consumption of wild animals \\[[@ref11]\\].", "\n\nWith the pandemic now gradually under control in China, public concerns have changed accordingly \\[[@ref12]\\]. ", "Web searches for \"when to return to work and start school\" have increased eight-fold from those during the previous month based on the web search data in the Baidu Index. ", "Searches for content related to \"closed community and remote office\" have continued to rise, and searches for \"remote office demand\" have risen by 663% from the previous quarter. ", "Individuals from first-tier cities such as Beijing, Shanghai, Shenzhen, Guangzhou, and Chengdu have paid more attention to telecommuting. ", "Employees who have returned to work have mainly engaged in web searches on \"return to work and prevention measures,\" \"return to work guarantee policy,\" and \"time to return to work.\" ", "Men have been half as attentive as women regarding protective measures related to returning to work.", "\n\nPreviously, the four major industries most discussed by Chinese netizens have been online education, online medical care, online entertainment, and fresh electronic commerce (e-commerce) ([Table 1](#table1){ref-type=\"table\"}) \\[[@ref6]\\]. ", "During the pandemic, provinces with large populations of educated individuals (eg, Henan, Hebei, and Shandong) have been focusing on \"online education.\" ", "In contrast, fourth-tier cities have been paying more attention to \"online medical care\" (in China, cities are usually graded according to various levels such as city development, comprehensive economic strength, talent attractiveness, information exchange capability, international competitiveness, technological innovation capability, and transportation accessibility. ", "First-tier cities refer to metropolises that are important for social, political, and economic activities \\[eg, Shanghai, Beijing\\]. ", "Most fourth-tier cities are medium-sized cities.). ", "Additionally, young and middle-aged groups exhibited greater search volumes for \"online medical care\" \\[[@ref10]\\].", "\n\n###### \n\nGrowth rate of search volumes of the four major industries discussed by netizens \\[[@ref6]\\].", "\n\n Industry Search growth rate (%)\n --------------------------- ------------------------\n Online education 248\n Online medical care 200\n Online entertainment 170\n Fresh electronic commerce 120\n\nImpact of a Lack of Health Literacy on the COVID-19 Outbreak\n------------------------------------------------------------\n\nCOVID-19 is a novel and highly infectious disease \\[[@ref13],[@ref14]\\], but in the early days of the outbreak, people had limited knowledge of it and did not know how to prevent it, which concomitantly caused panic. ", "Despite the COVID-19 pandemic becoming gradually controlled in China, most people still remain under high alert. ", "Some misinterpretations of discussions on the resumption of work on the internet have caused another wave of panic among some groups and cities since returning to work. ", "This phenomenon can be observed on Weibo, a social media platform based on information sharing, dissemination, and acquisition in real time (similar to Facebook). ", "Many people are worried about whether a large number of people returning to work will reinitiate the spread of COVID-19. ", "However, individuals in areas with no outbreaks of the disease or low incidence of infections have also expressed this concern. ", "However, if cities continue to stagnate, such stagnation may lead to a higher mortality rate than that attributed to COVID-19. ", "Zhang Wenhong, head of a Shanghai medical treatment expert group, stated the following in an interview \\[[@ref15]\\]:\n\n> If the hospital does not return to work, cancer patients cannot receive chemotherapy and surgery, and other infected people cannot be treated. ", "Patients with trauma cannot get a good treatment. ", "Under such circumstances, the number of patients dying from other diseases will far exceed the number of people dying from the new coronavirus.", "\n\nWhy Are Public Health Emergencies of Greater Concern Than Highly Fatal Chronic Diseases Like Cancer?", "\n----------------------------------------------------------------------------------------------------\n\nCancer-related deaths in China account for approximately 27% of all global cancer-related deaths \\[[@ref16],[@ref17]\\]. ", "As presented in [Table 2](#table2){ref-type=\"table\"}, except for esophageal cancer, the search frequencies of all other cancers have decreased since the COVID-19 outbreak. ", "Meanwhile, the search frequency of COVID-19 has increased significantly \\[[@ref4]\\]. ", "Dr Tedros Adhanom Ghebreyesus, Director-General of the WHO, reported that the global mortality rate of COVID-19 is approximately about 3.4% \\[[@ref18]\\]. ", "According to the latest data from the National Health Commission of the People's Republic of China on February 3, 2020, the mortality rate of COVID-19 in Hubei Province is 3.1%, while the national COVID-19 mortality rate is even lower at 0.2% \\[[@ref19]\\].", "\n\n###### \n\nChanges in the search index of the top five cancers and coronavirus disease (COVID-19) in China from January 27, 2020, to March 17, 2020.", "\n\n Disease Incidence rate (per 100,000 persons)^a^ Mortality rate (per 100,000 persons)^a^ Daily mean value Overall search index^b^ Mobile search index^b^ \n ------------------------- ----------------------------------------- ----------------------------------------- ------------------ ------------------------- ------------------------ ------ ------ --------\n Lung cancer 58 49 3849 3538 --23 --1 --19 ---^c^\n Liver cancer 37 30 1301 1128 --63 --22 --65 --18\n Esophageal cancer 17 15 1108 957 11 3 14 3\n Colon and rectum cancer 31 13 215 120 --12 --9 --21 --16\n Breast cancer 26 6 2235 2034 --50 --15 --50 --13\n COVID-19 --- --- 25,256 19,614 --- 4263 --- 3784\n\n^a^Incidence and mortality rates for the five cancers were obtained from the Global Burden of Diseases Database \\[[@ref20]\\].", "\n\n^b^Negative values represent decline.", "\n\n^c^Not available.", "\n\nThe death rate associated with cancer is much higher than that of COVID-19. ", "If we use the national data (excluding Hubei Province), the mortality rate of COVID-19 is comparable to that of the general influenza, and the total incidence rate is far lower than that of influenza \\[[@ref21],[@ref22]\\]. ", "There are several reasons that may explain the relatively high mortality rates of COVID-19 in Wuhan and in Hubei Province in general, including the stronger virulence of SARS-CoV-2 in Wuhan, more cross-infection, and the prevalence of patients with mild symptoms who did not see a doctor \\[[@ref23]\\].", "\n\nDiscussion\n==========\n\nPrincipal Findings\n------------------\n\nHealth literacy includes two aspects: (1) knowledge, which comprises basic health knowledge and skills; and (2) ability, which refers to one's ability to acquire, understand, screen, and apply health information \\[[@ref24],[@ref25]\\]. ", "Health information literacy represents the core of health literacy---it can greatly improve the public\\'s capacity for self-protection in order to improve the overall response to public health emergencies.", "\n\nThe health literacy rate of Chinese residents in 2018 was 17.06% \\[[@ref26]\\]. ", "It mainly covers basic health knowledge and concept literacy, healthy lifestyle and behavior literacy, and basic health skills literacy. ", "Although the health literacy rate of residents has improved, the uneven distribution of health literacy levels between urban and rural areas, and across regions and populations, still exists. ", "The health literacy levels of rural residents, residents in the Midwest, and the elderly are relatively low. ", "As mentioned above, with the rapid development of internet technology, people can easily use the internet to search for health information. ", "However, the new coronavirus that caused the recent pandemic was previously unknown. ", "Since the COVID-19 outbreak, the related transmission characteristics, symptoms, transmission channels, and methods for protection have been gradually communicated to the public via recent publications on COVID-19--related research. ", "Public health emergencies have the characteristics of urgency and paroxysm since they require the public to respond quickly. ", "At such times, the ability to acquire, understand, and use health data will enable individuals to more quickly facilitate disease control and prevention in the face of a public health emergency.", "\n\nPeople use the internet for almost everything they do nowadays. ", "By uploading and downloading information, everyone can be a publisher and conveyor of the immense quantity of information available on the internet. ", "Search engines facilitate the acquisition and learning of health information from a variety of sources, which provides the public with more diversified content, autonomy, and greater control over choices. ", "However, search engines also lead to many problems. ", "For example, they may disseminate data from unreliable sources, making it difficult for the public to distinguish between high-quality and low-quality health information \\[[@ref27]-[@ref29]\\]. ", "In these unprecented times, people are more vulnerable and credulous to the impact of new information related to COVID-19. ", "Hence, the vast amount of information available on the internet undoubtedly has a considerable impact on public health literacy and may influence the control and prevention of further outbreaks.", "\n\nImproving health information literacy is the primary component of improving overall health literacy. ", "First, it is necessary to raise awareness of the important role of health information literacy, and to realize how it can improve public health and promote the development of health services. ", "Second, the state should take the lead in setting up a network containing national health information and support services from multiple sources (eg, the public, medical organizations, governmental sectors) to ensure that the public can receive assistance in obtaining health information and related skills. ", "Because health information involves professional knowledge and also concerns the health of every citizen, the release of authoritative information from professional agencies during the pandemic is particularly important. ", "During the COVID-19 pandemic, many users of social media often cited the words of academician Zhong Nanshan, a trusted public academic, to spread information from a credible source. ", "This pattern demonstrates that individuals need authoritative health information. ", "Timely and authoritative information may be one of the most effective ways to eliminate doubt and reduce panic, especially in the face of public health emergencies,. ", "Third, education should be strengthened to improve the level of public health literacy throughout schools, communities, and villages. ", "Individual education levels and education systems are important factors affecting public health information literacy, and health information literacy education through comprehensive linkage represents the most inclusive and cost-effective precautionary measure \\[[@ref30],[@ref31]\\]. ", "As recommended by the WHO, the following response strategies are required: rapidly establishing international coordination and operational support; scaling up of national readiness and responses to operations; and accelerating priority research and innovation \\[[@ref32]\\].", "\n\nIn addition to face-to-face communication, the online role of health care providers in public health communications is also important for mitigating medical misinformation. ", "Examples of such online public health communications are WebMD in the United States, AskDr in Singapore, HaoDF in China, etc. ", "Through these outlets, health care authorities can communicate with the public via the internet and provide professional and reliable health information.", "\n\nWhile classic public health measures still play very important roles in tackling the current COVID-19 pandemic, there are also many new potentially enabling technological domains that can be applied to help monitor, survey, detect, and prevent such pandemics, including the Internet of Things, big data analytics, artificial intelligence, and blockchain technology. ", "Therefore, we should make full use of these emerging technologies to contain the current COVID-19 outbreak, as well as future pandemics in a timely and effective manner \\[[@ref33]-[@ref39]\\].", "\n\nConclusions\n-----------\n\nSince the Chinese New Year, the COVID-19 outbreak has become the most important issue in China. ", "Fear of an unknown virus represents the beginnings of panic, and the internet and social media networks have since become incubators and catalysts of panic. ", "Individual cases can be shared by tens of millions of people in a single day through social media dissemination. ", "Hundreds of millions of people are eagerly absorbing information about the novel coronavirus, but reading about new cases is undoubtedly causing concerns among citizens about their own health status.", "\n\nThe pandemic has diverted attention from other health issues. ", "China accounts for 19% of the global population, and its incidence of cancer accounts for 22% of the total global prevalence of cancer. ", "Nevertheless, these diseases have never caused panic at the level of COVID-19. ", "The mortality rate of COVID-19 cannot be compared with that of any other major noncommunicable disease. ", "Despite this discrepancy, the Chinese population continues to be horrified by COVID-19 but at ease with other known diseases. ", "We need to collectively alter our minds about COVID-19 in order to manage such public health emergencies more rationally. ", "This change requires a more logical use of information obtained from social media and the internet during public health emergencies, as well as improvements in health literacy and the ability to cope with social panic.", "\n\nThis research was funded by the National Natural Science Foundation of China (\\#71910107004). ", "We also want to acknowledge Baidu Corporation for its open-data support.", "\n\nAuthors\\' Contributions: All authors critically revised the manuscript, reviewed, and contributed to the final version and approved it. ", "YW is responsible for the study design.", "\n\nConflicts of Interest: None declared.", "\n\nCOVID-19\n\n: coronavirus disease\n\ne-commerce\n\n: electronic commerce\n\nSARS-CoV-2\n\n: severe acute respiratory syndrome coronavirus 2\n\nWHO\n\n: World Health Organization\n" ]
{ "pile_set_name": "PubMed Central" }
[ 0.008333333333333333, 0.003787878787878788, 0.015503875968992248, 0.004, 0, 0.024691358024691357, 0.012903225806451613, 0, 0.011111111111111112, 0.004545454545454545, 0, 0.005208333333333333, 0, 0.004694835680751174, 0, 0, 0, 0.03278688524590164, 0.02727272727272727, 0.0034482758620689655, 0, 0, 0.005305039787798408, 0.01020408163265306, 0.010238907849829351, 0.0032679738562091504, 0, 0, 0, 0, 0.010723860589812333, 0, 0.008620689655172414, 0.013100436681222707, 0.008849557522123894, 0.005847953216374269, 0, 0.007246376811594203, 0, 0, 0.008298755186721992, 0, 0, 0, 0, 0.008695652173913044, 0.009615384615384616, 0.005110732538330494, 0, 0, 0.006134969325153374, 0.008264462809917356, 0, 0, 0.0076045627376425855, 0, 0, 0, 0.008968609865470852, 0.005813953488372093, 0.023529411764705882, 0.012987012987012988, 0.015625, 0, 0.0011527377521613833, 0, 0, 0.01282051282051282, 0.008968609865470852, 0.0033222591362126247, 0.006688963210702341, 0.004878048780487805, 0.012345679012345678, 0, 0, 0, 0, 0, 0.004291845493562232, 0, 0, 0, 0, 0, 0, 0.010362694300518135, 0, 0, 0, 0, 0, 0, 0.005494505494505495, 0, 0, 0, 0.007042253521126761, 0.007326007326007326, 0, 0, 0, 0, 0.010471204188481676, 0.008130081300813009, 0, 0, 0, 0, 0, 0.012658227848101266, 0, 0, 0, 0, 0.010416666666666666, 0.013888888888888888, 0, 0.02564102564102564, 0, 0.005747126436781609 ]
0.00445
5
[ "Q:\n\nMatch at exact 3 character in a word length of 4 with all possible combinations\n\nIn the following word list\nABCD\nAAAA\nAAAD\nAAAB\nBBDA\nCCCC\nCCCA\nDADA\nBABC\n\n...\nall possible 256 combinations\n\nUsing regrex, I want to select words which has my pattern, A or B in any combination covering exact 3 position out of 4.", "\nExpected output:\nAAAD\nBBDA\nBABC\n\nI know using [AB]{4} I can match whole world but conditional search with exact 3 position out of 4 creating confusion.", "\n\nA:\n\nJust rephrasing and confirming the rules of matching you said so my solution is adhering to it,\n\nthree positions out of four should be occupied by either A and B\nOnly one position goes reserved for C or D\n\nIf this is correct, you can use this regex to match the strings you want.", "\n^(?=[AB]*[CD][AB]*$).{4}$\n\nExplanation of above regex:\n\n^ - Start of string\n(?", "=[AB]*[CD][AB]*$) - Positive look ahead to ensure either C or D appears only once in the string so other three positions are occupied by As and Bs\n.{4}$ - Capture four letters A to D using dot as they are already validated to be A to D by positive look ahead.", "\n\nRegex Demo\nHere is a regulex graph for better visualization\n\nEdit:\nDetailed explanation of (?", "=[AB]*[CD][AB]*$)\nA positive look ahead is written as (?", "=some regex) and contrary to normal regex matching and consumption, a lookaround (positive/negative lookahead/lookbehind) just matches the characters and does not consume them, meaning as soon as the lookaround expression is over, the regex marker is reset back to where it was before the lookround began matching. ", "In this regex we have [AB]*[CD][AB]*$ as expression inside it, where [AB]* means it will match any character in the set (A or B) zero or more times followed by [CD] which means it needs to match exactly one character (as there is no quantifier here) from char set which is either C or D and further again [AB]* matches any character A or B zero or more times and finally ensures end of string is reached as it has $.", "\nIn summary, logical meaning of this expression is, that there will be exactly one occurrence of either C or D while it can be surrounded by As or B on either side as needed to form four alphabets matching all combinations of four letters having only one occurrence of C or D.\nAlso, incorporated suggestion by revo where [A-D] can be changed to just . ", "Many thanks to Revo.", "\n\nA:\n\nTry the following regex:\n^([^AB\\r\\n]*[AB]){3}(?!(?1)).*$\n\nSee live demo here\nIf recursions ((?", "1) cluster) aren't supported in the engine you are working with go with this instead:\n^(?:[^AB\\r\\n]*[AB]){3}(?![^AB\\r\\n]*[AB]).*$\n\nSee live demo here\n\n" ]
{ "pile_set_name": "StackExchange" }
[ 0.003194888178913738, 0, 0, 0, 0, 0, 0, 0, 0, 0, 0.05, 0, 0 ]
0.004092
5
[ "Q:\n\nPython Algorithm To Maximize Number of Equal Elements in List\n\nI am trying to make an algorithm in Python that will take a list of random numbers from 0 to a 1,000,000 no more than 100 elements in length and will even this array out as much as possible giving me the maximum number of equal elements. ", "This is what I have so far:\ndef answer(x):\n diff = max(x) - min(x)\n while diff > 1:\n x[x.index(max(x))] = x[x.index(max(x))] - (diff / 2)\n x[x.index(min(x))] = x[x.index(min(x))] + (diff / 2)\n diff = max(x) - min(x)\n return count(x)\n\ndef count(x):\n from collections import Counter\n c = Counter(x)\n return max(c.values())\n\nThis will take an array such as [0,50] and create an array [25,25] and return the integer 2 because there are two equal elements in the array. ", "I know for a fact this algorithm works in most cases however it doesn't in all.", "\nCan anyone please point out any array of integers this would not yield the correct answer for? ", "Thanks\nEdit:\nFor those who don't want to read the while loop the code finding the range of the entire list. ", "Splitting the range in half and adding half to the min, and subtracting half from the max. ", "It is trying to equalize the entire list while keeping the same sum\n[1,4,1] = [2,3,1] = [2,2,2] = (number of equal elements) 3\n[2,1,4,9] = [2,5,4,5] = [3,4,4,5] = [4,4,4,4] = (number of equal elements) all4\n\nA:\n\nWhat about this?", "\nl = [1, 2, 5, 10]\n\n# \"best\" possible case\nl_m = [sum(l) / len(l)] * len(l)\n\n# see if lists fit (division can cause rounding errors)\n\nif sum(l_m) !", "= sum(l):\n # if they don't this means we can only have len(l) - 1 similar items\n print len(l) - 1\nelse:\n # if sums fit the first list can be spread like this\n print len(l)\n\n" ]
{ "pile_set_name": "StackExchange" }
[ 0, 0.003968253968253968, 0, 0, 0, 0.01098901098901099, 0, 0, 0 ]
0.001662
5
[ "Q:\n\nUse a for loop variable to call a predefined variable\n\nI'm looking to use a variable which gets defined inside a for loop and use it to call a predefined variable. ", "\nFor example: To start the program \"Slack\" I need to call the variable %Slack%.", "\nThe batch file will take your input, so If I input \"Slack\", the variable !", "filename[%%i]! ", "will be assigned \"Slack\". ", "Once that is done, I wish to call %Slack% using !", "filename[%%i]!. ", "As per logic: %!", "filename[%%i]!%\nUsing %!", "filename[%%i]!% will make the batch file crash and close. ", "\nSetting set file=!filename[%%i]! ", "and running it with %file% will return nothing.", "\nAny smart trick?", "\n@echo off\n\nsetlocal EnableDelayedExpansion\n\n:: Predefined variables\nfor /f \"delims== tokens=1,2\" %%G in (D:\\programs.txt) do set %%G=%%H\n\nset /P \"filenames=Enter programs to install: \"\n\n:: Puts input into array\nset n=0\nfor %%a in (%filenames%) do (\n set /A n+=1\n set \"filename[!n!]=%%~a\"\n)\n\n:: Run variables imported from programs.txt\nfor /L %%i in (1,1,%n%) do (\n\n :: This is where I want !", "filename[%%i]! ", "to be called as %!", "filename[%%i]!%\n\n)\n\npause\nendlocal\n\nprogram.txt example\nSlack=C:\\Users\\username\\AppData\\Local\\slack\\slack.exe\nText=C:\\textfile.txt\nProgram=C:\\program.bat & C:\\program_license.txt\n\nA:\n\nI don't like to use the CALL hack described in the comments because 1), it is relatively slow, and 2) it causes an extra round of parsing that can require additional escaping (not an issue here). ", "Even worse, it will corrupt quoted ^ - \"^\" becomes \"^^\", and there is no escape sequence that can prevent that.", "\nMy solution is to add an additional FOR loop so that the expansion can be staged.", "\nfor /L %%i in (1,1,%n%) do for %%A in (\"!", "filename[%%i]!\") ", "do echo !%%", "~A!", "\n\nThis is certainly more verbose than the CALL hack, but it is a good code pattern to learn because it is more universally applicable.", "\n\n" ]
{ "pile_set_name": "StackExchange" }
[ 0, 0, 0, 0, 0, 0, 0, 0, 0, 0, 0, 0, 0, 0.0024937655860349127, 0, 0, 0.002631578947368421, 0, 0, 0, 0, 0, 0.3333333333333333, 0, 0 ]
0.013538
5
[ "services:\n nextcloud:\n labels:\n com.dockstarter.appinfo.deprecated: \"false\"\n com.dockstarter.appinfo.description: \"Gives you access to all your files wherever you are\"\n com.dockstarter.appinfo.nicename: \"Nextcloud\"\n com.dockstarter.appvars.nextcloud_datadir: \"/mnt/nextcloud\"\n com.dockstarter.appvars.nextcloud_enabled: \"false\"\n com.dockstarter.appvars.nextcloud_network_mode: \"\"\n com.dockstarter.appvars.nextcloud_port_443: \"444\"\n" ]
{ "pile_set_name": "Github" }
[ 0 ]
0
5
[ "25-Hydroxyvitamin D3 and 1,25 Dihydroxyvitamin D3 as an Antiviral and Immunomodulator Against Herpes Simplex Virus-1 Infection in HeLa Cells.", "\nThe antiviral and immunomodulatory role of vitamin D has been shown in various viral infections. ", "However, there is scanty literature available about the effect of vitamin D supplementation in herpes simplex virus-1 (HSV-1) infection. ", "Therefore, the present study aimed to evaluate the role of two different forms of vitamin D: 25-hydroxyvitamin D3 (25D3) and 1,25-dihydroxyvitamin D3 (1,25D3) against HSV-1 in HeLa cells. ", "The HeLa cells were supplemented with either 25D3 or 1,25D3 before HSV-1 infection and were studied after 6, 12, and 24 h postinfection (p.i.). ", "The mRNA levels of toll-like receptors (TLRs), (2, 3, 4, 7, and 9), vitamin D signaling genes, and HSV-1 were studied using real-time PCR. ", "The HSV-1 DNA load was estimated in culture supernatant. ", "The supplementation of 25D3 and 1,25D3 significantly downregulated the mRNA levels of TLR2 (p < 0.0001) at 12 h p.i. ", "The mRNA levels of TLR9 were found to be significantly downregulated in 1,25D3-supplemented cells at 12 h p.i. ", "Furthermore, the significant downregulation was observed in HSV-1 titer in both 25D3- and 1,25D3-supplemented cells at 24 h p.i.(p < 0.0001). ", "However, the effect of 25D3 supplementation persisted till 24 h p.i. ", "with significant downregulation of TLR2 (p < 0.05) mRNA levels. ", "The supplementation of both 25D3 and 1,25D3 before HSV-1 infection was found to downregulate the viral titer and TLR2 mRNA during the intial phase of infection. ", "However, the effect of 25D3 supplementation was found to last for a longer duration compared with 1,25D3." ]
{ "pile_set_name": "PubMed Abstracts" }
[ 0.014184397163120567, 0, 0, 0.010638297872340425, 0, 0, 0, 0.008547008547008548, 0, 0.007042253521126761, 0, 0, 0, 0.009523809523809525 ]
0.003567
5
[ "Occipital neuralgia as a presenting symptom of cervicomedullary dural arteriovenous fistula.", "\nDural arteriovenous fistulas (DAVFs) at the cervicomedullary junction are rare and have a wide variation in presentation. ", "We report a case of occipital neuralgia (ON) as a rare presenting symptom of cervicomedullary DAVF causing intramedullary hemorrhage at the C1 level. ", "It is important to consider the underlying causes of ON, and precise neurological examinations and radiological evaluations are needed." ]
{ "pile_set_name": "PubMed Abstracts" }
[ 0, 0, 0.006666666666666667, 0.007407407407407408 ]
0.003519
5
[ "ters picked without replacement from {w: 5, s: 1, v: 1, x: 6, o: 3, c: 3}?", "\n0\nCalculate prob of picking 1 i and 1 x when two letters picked without replacement from ixxliili.", "\n2/7\nWhat is prob of picking 2 x when two letters picked without replacement from {a: 2, x: 10}?", "\n15/22\nTwo letters picked without replacement from qlxxtqxcqxttqtxxc. ", "Give prob of picking 1 c and 1 t.\n1/17\nTwo letters picked without replacement from eecjcjcjjthej. ", "Give prob of picking 2 j.\n5/39\nWhat is prob of picking 2 x and 1 k when three letters picked without replacement from kkxxkkkk?", "\n3/28\nTwo letters picked without replacement from {k: 17, y: 3}. ", "Give prob of picking 2 k.\n68/95\nWhat is prob of picking 1 v and 2 j when three letters picked without replacement from jjjjjjjjvjjjjjjjjjjj?", "\n3/20\nThree letters picked without replacement from {t: 5, l: 2, o: 2, z: 8}. ", "What is prob of picking 1 o, 1 t, and 1 l?", "\n1/34\nCalculate prob of picking 1 w and 1 s when two letters picked without replacement from xsxgwgxsxgkxxgxg.", "\n1/60\nThree letters picked without replacement from zzrzrrrrzrzrzrzzrz. ", "What is prob of picking 1 r and 2 z?", "\n27/68\nCalculate prob of picking 2 o, 1 a, and 1 c when four letters picked without replacement from ntantooxactc.", "\n4/495\nFour letters picked without replacement from aaakakaaak. ", "Give prob of picking 2 a and 2 k.\n3/10\nTwo letters picked without replacement from {n: 10, z: 2, j: 1, c: 3, r: 1}. ", "What is prob of picking 2 z?", "\n1/136\nCalculate prob of picking 1 w and 2 l when three letters picked without replacement from wwwwwllluwl.", "\n12/55\nCalculate prob of picking 1 w, 2 n, and 1 y when four letters picked without replacement from yyqnyyyiqywyqyyqqn.", "\n1/340\nTwo letters picked without replacement from niji. ", "Give prob of picking 1 n and 1 j.\n1/6\nTwo letters picked without replacement from {t: 6, z: 7, l: 3, o: 1}. ", "Give prob of picking 2 z.\n21/136\nCalculate prob of picking 1 d, 1 z, and 1 o when three letters picked without replacement from ddonzdzodo.", "\n1/5\nCalculate prob of picking 1 n and 1 e when two letters picked without replacement from {n: 2, c: 2, b: 1, e: 1, t: 1}.", "\n2/21\nTwo letters picked without replacement from ohffppfoafpoo. ", "Give prob of picking 2 o.\n1/13\nTwo letters picked without replacement from {k: 3, w: 1, u: 5}. ", "Give prob of picking 1 u and 1 k.\n5/12\nThree letters picked without replacement from nnnnynyynyyy. ", "Give prob of picking 3 y.\n1/11\nCalculate prob of picking 2 f when two letters picked without replacement from ccff.", "\n1/6\nWhat is prob of picking 1 n and 2 f when three letters picked without replacement from {n: 15, f: 3}?", "\n15/272\nThree letters picked without replacement from {g: 7, d: 6}. ", "What is prob of picking 3 d?", "\n10/143\nCalculate prob of picking 4 h when four letters picked without replacement from hbahbhbhhwj.", "\n1/66\nCalculate prob of picking 1 w and 1 d when two letters picked without replacement from {w: 1, z: 8, d: 4}.", "\n2/39\nWhat is prob of picking 2 h when two letters picked without replacement from zhch?", "\n1/6\nCalculate prob of picking 3 l when three letters picked without replacement from {w: 3, q: 9, r: 1, l: 4, x: 2}.", "\n4/969\nFour letters picked without replacement from {z: 2, x: 1, h: 3, s: 1, n: 1, i: 5}. ", "What is prob of picking 1 z, 2 h, and 1 n?", "\n6/715\nFour letters picked without replacement from {c: 2, h: 2, j: 1, o: 2, s: 1}. ", "Give prob of picking 1 j and 3 s.\n0\nThree letters picked without replacement from {f: 2, d: 5, e: 2, w: 3}. ", "What is prob of picking 1 w and 2 e?", "\n3/220\nWhat is prob of picking 3 s when three letters picked without replacement from {c: 2, r: 4, s: 4, k: 3, f: 3}?", "\n1/140\nThree letters picked without replacement from {y: 3, i: 2, o: 4, l: 1}. ", "Give prob of picking 2 o and 1 y.\n3/20\nWhat is prob of picking 2 u when two letters picked without replacement from uuuvuuuuuuuu?", "\n5/6\nThree letters picked without replacement from iiaiiiqqqiqiiiqqqqi. ", "What is prob of picking 2 q and 1 a?", "\n28/969\nCalculate prob of picking 1 x and 1 v when two letters picked without replacement from {a: 1, v: 1, x: 1}.", "\n1/3\nTwo letters picked without replacement from susuussssssssu. ", "Give prob of picking 1 s and 1 u.\n40/91\nCalculate prob of picking 2 x when two letters picked without replacement from {x: 17, u: 2}.", "\n136/171\nCalculate prob of picking 1 b and 1 y when two letters picked without replacement from pypbhjjjahpapyya.", "\n1/40\nWhat is prob of picking 2 d, 1 w, and 1 r when four letters picked without replacement from {o: 1, u: 1, d: 2, l: 1, w: 1, r: 1}?", "\n1/35\nFour letters picked without replacement from mmmmmmmmm. ", "Give prob of picking 4 m.\n1\nWhat is prob of picking 4 w when four letters picked without replacement from {t: 1, w: 12}?", "\n9/13\nFour letters picked without replacement from {u: 2, r: 2, z: 6, a: 2, w: 3}. ", "What is prob of picking 1 a, 1 r, and 2 u?", "\n4/1365\nCalculate prob of picking 1 i and 2 j when three letters picked without replacement from {o: 2, j: 5, i: 1, a: 1}.", "\n5/42\nTwo letters picked without replacement from ooaftopofototfaappa. ", "What is prob of picking 1 f and 1 o?", "\n2/19\nThree letters picked without replacement from xrrrrqrxrqrxxx. ", "Give prob of picking 3 x.\n5/182\nWhat is prob of picking 1 j, 1 e, and 1 r when three letters picked without replacement from {e: 1, j: 6, r: 4}?", "\n8/55\nTwo letters picked without replacement from {k: 1, g: 2, d: 1, e: 2}. ", "What is prob of picking 2 g?", "\n1/15\nCalculate prob of picking 1 v and 1 z when two letters picked without replacement from {v: 1, z: 1, h: 1, g: 9}.", "\n1/66\nWhat is prob of picking 2 l when two letters picked without replacement from {s: 11, l: 6, e: 1}?", "\n5/51\nWhat is prob of picking 2 f when two letters picked without replacement from nenageefennnfngegen?", "\n1/171\nTwo letters picked without replacement from {d: 12, a: 3, g: 2}. ", "What is prob of picking 2 g?", "\n1/136\nTwo letters picked without replacement from {v: 6, a: 4, f: 4}. ", "What is prob of picking 2 f?", "\n6/91\nFour letters picked without replacement from {w: 3, b: 1, a: 4, c: 7, r: 3}. ", "What is prob of picking 2 c, 1 b, and 1 w?", "\n7/340\nCalculate prob of picking 1 d and 3 l when four letters picked without replacement from llllllldd.", "\n5/9\nCalculate prob of picking 1 t and 1 w when two letters picked without replacement from {v: 1, w: 2, k: 2, t: 1}.", "\n2/15\nTwo letters picked without replacement from {e: 6}. ", "What is prob of picking 2 e?", "\n1\nWhat is prob of picking 4 t when four letters picked without replacement from ttvjjvvttvttjttv?", "\n1/26\nThree letters picked without replacement from {j: 1, m: 1, r: 3, p: 6, y: 6}. ", "What is prob of picking 2 y and 1 m?", "\n3/136\nFour letters picked without replacement from rxrxxxxxxx. ", "Give prob of picking 4 x.\n1/3\nThree letters picked without replacement from ddpqqdqqdqppqqdq. ", "Give prob of picking 1 d and 2 p.\n3/112\nCalculate prob of picking 1 n, 1 y, and 1 c when three letters picked without replacement from yyyucnuyyu.", "\n1/24\nCalculate prob of picking 1 d and 2 g when three letters picked without replacement from hxhmdxgddgdhmdd.", "\n6/455\nFour letters picked without replacement from {b: 2, l: 8}. ", "What is prob of picking 2 l and 2 b?", "\n2/15\nThree letters picked without replacement from {u: 3, y: 1}. ", "Give prob of picking 3 u.\n1/4\nCalculate prob of picking 1 f and 1 k when two letters picked without replacement from {k: 8, a: 7, f: 3}.", "\n8/51\nCalculate prob of picking 4 s when four letters picked without replacement from {c: 4, s: 5, g: 6}.", "\n1/273\nCalculate prob of picking 1 x and 1 s when two letters picked without replacement from isxsix.", "\n4/15\nCalculate prob of picking 1 d, 1 r, and 1 i when three letters picked without replacement from dnirdnq.", "\n2/35\nWhat is prob of picking 1 s and 1 b when two letters picked without replacement from ssbsbsssssbs?", "\n9/22\nCalculate prob of picking 2 j when two letters picked without replacement from {j: 7, u: 1, f: 4, x: 4, g: 4}.", "\n21/190\nCalculate prob of picking 1 i, 1 c, and 1 w when three letters picked without replacement from {c: 3, i: 1, u: 1, p: 1, w: 1}.", "\n3/35\nWhat is prob of picking 2 n and 1 a when three letters picked without replacement from {a: 6, n: 6}?", "\n9/22\nFour letters picked without replacement from gnlannlvanljannnj. ", "What is prob of picking 2 n, 1 a, and 1 j?", "\n9/170\nCalculate prob of picking 1 v and 1 s when two letters picked without replacement from vspvgp.", "\n2/15\nFour letters picked witho" ]
{ "pile_set_name": "DM Mathematics" }
[ 0, 0, 0, 0, 0, 0, 0, 0, 0, 0, 0, 0, 0, 0, 0, 0, 0, 0, 0, 0, 0, 0, 0, 0, 0, 0, 0, 0, 0, 0, 0, 0, 0, 0, 0, 0, 0, 0, 0, 0, 0, 0, 0, 0, 0, 0, 0, 0, 0, 0, 0, 0, 0, 0, 0, 0, 0, 0, 0, 0, 0, 0, 0, 0, 0, 0, 0, 0, 0, 0, 0, 0, 0, 0, 0, 0, 0, 0, 0, 0, 0, 0, 0, 0, 0, 0, 0, 0, 0, 0, 0, 0, 0, 0, 0 ]
0
5
[ "Unlu has asked of the prime minister, “Why were these two senior MIT officials sent to the region? ", "What is the legal basis for sending MIT officials with diplomatic passports to the Kurdistan federal region to carry out such an operation in another country?”", "\n\n“The prime minister is not responding to our queries to avoid exposing MIT operations in that region,” Unlu told Al-Monitor. “", "The PKK had earlier claimed several times that the MIT had attempted assassinations. ", "It said the culprits were caught by the PKK and exposed. ", "This is the first time that MIT’s officials were themselves being used in such an operation instead of freelancers.”", "\n\nAccording to the PKK, its forces captured two senior MIT officers on Aug. 3 near Sulaimaniyah, in Iraqi Kurdistan, where they were preparing for an operation against senior PKK leader Cemil Bayik. ", "The PKK went public about the abductions on Aug. 30. ", "Pro-government media reported that the MIT officers had been dispatched to capture Bayik, while Kurdish media and PKK publications claimed the goal was to assassinate Bayik .", "\n\nAlican Unlu, a representative from the pro-Kurdish Peoples’ Democratic Party (HDP) who has filed a parliamentary query, told Al-Monitor that the abducted MIT personnel had been tasked with assassinations, which is why the government is maintaining its silence.", "\n\nAn undeclared number of senior officers from Turkey’s National Intelligence Organization (MIT) have been held by the Kurdistan Workers Party (PKK) for more than three months, but the government continues to refuse to respond to opposition requests for information on the situation. ", "The refusal of intelligence chief Hakan Fidan and Prime Minister Binali Yildirim to appear before the legislature or otherwise provide answers to parliamentary inquiries has led the opposition to conclude that the government is trying to avoid acknowledging that the MIT has been engaged in “activities exceeding its mission.”", "\n\nThe main opposition Republican People’s Party (CHP) is also interested in the affair and has submitted several queries.", "\n\n“How many MIT officials are hostages? ", "What are their names and duties? ", "We are not told anything,” CHP lawmaker Musa Cam told Al-Monitor. “", "The MIT chief is not informing the parliament. ", "He has to give an account, but his boss, the president, is not letting him. ", "The MIT chief has instructions from the president to totally ignore the parliament.”", "\n\nAt the moment, the parliament does not even know how many MIT personnel are actually missing. ", "The ANF news agency, which is close to the PKK and first reported on the situation, said only that two MIT personnel with diplomatic passports had been seized.", "\n\nAl-Monitor broke the first and only official statement, from Foreign Minister Mevlut Cavusoglu, confirming the abductions. “", "We have no contacts with the PKK to secure the return of those two persons,” Cavusoglu said in an exclusive interview in September.", "\n\nIn an address to the parliament, another CHP lawmaker, Tuncay Ozkan, asserted that the PKK is holding four MIT personnel captive. ", "Reiterating his claim, Ozkan told Al-Monitor, “The PKK is holding four MIT personnel, two department heads and two senior officials. ", "They are holding a total of 22 people.” ", "Also included in the total are military personnel and village guards.", "\n\nThe missing personnel are on the agenda of the parliament's Security and Intelligence Commission, which has oversight over MIT and whose meetings are behind closed doors. ", "Ozkan said the commission had requested Fidan to testify about the missing personnel, but he has so far refused to do so.", "\n\n“Senior MIT personnel are held captive. ", "How can we be denied information about this affair?” ", "Ozkan told Al-Monitor. “", "We want to learn how our personnel were abducted. ", "Does anyone know why MIT is launching such operations and refusing to discuss it?”", "\n\nAn emergency decree issued under the state of emergency, and thus with the power of law, was passed immediately after the abductions to move MIT from under the authority of the Prime Ministry to the presidency. ", "In corridors in Ankara, rumor has it that the abortive Bayik operation had prompted the move. ", "If the operation had succeeded, however, it would have been presented as a success for President Recep Tayyip Erdogan and certainly would have been exploited in campaigns for next year's local, general and presidential elections.", "\n\nThe failure of the operation has instead made life difficult for Erdogan. ", "Ozkan explained, “You catch Ocalan abroad and bring him to Turkey. ", "Then you lose the department heads who had been fighting [his] organization for 16 years. ", "This is not something you can sweep under the carpet.”", "\n\nAccording to the CHP, the MIT's recent failures stem from security policies implemented by Erdogan. ", "The CHP's Cam stated in his parliamentary query, “Developments that followed putting MIT under the AKP [Justice and Development Party] chairman [Erdogan] signal changes in the structure and policies of the organization.”", "\n\nThat Erdogan, through the emergency decree, would be deciding on exchanges of detainees and prisoners also sparked controversy. ", "The CHP's Ozkan asked in his parliamentary questions, “Why did you authorize President Erdogan to decide on such exchanges? ", "Is this authorization aimed at securing the release of department heads held by the PKK?”" ]
{ "pile_set_name": "OpenWebText2" }
[ 0.010101010101010102, 0.006289308176100629, 0.015625, 0.023529411764705882, 0.017543859649122806, 0.008620689655172414, 0.020100502512562814, 0.018867924528301886, 0.022988505747126436, 0.015267175572519083, 0.014084507042253521, 0.009202453987730062, 0.01652892561983471, 0.025, 0, 0.014925373134328358, 0.02127659574468085, 0, 0.011904761904761904, 0.010416666666666666, 0.018867924528301886, 0.007936507936507936, 0.015267175572519083, 0.030303030303030304, 0.03007518796992481, 0, 0, 0.011560693641618497, 0.01652892561983471, 0.023809523809523808, 0, 0.08333333333333333, 0, 0.012195121951219513, 0.009389671361502348, 0.010638297872340425, 0.004366812227074236, 0.013157894736842105, 0.014925373134328358, 0, 0, 0.029411764705882353, 0.022727272727272728, 0.007692307692307693, 0.024193548387096774, 0.011235955056179775 ]
0.01478
5
[ "Many types of tobacco smoke modifying agents are known in the art to be added to smoking products to modify the tobacco smoke. ", "For example, flavorants are added to smoking products to enhance their taste and to compensate for variations in tobacco quality and blend. ", "Although flavorants are traditionally applied to the tobacco portion of the smoking product, this practice results in only a small fraction of the flavorant ever reaching the smoker. ", "Most of a flavorant added to the tobacco is lost in the sidestream smoke produced during the static burn period of the smoking article or is removed by the smoke filter. ", "The low flavorant delivery efficiencies associated with application on tobacco necessitates the use of relatively large quantities of flavorant to achieve the desired effect. ", "Because many of these flavorants, such as menthol, for example, are expensive, inefficient utilization can add significantly to the cost of the smoking product. ", "In addition, flavorants applied to the tobacco are subjected to the high heat of combustion which can undesirably alter their organoleptic characteristics.", "\nIn response to these problems, there has been substantial effort to apply flavorants to the filter. ", "It was shown many years ago that smoke aerosols could transport significant quantities of relatively non-volatile materials from a structure of moderate surface area, even though a gas at a comparable temperature is ineffective in this regard. ", "Attempts at the practical implementation of this phenomenon using cellulose acetate filters revealed, however, that although aerosols transported flavorant very efficiently from freshly made filters, this advantage was lost as the flavorant diffused away from the surface and into the bulk of the filter fibers.", "\nEfforts to solve this problem by using polymers impermeable to the flavorants, such as polypropylene, eliminated the time dependence of flavorant delivery observed with cellulose acetate filters, but did not permit the development of a functional flavorant delivery system. ", "The causes of this failure were, first, the flavorant delivery efficiencies for these nonpermeable polymer systems were too low to be useful, and second, impermeable filter media had no affinity for the flavorant which consequently diffused to the tobacco where it endured the same fate as flavorants applied directly to the tobacco.", "\nIn spite of years of concerted effort, neither the cigarette nor the filter material industry has developed an efficient general flavorant delivery system that does not absorb or lose the flavorant over time.", "\nPrior art of this area reflects a strong interest in technology for the efficient and consistent delivery of tobacco smoke modifying agents, especially flavorants. ", "However, the abundant patented technologies for flavorant delivery almost invariably employ one of the following four strategies:\n1. ", "A flavorant is contained by some physical means and is released either by mechanical destruction of the containment apparatus or by controlled leakage (see, for example, U.S. Pat. ", "Nos. ", "3,219,041; 3,297,038; 3,339,557; and 4,720,423).", "\n2. ", "A flavorant is adsorbed on a material whose surface has been customized so that the flavorant will be displaced by the moisture or heat in the smoke (see, for example, U.S. Pat. ", "Nos. ", "3,236,244; 3,280,823; and 4,662,384).", "\n3. ", "A flavorant is absorbed in a polymeric matrix and is then released by the plasticizing action of moisture or heat in the smoke (see, for example, U.S. Pat. ", "Nos. ", "4,662,384; 3,144,024; and 4,729,391). ", "A portion of the prior art in this area addresses the concept of modifying the fiber shape or filter geometry of current cellulose acetate filters to achieve improved flavorant containment or delivery (see, for example, U.S. Pat. ", "Nos. ", "4,180,536, 4,619,279; and 4,821,750).", "\n4. ", "A flavorant undergoes a chemical reaction with another compound to form a new compound that will regenerate the original flavorant upon thermal decomposition (see U.S. Pat. ", "No. ", "3,288,146).", "\nAlthough there is substantial prior art, virtually every implementation of this art possesses limitations which render its commercial application impractical. ", "These limitations are largely defined by the flavorant delivery strategy employed and will, therefore, be so organized here.", "\nMechanical or physical flavorant containment devices which are incorporated into the filter and ruptured prior to smoking are very complex and expensive to produce. ", "They introduce significant variation into the performance of the smoking article because of inconsistencies in the pattern of their breakage, and they interfere with the normal function of the filter by altering smoke flow through the filter. ", "They also increase the effort and complexity to the consumer who uses the product.", "\nAdsorbed flavorants which are incorporated into the filter and released by the heat or moisture content of the smoke are not efficiently delivered until enough of the smoking article has been consumed to allow adequate moisture and heat to reach the filter. ", "As a consequence, the flavorant is not available to augment smoke taste during the first few puffs, when it is generally acknowledged as being most needed. ", "In addition, absorbants must be customized to achieve the desired release characteristics for each flavorant and, therefore, are not useful for delivering naturally occurring flavoring materials which consist of large numbers of independent chemical entities.", "\nAbsorbed flavorants which are dissolved in polymer matrices and released by the plasticizing action of moisture or heat in the smoke are subject to the same limitations as adsorbed flavorants. ", "In addition, absorbed flavorants are subject to time dependent losses in delivery efficiency because of diffusion of the flavorant into the bulk of the fiber polymer. ", "This limitation is especially evident when a conventional cellulose acetate filter is used as the flavorant absorber.", "\nDerivatized flavorants are almost always inappropriate for use in filter flavorant delivery systems because relatively high temperatures are required for their release. ", "Derivatized flavorants are, therefore, typically applied to the tobacco portion of the smoking product, where the liberated flavorant produced during combustion is subject to chemical alteration and loss during the static burn period of the smoking article. ", "The development of derivatized flavorants is highly specific for each flavorant and, therefore, excludes naturally occurring flavoring materials which are composed of a large number of independent chemical entities.", "\nAlthough flavorants are the most commonly used tobacco smoke modifying agents, selective removal additives can also serve as tobacco smoke modifying agents. ", "In contrast to flavorants, selective removal additives modify tobacco smoke by removing, rather than adding, certain compounds or classes of compounds. ", "Selective removal additives are applied to the filter and, therefore, like flavorants, can be absorbed by the filter fibers and lose their effectiveness. ", "Here, too, significant improvements in the performance of selective removal additives could be achieved by overcoming the limitations imposed by the substrate to which the additives are applied.", "\nWe have unexpectedly discovered that if spontaneously wettable fibers described below are combined with a conventional additive, and used in a tobacco smoke filter in accordance with this invention, enhanced flavor and filtering are realized. ", "Preferably, the spontaneously wettable fibers are formed into a nonwoven web and used as a wrap around a conventional tobacco smoke filter, i.e., as a circular layer between the conventional fibrous filter and the conventional filter wrap.", "\nPatents of interest further include U.S. Pat. ", "No. ", "4,807,809 which relates a filter rod making apparatus, and U.S. Pat. ", "No. ", "5,105,834 which relates to cigarette filter containing a spray extract." ]
{ "pile_set_name": "USPTO Backgrounds" }
[ 0, 0, 0, 0, 0, 0, 0, 0, 0, 0, 0, 0, 0, 0, 0, 0, 0, 0, 0, 0, 0, 0, 0, 0, 0, 0, 0, 0, 0, 0, 0, 0, 0, 0, 0, 0, 0, 0, 0, 0, 0, 0, 0, 0, 0, 0, 0, 0, 0, 0, 0, 0, 0, 0, 0, 0, 0, 0 ]
0
5
[ "Environmental monitoring of water resources around a municipal landfill of the Rio Grande do Sul state, Brazil.", "\nIn Brazil, landfills are commonly used as a method for the final disposal of waste that is compliant with the legislation. ", "This technique, however, presents a risk to surface water and groundwater resources, owing to the leakage of metals, anions, and organic compounds. ", "The geochemical monitoring of water resources is therefore extremely important, since the leachate can compromise the quality and use of surface water and groundwater close to landfills. ", "In this paper, the results of analyses of metals, anions, ammonia, and physicochemical parameters were used to identify possible contamination of surface water and groundwater in a landfill area. ", "A statistical multivariate approach was used. ", "The values found for alkali metals, nitrate, and chloride indicate contamination in the regional groundwater and, moreover, surface waters also show variation when compared to the other background points, mainly for ammonia. ", "Thus, the results of this study evidence the landfill leachate influence on the quality of groundwater and surface water in the study area." ]
{ "pile_set_name": "PubMed Abstracts" }
[ 0, 0, 0, 0, 0, 0, 0, 0 ]
0
5
[ "'''Get a push notification when severe weather is in your area.", "'''=Budete automaticky upozorněni, když je ve vaší oblasti extrémní počasí.", "\n'''Note:'''=Poznámka:\n'''Zip code'''=Poštovní směrovací číslo\n'''Send notifications to'''=Odesílat oznámení na\n'''Phone Number 1'''=Telefonní číslo 1\n'''Phone Number 2'''=Telefonní číslo 2\n'''Phone Number 3'''=Telefonní číslo 3\n'''Weather Alert! {{", "alert.description}} from {{alert.date}} until {{alert.expires}}'''=Varování před špatným počasím! {{", "alert.description}} od {{alert.date}} do {{alert.expires}}\n'''Severe Weather Alert'''=Závažná upozornění na počasí\n'''Set for specific mode(s)'''=Nastavit pro konkrétní režimy\n'''Assign a name'''=Přiřadit název\n'''Tap to set'''=Nastavte klepnutím\n'''Phone'''=Telefonní číslo\n'''Which?'''=Který?", "\n'''Choose Modes'''=Zvolte režim\n'''Set your location'''=Nastavte svou polohu\n'''Away'''=Pryč\n'''Home'''=Doma\n'''Night'''=Noc\n'''Add a name'''=Přidejte název\n'''Tap to choose'''=Klepnutím zvolte\n'''Choose an icon'''=Zvolte ikonu\n'''Next page'''=Další stránka\n'''Text'''=Text\n'''Number'''=Číslo\n" ]
{ "pile_set_name": "Github" }
[ 0, 0.02666666666666667, 0, 0.01, 0, 0.003401360544217687 ]
0.006678
5
[ "UPDATED August 2020: The Input System is now is now verified for Unity 2019 LTS and later. ", "See this page for up to date information.", "\n\nUPDATED Dec 12, 2017: We have made significant changes to our plans for the input system. ", "Please read our forum post for details.", "\n\nIn Input Team we’ve been working on designing and implementing a new input system. ", "We’ve made good progress, and though there’s still a long way to go, we want to get you involved already now.", "\n\nWe’ve build a new foundation for working with input in a new way that we’re excited to show you, and we want to continue the development with your input on the existing and continued design.", "\n\nDevelopment process\n\nThe new input system will consist of two parts. ", "The low-level part is integrated into the C++ core of Unity. ", "The high-level part is implemented in managed (C#) code that will be open-source in the same way as e.g. the UI system.", "\n\nOur development process for the new input system is to design and implement large parts of the high level system first. ", "Initially this is based on top of the current input system that already exists in Unity. ", "For now, we call this the input system prototype. ", "Later, once the new low-level core is more mature, we’ll change the high-level part to be based on the new low-level system.", "\n\nThis means that the current high-level system (the prototype) lacks specific features that depend on the new low-level core, such as robust registration of connected and disconnected input devices while the game is running. ", "However, many features of the design can already be used and tested, and this is particularly what we want early feedback on.", "\n\nWorking with you\n\nHere’s how we’d like you to get involved:\n\nLearn about the design of the new system Try out the new input system prototype for yourself Tell us about your experience\n\nLearn about the design of the new system\n\nInput that works well for a plethora of different use cases is a surprisingly tricky matter. ", "We’ve prepared some resources for you to learn about how the new design attempts to address this.", "\n\nFirst of all, we’ve created this video covering the design of action maps and player management. ", "It’s a good introduction to the new design.", "\n\nWe’ve also prepared a site with more information about the design, including a Quick Start Guide.", "\n\nHead to the Experimental New Input System site to learn more.", "\n\nTry out the new input system prototype for yourself\n\nWe have a project folder which contains the input system prototype as well as a demo project which uses it. ", "This can be used with regular Unity 5.3 without needing a special build.", "\n\nDownload it from the Experimental New Input System site.", "\n\nThe input system prototype can be tested with other projects by copying the folder Assets/input-prototype into the Assets folder of another project. (", "Please create a backup of your project first.)", "\n\nTell us about your experience\n\nWhat do you think about the design? ", "How does it work (or not) for your project? ", "Anything that’s confusing or unclear? ", "For now we’re interested in discussion around the design of the system. ", "We don’t need bug reports quite yet at this stage in development.", "\n\nHead to the New Input System Forum to discuss the new input system.", "\n\nWe’re looking forward to working with you!" ]
{ "pile_set_name": "OpenWebText2" }
[ 0.02197802197802198, 0, 0, 0, 0.011764705882352941, 0, 0, 0, 0.01639344262295082, 0.008403361344537815, 0, 0.011235955056179775, 0, 0, 0, 0, 0, 0, 0, 0, 0.010101010101010102, 0.015873015873015872, 0, 0, 0.017241379310344827, 0, 0, 0, 0, 0, 0, 0, 0.014492753623188406, 0 ]
0.00375
5
[ "Ali James\n\nAli James (born 11 October 1979 in Rugby, Warwickshire) is an English rugby union footballer, currently playing for Esher. ", "A product of the Gloucester Rugby Academy, he moved to Newbury before rejoining Gloucester Rugby in the summer of 2007. ", " He plays as a centre. ", " During the 2007/08 season he played for Moseley as a loan player.", "\n\nExternal links\nGloucester Rugby profile\n\nCategory:Living people\nCategory:1979 births\nCategory:English rugby union players\nCategory:Sportspeople from Rugby, Warwickshire" ]
{ "pile_set_name": "Wikipedia (en)" }
[ 0.022388059701492536, 0.025, 0, 0.015151515151515152, 0.0058823529411764705 ]
0.013684
5
[ "CHARLOTTESVILLE, VA – Snowflakes throughout the American south are once again up in arms, after Charlottesville city council passed a unanimous vote to remove the city’s statue of confederate general Thomas “Stonewall” Jackson, a participation trophy awarded for Jackson’s failed attempt at winning the Civil War.", "\n\nJust weeks after the city became a flashpoint of controversy for voting to take down the large bronze pat on the head it once awarded to second-place general Robert E. Lee, southern snowflakes and crybabies (and northern conservative virtue-signallers) are once again experiencing mass triggering over the planned removal of Jackson’s statue, erected In Charlottesville’s Emancipation Park as a reward for doing his best.", "\n\n“Credit where credit’s due, Jackson made a great effort in the Civil War. ", "He fought really hard in that war and gave it 110 percent, often at a severe tactical disadvantage,” explained Mayor Mike Signer. “", "But let’s be real… he didn’t win. ", "In fact, he was accidentally shot by his own side and died of pneumonia, which a decidedly non-winner move. ", "But somewhere along the line, people decided that everybody should get a statue just for being in a war.”", "\n\n“And of course, that just leads to a generation of whiny, entitled white nationalists, who think slave owners who go to war with their own nation deserve a prize just for showing up,” added Signer. “", "Honestly, it’s no wonder these guys can’t even make their own torches anymore. ", "They’ve been taught that it’s okay not to try.”", "\n\nWhile the majority of the snowflakes are like, “muh heritage,” many Charlottesville residents reject the argument that the monument is crucial to the preservation of Confederate history. “", "When I was 10, I was on a softball team,” said local contractor Andre Wright. “", "Near the end of the season our coach said that if we won this one really big game, he’d take us all to Pizza Hut. ", "But we didn’t win the game, so we didn’t go.”", "\n\n“It was a drag, but not going to Pizza Hut didn’t erase me from history, and people still knew who I was,” he explained. “", "Anyway, I feel like the same lessons apply here, except Pizza Hut is statues and softball is treason.”", "\n\nWhile neither Jackson or Lee’s statues can be immediately removed due to pending lawsuits, council says they’re confident the work can be completed quickly once the wahmbulance rolls out of town." ]
{ "pile_set_name": "OpenWebText2" }
[ 0.009584664536741214, 0.004728132387706856, 0.013157894736842105, 0.007633587786259542, 0, 0, 0, 0.004975124378109453, 0, 0, 0.005263157894736842, 0.012658227848101266, 0.008771929824561403, 0, 0.008064516129032258, 0.00980392156862745, 0.01015228426395939 ]
0.005576
5
[ "Accommodations\n\nFeatured\n\nWarm dark wood meets contemporary design in this chalet-style suite. ", "Entertain in the living room, complete with a fireplace and wrap-around balcony, or escape to your deep soaking tub after a day on the slopes.", "\n\nA rooftop escape with modern Alpine styling and floor-to-ceiling windows make for an chic mountain retreat. ", "Settle in after a day outdoors in front of your fireplace or outside on your valley-view terrace.", "\n\nPolicies\n\nCheck-in Time: 3:00 PM Check-out Time: 12:00 PM\n\nIf you expect to arrive early in the day and would like immediate access to your room, we recommend booking the room for the prior night to guarantee immediate access. ", "Similarly, for late departures, reserving an additional night will guarantee access to your guest room until you depart. ", "If you choose not to reserve, we will be glad to store your bags and make our fitness facilities available for you to freshen up.", "\n\nAt Four Seasons, a guaranteed reservation assures you of a room even if you check in late (after 6:00 pm). ", "If a room is not available, we will arrange your accommodation in another hotel at our expense, and provide transportation to and from Four Seasons as reasonable. ", "All reservations made through the website must be guaranteed by a major credit card. ", "Certain arrival dates and rates may require a deposit. ", "Please check for full deposit requirements at time of booking.", "\n\nCancellation and deposit requirements may differ by arrival date, room type and offer booked. ", "For full details, please check at time of booking.", "\n\nAmerican Express, Visa, Diners Club, JCB, Mastercard, Maestro\n\nFour Seasons charges no additional fee for children 18 years of age and younger occupying the same guest room as their parents or guardians (space permitting). ", "Special rates may be available for two adults and two children 18 years of age and younger occupying two rooms, subject to availability. ", "Age limits for children occupying parents' or guardians' guest rooms at no additional charge may vary at some properties. ", "For reservations and information, please contact the Hotel directly.", "\n\nWe use cookies in accordance with our Cookie Notice. ", "Please refer to our Cookie Notice here for more information on how we use cookies and how to adjust your browser settings to block cookies. ", "If you consent to the use of cookies, please click Accept. ", "We will not ask for consent for strictly necessary cookies, but if you choose to block strictly necessary cookies this may affect your website experience." ]
{ "pile_set_name": "Pile-CC" }
[ 0, 0, 0.00909090909090909, 0.010309278350515464, 0.004366812227074236, 0, 0, 0, 0, 0, 0, 0, 0, 0, 0.022222222222222223, 0, 0, 0.014705882352941176, 0, 0, 0, 0 ]
0.002759
5
[ "Dusk Network Development Update — July\n\nA monthly development update series created to embrace the transparency and inclusivity that we strive for at Dusk Network.", "\n\nWe’re back with another update! ", "An extremely important month for the project has come to an end. ", "From the public release of the first iteration of the Testnet (called Shin (殉星)) to the launch of DUSK token on numerous exchanges, July has been a non-stop whirlwind of events. ", "Readers are invited to join us for this recap of the past month.", "\n\nDevelopment\n\nJuly marked the release of Testnet v1 Shin (殉星) and accompanying tools. ", "Aside from a few minor hiccups, the launch has gone smoothly with more than 50 nodes participating in the consensus producing a block once every 7 seconds on average. ", "This Testnet release marks the beginning of the public testing phase of the protocol which will proceed until the launch of our Mainnet, scheduled for late 2019.", "\n\nWith the Testnet v1 finally revealed to the gazes of the public, the team had continued working on the core protocol and all accompanying modules. ", "The publication of ZeroCaf will speed up the work on the zero-knowledge Virtual Machine (VM) and real-life implementation of the XSC standard. ", "As mentioned in previous articles, ZeroCaf is an efficient implementation of a zero-knowledge proof-friendly elliptic curve on which to build the cryptographic primitives. ", "As two new developers join our ranks (stay tuned for a separate article about our new recruits!), ", "who will aid in accelerating the work on the VM, we are still on track to complete the VM development in time for our Testnet v2 launch.", "\n\nResearch\n\nFollowing the launch of the Testnet, our research into block compression has intensified as the first days of testing had already provided the team with concrete proof that the consensus can achieve a major speed up if coupled with block compression. ", "Specifically, the team has been experimenting with Graphene and Txilm, both of which enable to drastically decrease the sizes of propagated blocks.", "\n\nThe core consensus has been on a receiving end of an important theoretical upgrade that significantly decreased the ability of the Adversary to bias the committee extraction in his/her favour. ", "The upgrade tackles the seed generation procedure, the security of which prior to the aforementioned upgrade depended on a relatively strict computational bound imposed on the Adversary, which meant that the Adversary gained an advantage as his/her computational power increased. ", "However, following the upgrade, the ability of the Adversary to bias the seed (outside a negligible bound predefined by the protocol) has been decreased to a negligible probability without regard to the computing power that the Adversary has in his/her possession.", "\n\nCommunity\n\nThe launch of the Testnet is not the only major milestone achieved before the dusk (no pun intended!) ", "of July. ", "DUSK has been launched on Binance DEX, Bitfinex and Ethfinex, Bittrex International, Binance, BitcoinMeester and Bitladon, Litebit.eu and Coinmerce exchanges. ", "In addition, DUSK has been fully integrated with the Delta cryptocurrency portfolio app, enabling users to track DUSK in real-time and be instantly notified about all the latest announcements.", "\n\nThe team had also participated in numerous AMAs, including an Ethfinex AMA, Vietnamese Blockchain Community AMA and CryptoDiffer AMA. ", "Aside from that, Carlos Perez has given a presentation on Confidential Smart Contracts at Barcelona Ethereum Dev Meetup, while the video from Luke Pearson’s presentation on Confidential Smart Contracts for Securities at ZKLux#1 has been published on Youtube. ", "For a more business-centered discussion, Jelle Pol had joined Brad Laurie to give an extended interview.", "\n\nA community campaign has been successfully concluded, garnering in excess of 64,000 participants enabling us to grow Dusk Network’s social media presence immensely. ", "The results of the campaign will be communicated to the participants within 28 days of the campaign completion.", "\n\nOn the advisory front, Balder Bomans had joined Dusk Network as a Strategic Advisor. ", "Balder is the Head of Investment and Trading at Maven11 Capital and an experienced blockchain technology investor who has a rich traditional financial markets background.", "\n\nDusk — Technology for Securities\n\nDusk streamlines the issuance of digital securities and automates trading compliance with the world’s first programmable and confidential securities." ]
{ "pile_set_name": "OpenWebText2" }
[ 0.012269938650306749, 0, 0, 0.0056179775280898875, 0, 0.011494252873563218, 0, 0, 0, 0.02097902097902098, 0.005813953488372093, 0, 0, 0, 0.013605442176870748, 0, 0, 0, 0, 0, 0.03773584905660377, 0, 0.022058823529411766, 0.019305019305019305, 0.019230769230769232, 0.005988023952095809, 0, 0.022988505747126436, 0, 0 ]
0.00657
5
[ "Granada urban buses\n\nThe city buses of Granada is a public transport system from the city of Granada (Spain) , operated by the companies Transportes Rober , Alhambra Bus and Herederos de Gómez. ", "Granada is one of the Spanish cities where the bus is most used, with 160 trips per inhabitant every year.", "\n\nHistory \nThe first public transport route in Granada was a tram connecting Paseo de la Bomba with Plaza Nueva, inaugurated 7 July 1904 operated by Tranvías Eléctricos de Granada (Electric Trams of Granada). ", "Until the 1960s, when trams were gradually substituted by buses, the company operated 90 kilometers of tramways.", "\n\nIn 1962, the Madridian bus company Transportes Rober, founded in 1957, won the competition to operate the bus line system in Granada. ", "The concession has been constantly renovated up until 2020, but the city council can rescue the service by paying a compensation.", "\n\nDuring the 1990s, Transportes Rober created an affiliate company, Alhambra Bus, to operate the lines which went through the Alhambra, Albayzin, Sacromonte and Realejo districts. ", "Alhambra Bus uses minibuses due to the narrowness of the streets. ", "Also, a new line was created which went to the nearby town of Cenes de la Vega , operated by a separate company, Herederos de Gómez.", "\n\nIn 2002, the Metropolitan Transport Consortium of the Granada Area was created, and the urban bus network was integrated within it.", "\n\nFleet \nThe Transportes Rober fleet is composed of approximately 150 buses with an average age of 6 years. ", "Most of the fleet is composed of Iveco chassis buses with 3 doors and 2 PRM spaces, although there are Volvo and Scania chassis buses. ", "These buses have glass partition on the driver’s seat to avoid robbery. ", "Also, some of the buses have upper compartments for suitcases (for lines that go to the bus station)\n\nThe Alhambra Bus fleet is composed of Iveco-Mercedes 2 door minibuses, some of them with a space for PRM. ", "Herederos de Gómez’s buses are similar to those of Transportes Rober, but they only have two doors and no glass separations between the driver and passengers.", "\n\nAll buses are red (except LAC line buses, which are blue with 4 doors), have air conditioning and can sell tickets on board.", "\n\nTicket prices \nA ticket costs 1,40€ and is valid for 60 minutes. ", "A night bus ticket costs 1,50€. Travelcards (known as Credibús) are available with a cost of 0,87€, 0,85€ and 0,83€ for 60 minutes when charged 5€, 10€ and 20€ respectively. ", "Junior and University students Travelcards (Bono Joven and Bono Universitario) have similar prices. ", "Travelcard holders must pay a 2€ deposit. ", "Fair buses have a cost of 2€. Monthly passes cost 41€, while pensioners living in Granada can travel for free.", "\n\nBus routes \nThe current bus line scheme in Granada is organised with transport exchanges on the northern part of the city (Caleta and Triunfo) and the southern part (Paseo del Violón). ", "Lines either connect these two locations or go from the transport exchanges to peripheral districts. ", "Each line has a letter followed by a number. ", "Letters identify which part of the city lines go through and numbers go from 0 to 9 (0 if it doesn’t connect with a transport exchange and 1 to 9 following a clockwise starting point order in the north and anticlockwise starting point order in the south):\n\n LAC line (Línea de Alta Capacidad, High Capacity Line). ", "It’s the main line going through the city center.", "\n C lines. ", "They go through the central part of the city (this includes the historical districts).", "\n N lines. ", "They connect the northern transport exchanges with northern districts.", "\n S lines. ", "They connect the southern transport exchange with southern districts.", "\n U lines. ", "They connect various parts of the city with the different campuses of the University of Granada.", "\n SN lines. ", "They connect the southern part with the northern part of the city. ", "Originally all of these lines went through Camino de Ronda, now some of these lines go through the city center.", "\n Lines 111 and 121. ", "They are clockwise and anticlockwise night routes around the city. ", "Only operate weekends and holiday eves.", "\n F lines. ", "During Corpus Christi week, these lines connect all districts with the Fair in the Almanjayar district. ", "They have different prices to the other lines.", "\n\nThe bus transport system has direct connection with the Granada Metro on 12 stations.", "\n\nReferences\n\nExternal links\nhttp://www.ideal.es/granada/201406/29/arranca-20140628231639.html\n\nhttp://www.ideal.es/granada/20100827/local/granada/trabajadores-rober-granada-iran-huelga-partir-de-septiembre-protesta-segregacion-lineas-201008271428.html\n\nhttps://elpais.com/ccaa/2012/08/19/andalucia/1345372288_487850.html\n\nhttps://elpais.com/ccaa/2012/03/04/andalucia/1330889547_581158.html\n\nhttp://www.ideal.es/granada/201411/01/linea-autobus-cementerio-20141031215230.html\n\nhttp://www.ideal.es/datalab/conexiones-transbordos-paradas-20170929165050-nt.html\n\nhttp://www.ideal.es/granada/pg060107/prensa/noticias/Local_Granada/200601/07/COS-GRA-078.html\n\nhttps://ecomovilidad.net/granada/cambios-en-el-transporte-publico-de-granada-nacen-las-lineas-sn3-sn4-y-sn5/\n\nhttp://www.transportesrober.com/transporte/tarifas.html\n\nhttps://web.archive.org/web/20110310043545/http://www.transportesrober.com/empresa/flota.html\n\nhttp://www.transportesrober.com/transporte/lineas.htm\n\nhttps://web.archive.org/web/20101013114255/http://www.transportesrober.com/empresa/rober.html\n\nCategory:Granada" ]
{ "pile_set_name": "Wikipedia (en)" }
[ 0.015463917525773196, 0.009433962264150943, 0.014354066985645933, 0, 0, 0, 0.022222222222222223, 0, 0.007575757575757576, 0.007518796992481203, 0, 0.014814814814814815, 0, 0.009615384615384616, 0.006329113924050633, 0, 0, 0.005747126436781609, 0.01, 0, 0.00909090909090909, 0.016042780748663103, 0, 0, 0.0031847133757961785, 0, 0, 0, 0, 0, 0, 0, 0, 0.010416666666666666, 0.08333333333333333, 0, 0.009009009009009009, 0, 0, 0, 0, 0, 0, 0, 0.011090573012939002 ]
0.005894
5
[ "The first example of a mixed valence ternary compound of silver with random distribution of Ag(I) and Ag(II) cations.", "\nThe reaction between colourless AgSbF6 and sky-blue Ag(SbF6)2 (molar ratio 2 : 1) in gaseous HF at 323 K yields green Ag3(SbF6)4, a new mixed-valence ternary fluoride of silver. ", "Unlike in all other Ag(I)/Ag(II) systems known to date, the Ag(+) and Ag(2+) cations are randomly distributed on a single 12b Wyckoff position at the 4̄ axis of the I4̄3d cell. ", "Each silver forms four short (4 × 2.316(7) Å) and four long (4 × 2.764(6) Å) contacts with the neighbouring fluorine atoms. ", "The valence bond sum analysis suggests that such coordination would correspond to a severely overbonded Ag(I) and strongly underbonded Ag(II). ", "Thorough inspection of thermal ellipsoids of the fluorine atoms closest to Ag centres reveals their unusual shape, indicating that silver atoms must in fact have different local coordination spheres; this is not immediately apparent from the crystal structure due to static disorder of fluorine atoms. ", "The Ag K-edge XANES analysis confirmed that the average oxidation state of silver is indeed close to +1⅓. The optical absorption spectra lack features typical of a metal thus pointing out to the semiconducting nature of Ag3(SbF6)4. ", "Ag3(SbF6)4 is magnetically diluted and paramagnetic (μ(eff) = 1.9 μ(B)) down to 20 K with a very weak temperature independent paramagnetism. ", "Below 20 K weak antiferromagnetism is observed (Θ = -4.1 K). ", "Replacement of Ag(I) with potassium gives K(I)2Ag(II)(SbF6)4 which is isostructural to Ag(I)2Ag(II)(SbF6)4. ", "Ag3(SbF6)4 is a genuine mixed-valence Ag(I)/Ag(II) compound, i.e. Robin and Day Class I system (localized valences), despite Ag(I) and Ag(II) adopting the same crystallographic position." ]
{ "pile_set_name": "PubMed Abstracts" }
[ 0, 0.0111731843575419, 0.005649717514124294, 0, 0, 0.0033112582781456954, 0, 0.0070921985815602835, 0, 0.009259259259259259, 0.010752688172043012 ]
0.004294
5
[ "Terminally functionalized polymers are useful precursors for the preparation of polymers containing functional end groups. ", "Examples of terminally functionalized polymers are vinylidene-terminated polyolefins. ", "Polymers containing functional end groups have several useful purposes. ", "For example, polyisobutylene (PIB) containing vinylidene chain ends are utilized in the synthesis of PIB-succinic anhydrides (PIBSAs), which are key intermediates in the production of PIB-based succinimide dispersants for use as additives for engine lubricants. ", "Vinylidene-terminated PIBs are also utilized in the production of PIB-amines, which are useful as fuel additives. ", "An opportunity exists to create polyolefins containing sulfide end groups for use in lubricant applications. ", "These thio-terminated polyolefins can offer oxidation inhibition and surface affinity, two important factors in the design of lubricant additive technology. ", "Thus, there is a need for new classes of terminally functionalized polymers, as well as methods of selectively or exclusively producing terminally functionalized polymers, such as vinylidene-terminate polyolefins and sulfide-terminated polyolefins." ]
{ "pile_set_name": "USPTO Backgrounds" }
[ 0, 0, 0, 0, 0, 0, 0, 0 ]
0
5
[ "The presence of inorganic carbon (as CO.sub.2, HCO.sub.3.sup.-, and CO.sub.3.sup.=) in samples to be anaIyzed for total organic carbon (TOC) creates two adverse affects: a positive interference with the CO.sub.2 detector and a decrease in the oxidation rate of organics. ", "It is desirable to remove inorganic carbon (IC) to allow direct TOC measurement of reclaimed water streams.", "\nInorganic carbon (IC) as dissolved CO.sub.2 causes positive interference with total organic carbon (TOC) measurements, and as carbonates, inorganic carbon adversely affects oxidation of organics. ", "Therefore, it is desirable to remove inorganic carbon (IC) to allow direct TOC measurement of reclaimed water streams. ", "The most sensitive standardized method of total carbon measurement, with a reported detection limit of 50 ug organic carbon/liter, which is described in STANDARD METHODS FOR THE EXAMINATION OF WATER AND WASTE WATER, 16th Ed. ", "as follows: Inorganic carbon, present in water as carbonate and/or bicarbonate ions above pH 4-5, is converted to CO.sub.2 by addition of aqueous acid (e.g. 10% phosphoric acid). ", "Carbon dioxide is then removed from the sample using an inert gas and carried to a non-dispersive infrared (NDIR) detector to measure CO.sub.2 (hence inorganic carbon concentration). ", "The sample is then subjected to UV-persulfate oxidation; the organic carbon is oxidized to CO.sub.2 and removed using purge gas. ", "The organic carbon concentration is measured as CO.sub.2 in the NDIR detector. ", "This method has significant disadvantages as applied to on-Iine monitoring of waste streams in microgravity. ", "These disadvantages include the requirement for sample acidification to convert carbonate and bicarbonate to CO.sub.2 using an expandable liquid reagent (phosphoric acid). ", "It is also considered disadvantageous to use expandable aqueous persulfate solutions in the Space Station Freedom (SSF) environment because of the weight limitations thereof. ", "Further, transfer of inorganic carbon as CO.sub.2 out of the sample using purge gas is a gravity-dependent process which, of course, can be non-functional in the microgravity of the SSF environment. ", "It is therefore considered desirable to provide a reliable gravity independent process for total organic carbon and total inorganic carbon measurement. ", "More specifically, it is considered desirable to provide a reagentless inorganic carbon separator which includes a solid-phase acid module integrated with a CO.sub.2 -permeable degasser. ", "It is also considered desirable to provide a passive total inorganic carbon (TIC) removal module containing a CO.sub.2 adsorbent. ", "The module should remove the TIC and fix it in the solid form in a single process step. ", "Even further, it is considered desirable to provide solid-phase TOC and TIC functional check modules to provide a means of verifying instrument performance without storage of unstable liquid reagents.", "\nThe water quality monitor (WQM) hardware presently being developed for the Space Station Freedom does not require persulfate reagents, but at present will require addition of liquid acid and uses centrifical gas/liquid separators. ", "It is considered desirable to provide an alternative to the use of liquid-phase acid reagents such as phosphoric acid and to provide a suitable alternative for the present requirement for purge gas such as nitrogen for removaI of dissolved CO.sub.2 from water. ", "Further, from the standpoint of use in orbital environments such as space stations and for other interpIanetary activities, it is considered desirable to provide a solid-phase system for removal of CO.sub.2 without using a purge gas. ", "It is also considered desirable to provide for conversion of carbonate and bicarbonate to CO.sub.2 without the addition of an expendable liquid reagent such as phosphoric acid. ", "This would permit elimination of corrosive liquid acids in the space station or space-craft environment." ]
{ "pile_set_name": "USPTO Backgrounds" }
[ 0.0036900369003690036, 0, 0, 0, 0.008888888888888889, 0, 0.00546448087431694, 0, 0.012658227848101266, 0, 0, 0, 0.005025125628140704, 0, 0, 0, 0, 0.005, 0.008620689655172414, 0, 0, 0, 0 ]
0.002146
5
[ "Q:\n\nНастройка формы\n\nМне нужно отправить форму с телефоном, НО, мне приходило пустое сообщение, а когда прописывал button с верхней формой, то все приходило. ", "Как видите, я пытался разделить две формы, чтобы структура кнопки и формы не пропала на сайте, но тогда не приходит сообщение с номером телефона. ", "Можно как объединить их, оставив такую структуру, как на картинке? ", "PHP код\n<?", "php\n$message = 'Телефон: '.$_POST['phone']; \n\nmail('почта', 'Хайф', $message, \"From: почта\");\n?", ">\n\n<div class=\"col-md-5 clearfix\">\n <div class=\"low_tel\">\n <form action=\"send.php\" method=\"post\"> \n <input id=\"low_tel\" name=\"tel\" type=\"text\" placeholder=\"Телефон\">\n </form>\n </div>\n</div>\n \n<div class=\"col-md-3 clearfix\">\n <div class=\"blue_btn\" id=\"form\">\n <form action=\"send.php\" method=\"POST\">\n <button id=\"zakaz_5min\" class=\"ya-link\" data-goal=\"form5min\" style=\"background: rgba(0, 0, 0, 0) url(../img/blue_btn.png) no-repeat scroll left center; border: none; font-size: 24px;\n color: #fff;\n text-transform: uppercase;\n font-family: 'officinaserifcregular'; \">Отправить заявку</button>\n </form>\n </div> \n</div>\n<script type=\"text/javascript\">\n$(\"#form\").submit(function (e) { // Устанавливаем событие отправки для формы с id=form\n e.preventDefault();\n var form_data = $(this).serialize(); // Собираем все данные из формы\n $.ajax({\n type: \"POST\", // Метод отправки\n url: \"send.php\", // Путь до php файла отправителя\n data: form_data,\n success: function () {\n // Код в этом блоке выполняется при успешной отправке сообщения\n alert(\"Ваше сообщение отправлено!\");", "\n }\n });\n });\n</script>\n</div>\n\nA:\n\nОбъедините формы в одну\nПриблизительно так\n<form action=\"send.php\" method=\"post\">\n <div class=\"row\">\n <div class=\"col-md-5 clearfix\">\n <div class=\"low_tel\">\n <input id=\"low_tel\" name=\"tel\" type=\"text\" placeholder=\"Телефон\">\n </div>\n </div> \n <div class=\"col-md-3 clearfix\">\n <div class=\"blue_btn\" id=\"form\">\n <button id=\"zakaz_5min\" class=\"ya-link\" data-goal=\"form5min\" style=\"background: rgba(0, 0, 0, 0) url(../img/blue_btn.png) no-repeat scroll left center; border: none; font-size: 24px;\n color: #fff;\n text-transform: uppercase;\n font-family: 'officinaserifcregular'; \">Отправить заявку</button>\n </div>\n </div>\n </div>\n</form>\n\n" ]
{ "pile_set_name": "StackExchange" }
[ 0.006329113924050633, 0, 0, 0.1, 0.010526315789473684, 0.00494641384995878, 0.0026954177897574125 ]
0.017785
5
[ "I would appreciate any donations. ", "Wishlist or send e-mail type donations to maekawa AT daemon-systems.org.", "\n\nThank you.", "\n\nPCQ(9) Kernel Developer's Manual PCQ(9)\nNAMEpcq - producer/consumer queue\nSYNOPSIS#include<sys/pcq.h>pcq_t*pcq_create(size_tmaxlen, km_flags_tkmflags);\nvoidpcq_destroy(pcq_t*pcq);\nvoid*pcq_get(pcq_t*pcq);\nsize_tpcq_maxitems(pcq_t*pcq);\nvoid*pcq_peek(pcq_t*pcq);\nboolpcq_put(pcq_t*pcq, void*item);\nDESCRIPTION\nThe machine-independent pcq interface provides lockless producer/consumer\nqueues. ", "A queue (pcq_t) allows multiple writers (producers), but only a\nsingle reader (consumer). ", "The consumer is expected to be protected by a\nlock that covers the structure that the pcq_t is embedded into (e.g.,\nsocket lock, ifnet hwlock). ", "These queues operate in a first-in, first-\nout (FIFO) manner. ", "The act of inserting or removing an item from a pcq_t\ndoes not modify the item in any way. ", "pcq does not prevent an item from\nbeing inserted multiple times into a single pcq_t.", "\nFUNCTIONSpcq_create(maxlen, kmflags)\nCreate a queue that can store at most maxlen items at one time.", "\nkmflags should be either KM_SLEEP, if pcq_create() is allowed to\nsleep until resources are available, or KM_NOSLEEP if it should\nreturn NULL immediately, if resources are unavailable.", "\npcq_destroy(pcq)\nFree the resources held by pcq.", "\npcq_get(pcq)\nRemove the next item to be consumed from the queue and return\nit. ", "If the queue is empty, return NULL. ", "The caller must\nprevent concurrent gets from occuring.", "\npcq_maxitems(pcq)\nReturn the maximum number of items that the queue can store at\nany one time.", "\npcq_peek(pcq)\nReturn the next item to be consumed from the queue but do not\nremove it from the queue. ", "If the queue is empty, return NULL.", "\npcq_put(pcq, item)\nPlace an item at the end of the queue. ", "If there is no room in\nthe queue for the item, return false; otherwise, return true.", "\nThe item must not have the value of NULL.", "\nCODEREFERENCES\nThe pcq interface is implemented within the file sys/kern/subr_pcq.c.", "\nSEEALSOatomic_ops(3), queue(3)HISTORY\nThe pcq interface first appeared in NetBSD 6.0.", "\nNetBSD 8.0 January 22, 2012 NetBSD 8.0" ]
{ "pile_set_name": "Pile-CC" }
[ 0, 0.013888888888888888, 0, 0.005089058524173028, 0, 0, 0, 0, 0, 0, 0.010869565217391304, 0, 0, 0.027777777777777776, 0, 0, 0, 0.02857142857142857, 0, 0, 0.023809523809523808, 0, 0.011627906976744186, 0 ]
0.005068
5
[ "ASADABAD, Afghanistan – The head of a presidential delegation investigating the deaths of 10 people in eastern Afghanistan concluded Wednesday that civilians – including schoolchildren – were killed in an attack involving foreign troops, but NATO officials disputed that.", "\n\nNATO spokesman Col. ", "Wayne Shanks said in a statement NATO has no direct evidence to substantiate the Afghan probe’s findings, and the international force has requested an immediate joint investigation to find out what happened.", "\n\nAsadullah Wafa, a senior adviser to President Hamid Karzai, gave conflicting numbers on the schoolchildren. ", "He told The Associated Press by telephone that eight schoolchildren between the ages of 12 and 14 were among the dead discovered in a village house in the Narang district of Kunar province. ", "But Reuters quoted him as saying that eight boys, ages 13 to 18, and two men in their 20s were killed.", "\n\nA NATO official had said that initial reports from troops involved in the fighting on Sunday indicated that those killed were insurgents — all young males.", "\n\nCivilian deaths are one of the most sensitive issues for foreign troops in Afghanistan, especially now when some additional 37,000 U.S. and NATO troops are being deployed to the war-ravaged country. ", "Although far more civilians are killed by the Taliban, those blamed on international forces spark widespread resentment and undermine the fight against militants.", "\n\nAfghans protested the deaths Wednesday in the eastern Afghan city of Jalalabad and in the capital of Kabul.", "\n\nIn the capital of Nangahar province, which borders Kunar, around 200 university students took to the streets to protest against the raid, demanding those responsible be brought to justice.", "\n\n“Death to Obama. ", "Down with Karzai,” they shouted.", "\n\nWafa said he was convinced all those killed in the Kunar incident were innocent civilians. …", "\n\nThe latest figures released by the United Nations show that 2,021 civilians died during clashes in the first 10 months of this year, up from 1,838 for the same period last year. ", "Taliban insurgents were blamed for 68 percent of the deaths this year — three times more than NATO forces, according to the U.N. …\n\nKABUL – The United Nations said Thursday that a weekend raid by foreign troops in a tense eastern Afghan province killed eight local students and that it warned against nighttime actions by coalition forces because they often cause civilian deaths.", "\n\nThe Afghan government said its investigation has established that all 10 people killed Sunday in a remote village in Kunar province were civilians. ", "Its officials said that eight of those killed were schoolchildren aged 12-14.", "\n\nNATO officials initially said all the dead were insurgents, but later backed off by saying there was no evidence to substantiate the claims that they were civilians. …", "\n\nThe coalition attack in Kunar has sparked protests by Afghans who have demanded that foreign troops leave the country.", "\n\n[UN special representative in Afghanistan Kai Eide] said the UN remained concerned about nighttime raids by coalition troops “given that they often result in lethal outcomes for civilians, the dangerous confusion that frequently arises when a family compound is invaded.” …", "\n\nA statement issued Thursday by the Afghan National Security Directorate said the government investigation showed no Afghan forces were involved and “international forces from an unknown address came to the area and without facing any armed resistance, put 10 youth in two rooms and killed them.", "\n\n“They conducted this operation on their own without informing any security or local authorities of Afghanistan,” the statement said.", "\n\nKABUL – The Taliban claimed responsibility Thursday for a suicide bombing at a base in eastern Afghanistan that killed eight American civilians and one Afghan, the worst loss of life for the U.S. in the country since October. ", "A U.S. congressional official said CIA employees are believed to be among the victims.", "\n\nFour Canadian soldiers and a journalist also were killed Wednesday by a roadside bomb in Afghanistan’s south, the bloodiest single incident suffered by that country’s military this year.", "\n\nTaliban spokesman Zabiullah Mujahid told The Associated Press that a Taliban bomber wearing a military uniform and a suicide vest entered the base in Khost province Wednesday evening and blew himself up inside the gym. ", "A U.S. official who was briefed on the blast also said it took place in the gym.", "\n\nThat official said eight U.S. civilians and one Afghan were killed; it was not clear if the Afghan victim was military or civilian. ", "Six Americans were wounded, the official said.", "\n\nThe CIA has not yet commented on or confirmed the deaths.", "\n\nA senior State Department official said all of the victims were civilians. ", "A former senior CIA officer who was stationed at the base said a combination of agency officers and contractors operated out of the remote outpost with the military and other agencies. ", "He said contractors also might be among those who died. …", "\n\nNATO said only that the base is used by provincial reconstruction teams, which consist of both soldiers and civilians, and other personnel.", "\n\nIn Kabul, a spokesman for the international coalition force in Kabul said no U.S. or NATO troops were killed in the afternoon explosion. ", "The attack was the bloodiest day for Americans since eight soldiers were killed in an insurgent attack on a base in eastern Afghanistan on Oct. 3.", "\n\nIn the south, NATO said that the four Canadian troops and a reporter embedded with their unit died when their armored vehicle hit a bomb while on an afternoon patrol south of Kandahar city. ", "It’s the third deadliest day for Canadians in Afghanistan since the war began.", "\n\nMichelle Lang, a 34-year-old health reporter with the Calgary Herald, was the first Canadian journalist to die in Afghanistan. ", "Lang arrived in Afghanistan just two weeks ago. …", "\n\nBrig. ", "Gen. Daniel Menard, commander of coalition forces in Kandahar, said that the soldiers were conducting a community security patrol in order to gather information about daily life in the area and how to maintain security.", "\n\nWednesday was the second lethal strike against the Canadian force in a week. ", "One Canadian soldier and an Afghan soldier were killed Dec. 23 during a foot patrol in Panjwayi district of Kandahar province. ", "According to figures compiled by The Associated Press, the latest casualties bring to 32 the number of Canadian forces killed in Afghanistan this year; in all, 138 have died in the war. …", "\n\nSuicide attack takes heavy toll on CIA(NBC Nightly News, Dec. 31, 2009)– The CIA is seeking answers after seven officials were killed Thursday in Afghanistan in what was one of the deadliest single attacks on U.S. intelligence personnel. ", "NBC’s Jim Maceda reports. (", "02:30)\n\nThe Associated Press and Reuters, via MSNBC.comDec. ", "31, 2009\n\nKABUL – The suicide bomber who killed seven CIA employees at a remote outpost in southeastern Afghanistan had been invited onto the base and had not been searched, two former U.S. officials told The Associated Press on Thursday\n\nA former senior intelligence official says the man was being courted as an informant and that it was the first time he had been brought inside the camp. ", "An experienced CIA debriefer came from Kabul for the meeting, suggesting that the purpose was to gain intelligence, the official said.", "\n\nThe former intelligence official and another former official with knowledge of the attack spoke on condition of anonymity because they were not authorized to speak publicly. …", "\n\nBlow to tight-knit spy agency\n\nThe bombing on Wednesday dealt a blow to the tight-knit spy agency. ", "Among those killed was the chief of the CIA post, whom former officials identified as a mother of three. ", "Six more agency personnel were wounded in what was considered the most lethal attack for the CIA since the war in Afghanistan began in 2001 and possibly even since the 1983 embassy bombing in Beirut. …", "\n\nThe CIA did not release information about the victims, citing the sensitivity of their mission and other ongoing operations. ", "Harold E. Brown Jr., a State Department employee of Fairfax, Va., died in the attack, his father, Harold E. Brown Sr., ", "told The Associated Press on Thursday.", "\n\nThe younger Brown, 37, who grew up in Bolton, Mass., served in the Army and remained a major in the reserves. ", "He is survived by a wife and three children ages 12, 10 and 2. …", "\n\nKABUL – A suicide bomber killed seven people at a busy bazaar in eastern Afghanistan on Thursday, and a bomb hidden in a garbage container outside a provincial governor’s compound slightly wounded the official, authorities said.", "\n\nThe attacks were in Paktia and Khost provinces, both of which border Pakistan and suffer frequent violence as insurgents gain momentum in their fight against Afghan and international troops.", "\n\nIn Gardez, the capital of Paktia province to the south, a suicide bomber on foot blew himself up in a bazaar near a six-vehicle convoy of security workers, said Deputy Gov. Abdul Rahman Mangal. ", "He said seven people were killed, including the commander of Afghan security guards at a base for a provincial reconstruction team in Logar province. ", "Another 24 were wounded, he said. …", "\n\nKhost is one of Afghanistan’s most troubled provinces. ", "Seven CIA employees were killed Dec. 30 when a suicide bomber attacked an agency base there. ", "On Wednesday, at least 15 people were wounded in a blast outside a shop in Khost city, the provincial center.", "\n\nRockets hit Kabul\n\nEarlier Thursday, three rockets were fired into a residential area of Kabul, wounding three civilians. ", "The Interior Ministry said in a statement that two of the rockets hit a house and the third landed in a garden. ", "Local police said the attack was in the Qalafa neighborhood, about three miles southeast of central Kabul. …", "\n\nIn the eastern city of Jalalabad, about 5,000 demonstrators gathered to protest the Wednesday deaths of children in an explosion that they blamed on U.S. forces. ", "Local officials said four children were killed, but NATO said Thursday that two died and that international troops were not responsible.", "\n\nThe blast about 15 miles south of Jalalabad, tore through a group of soldiers and civilians while the soldiers were visiting a road-construction project. ", "Some 80 civilians and three soldiers were wounded.", "\n\nAfghan police said the blast was caused by a passing police vehicle hitting a mine, but the protesters blamed it on the U.S. soldiers. ", "They shouted “Death to America” and burned an effigy of President Barack Obama. …", "\n\nWounded police officers are treated at a hospital after a bomb attack in Ramadi, Iraq, on Wednesday, Dec. 30, 2009. (", "Photo credit: Reuters)\n\nDec. 30, 2009\n\nBAGHDAD, Iraq – Staggered explosions Wednesday killed 23 people – 13 of them policemen – and wounded an Iraqi provincial governor, officials said, in the worst violence in months to hit the western province that was formerly al-Qaida’s top stronghold in Iraq. …", "\n\nThe attacks Wednesday were worrisome because the strategically important Anbar province was once the heartland of support for al-Qaida-linked militants, before many insurgents turned on the terror organization and joined forces with U.S. troops and the Iraqi government. ", "The governor is the most senior Sunni leader to be attacked since then.", "\n\nCheckpoint\n\nPolice official Lt. ", "Col. ", "Imad al-Fahdawi said two bombs exploded in Anbar’s capital of Ramadi, 70 miles west of Baghdad. ", "He says a suicide bomber in a car caused the first blast near a checkpoint on the main road near the provincial administration buildings.", "\n\nGov. Qassim al-Fahdawi, the deputy police chief and other officials came to inspect the damage, the police official said, when a suicide bomber on foot detonated a vest full of explosives nearby.", "\n\nThe deputy police chief was killed and the governor and other officials wounded, al-Fahdawi said. ", "Police have put a curfew in place, he added. …", "\n\nForensic investigations\n\nA doctor at the main hospital in Ramadi, Ahmed Abid Mohammed, said 23 people had been killed and 57 injured. ", "He said the governor had suffered burns on his face and injuries to his abdomen and other areas.", "\n\nIn the town of Khalis, about 50 miles northeast of Baghdad, a bomb killed six pilgrims taking part in a procession to commemorate the death of a Shiite revered saint, said a Diyala province police spokesman, Capt. ", "Ghalib al-Karkhi. ", "He said the blast also wounded 24 people.", "\n\nBAGHDAD – A series of blasts killed six people in Iraq’s western province of Anbar on Thursday, a police official said, in the latest attack to hit the province that was once the heartland of the al-Qaida-led insurgency.", "\n\nCol. ", "Fadhil Nimrawi said one explosion targeted a house belonging to Lt. ", "Colonel Walid Sulaiman al-Hiti, the director of the anti-terrorism unit in the town of Hit, about 85 miles west of Baghdad. ", "A second explosion targeted the home of his father next door. ", "Slaiman was wounded and his mother, two sisters, another family member and a child were killed.", "\n\nNimrawi said a lawyer was killed by another bomb at his home and a fourth bomb exploded at the home of a police officer, injuring him as he slept.", "\n\nSeven people were injured in the bombings, Nimrawi said. …", "\n\nThursday’s blasts follow last week’s attack on Ramadi, the capital of Anbar province, which wounded the Anbar governor and killed 24. ", "The Islamic state of Iraq, an umbrella group for al-Qaida affiliated insurgent groups, claimed responsibility for that attack.", "\n\nIn a separate incident in northeastern Iraq’s Diyala province, a bomb exploded near a police station, killing one policeman and injuring ten other people near the Iranian border, police and hospital officials said. …", "\n\nOne-year retrospective: One year ago today, I reported that Michele Bachmann, Minnesota’s 6th District representative in Congress, made a strong showing in several “Year in Review” lists – no easy feat, considering the U.S. House of Representatives has 435 members, each vying for media attention." ]
{ "pile_set_name": "Pile-CC" }
[ 0.007380073800738007, 0.045454545454545456, 0.00966183574879227, 0.00909090909090909, 0.005263157894736842, 0.00980392156862745, 0.006369426751592357, 0.004975124378109453, 0.006172839506172839, 0, 0.005263157894736842, 0, 0, 0.010638297872340425, 0.005555555555555556, 0.013157894736842105, 0, 0, 0.005917159763313609, 0, 0.007272727272727273, 0.0033783783783783786, 0, 0.008771929824561403, 0.011627906976744186, 0, 0.01809954751131222, 0, 0, 0, 0.01694915254237288, 0.012987012987012988, 0.005405405405405406, 0, 0.0070921985815602835, 0.007194244604316547, 0, 0.005208333333333333, 0, 0.015503875968992248, 0.02040816326530612, 0, 0.0045662100456621, 0, 0, 0.0053475935828877, 0.008333333333333333, 0.07407407407407407, 0.03333333333333333, 0.00510204081632653, 0.007462686567164179, 0, 0, 0.009523809523809525, 0.004975124378109453, 0.007874015748031496, 0.025210084033613446, 0.02631578947368421, 0.017857142857142856, 0, 0.004347826086956522, 0, 0.01020408163265306, 0, 0, 0, 0.010752688172043012, 0, 0, 0.008928571428571428, 0.009259259259259259, 0, 0.007352941176470588, 0, 0, 0, 0.012345679012345678, 0, 0.006666666666666667, 0.003663003663003663, 0, 0, 0, 0.010416666666666666, 0, 0.005076142131979695, 0.01, 0, 0.007352941176470588, 0, 0.004629629629629629, 0.05555555555555555, 0, 0.0045045045045045045, 0, 0.014705882352941176, 0.008064516129032258, 0, 0.010526315789473684, 0.006756756756756757, 0.016666666666666666, 0, 0.007936507936507936, 0, 0.013377926421404682 ]
0.007273
5
[ "Oculomotor paralysis: 3D-CISS MR imaging with MPR in the evaluation of neuralgic manifestation and the adjacent structures.", "\nTo evaluate the value of three-dimensional (3D) constructive interference in steady-state (CISS) magnetic resonance (MR) imaging with multi-planar reconstruction (MPR) in displaying the relationship between the oculomotor nerve and its adjacent structures for patients with oculomotor paralysis. ", "17 consecutive patients with oculomotor paralysis were examined with 3D-CISS and conventional spin-echo (SE) sequences on a 1.5-Tesla MR system. ", "Original transverse and MPR images were used for image interpretation. ", "The features of the oculomotor nerve and its adjacent structures were identified. ", "The diagnosis was surgically confirmed in all patients. ", "Through 3D-CISS with MPR images, obvious relationship of the oculomotor nerve and its adjacent structures was demonstrated on 17 patients. ", "Of those oculomotor nerves, 15 were compressed by the arteries (n=15), one by the craniopharyngioma (n=1), and another one by the neurofibroma (n=1). ", "3D-CISS MR imaging with MPR provides an excellent way to characterize the relationship between the nerve and its adjacent structures in the cisternal segment of the oculomotor nerve in the patients with oculomotor paralysis. ", "Moreover, this method shows anatomical details for imaging diagnosis and surgical procedure." ]
{ "pile_set_name": "PubMed Abstracts" }
[ 0.008130081300813009, 0.003367003367003367, 0, 0.014084507042253521, 0, 0, 0.007194244604316547, 0, 0.0044444444444444444, 0 ]
0.003722
5
[ "# Copyright (c) 2006-2010 Mitch Garnaat http://garnaat.org/\n# Copyright (c) 2010, Eucalyptus Systems, Inc.\n# All rights reserved.", "\n#\n# Permission is hereby granted, free of charge, to any person obtaining a\n# copy of this software and associated documentation files (the\n# \"Software\"), to deal in the Software without restriction, including\n# without limitation the rights to use, copy, modify, merge, publish, dis-\n# tribute, sublicense, and/or sell copies of the Software, and to permit\n# persons to whom the Software is furnished to do so, subject to the fol-\n# lowing conditions:\n#\n# The above copyright notice and this permission notice shall be included\n# in all copies or substantial portions of the Software.", "\n#\n# THE SOFTWARE IS PROVIDED \"AS IS\", WITHOUT WARRANTY OF ANY KIND, EXPRESS\n# OR IMPLIED, INCLUDING BUT NOT LIMITED TO THE WARRANTIES OF MERCHANTABIL-\n# ITY, FITNESS FOR A PARTICULAR PURPOSE AND NONINFRINGEMENT. ", "IN NO EVENT\n# SHALL THE AUTHOR BE LIABLE FOR ANY CLAIM, DAMAGES OR OTHER LIABILITY,\n# WHETHER IN AN ACTION OF CONTRACT, TORT OR OTHERWISE, ARISING FROM,\n# OUT OF OR IN CONNECTION WITH THE SOFTWARE OR THE USE OR OTHER DEALINGS\n# IN THE SOFTWARE.", "\n#\n\nfrom boto.regioninfo import RegionInfo\n\nclass RDSRegionInfo(RegionInfo):\n\n def __init__(self, connection=None, name=None, endpoint=None,\n connection_cls=None):\n from boto.rds import RDSConnection\n super(RDSRegionInfo, self).__init__(connection, name, endpoint,\n RDSConnection)\n" ]
{ "pile_set_name": "Github" }
[ 0.023255813953488372, 0.006825938566552901, 0.014084507042253521, 0.004098360655737705, 0.01466275659824047 ]
0.012585
5
[ " UNPUBLISHED\n\n UNITED STATES COURT OF APPEALS\n FOR THE FOURTH CIRCUIT\n\n\n No. ", "07-4428\n\n\n\nUNITED STATES OF AMERICA,\n\n Plaintiff - Appellee,\n\n versus\n\n\nJOE LEE NEWCOMB,\n\n Defendant - Appellant.", "\n\n\n\nAppeal from the United States District Court for the Middle\nDistrict of North Carolina, at Durham. ", "William L. Osteen, Senior\nDistrict Judge. (", "1:06-cr-00421-WLO)\n\n\nSubmitted: September 24, 2007 Decided: October 9, 2007\n\n\nBefore TRAXLER, KING, and DUNCAN, Circuit Judges.", "\n\n\nAffirmed by unpublished per curiam opinion.", "\n\n\nLouis C. Allen III, Federal Public Defender, William S. Trivette,\nAssistant Federal Public Defender, Greensboro, North Carolina, for\nAppellant. ", "Anna Mills Wagoner, United States Attorney, Michael\nAugustus DeFranco, Assistant United States Attorney, Greensboro,\nNorth Carolina, for Appellee.", "\n\n\nUnpublished opinions are not binding precedent in this circuit.", "\n\fPER CURIAM:\n\n Joe Lee Newcomb pled guilty to being a felon in\n\npossession of a firearm, in violation of 18 U.S.C. §§ 922(g)(1) and\n\n924(e) (2000). ", "The district court sentenced Newcomb to 180 months’\n\nincarceration, five years supervised release, and assessed a $100\n\nfine. ", " On appeal, Newcomb’s counsel filed a brief pursuant to\n\nAnders v. California, 386 U.S. 738 (1967), stating there were no\n\nmeritorious issues for appeal, but raising for review the issue of\n\nwhether the district court erred by not sentencing Newcomb to less\n\nthan 180 months’ incarceration. ", " Newcomb was advised of the right\n\nto file a pro se supplemental brief, but has not done so.", "\n\n After United States v. Booker, 543 U.S. 220 (2005), a\n\ndistrict court is no longer bound by the range prescribed by the\n\nsentencing guidelines. ", " However, Booker “did nothing to alter the\n\nrule that judges cannot depart below a statutorily provided minimum\n\nsentence” except upon the Government’s motion on the basis of\n\nsubstantial assistance. ", " United States v. Robinson, 404 F.3d 850,\n\n862 (4th Cir.), ", " cert. ", " denied, 546 U.S. 916 (2005). ", " For a\n\ndefendant, like Newcomb, who is found guilty of being a convicted\n\nfelon in possession of a firearm in violation of 18 U.S.C.\n\n§§ 922(g)(1), 924(e), the minimum term of incarceration required to\n\nbe imposed is 180 months. ", " The district court sentenced Newcomb to\n\nthe mandatory minimum penalty. ", "The district court did not have the\n\n\n\n\n - 2 -\n\flegal authority to impose any less time, and thus did not err by\n\ndeclining to do so.", "\n\n In accordance with Anders, we have reviewed the entire\n\nrecord in this case and have found no meritorious issues for\n\nappeal. ", " We therefore affirm Newcomb’s conviction and sentence.", "\n\nThis court requires that counsel inform Newcomb, in writing, of the\n\nright to petition the Supreme Court of the United States for\n\nfurther review. ", " If Newcomb requests that a petition be filed, but\n\ncounsel believes that such a petition would be frivolous, then\n\ncounsel may move in this court for leave to withdraw from\n\nrepresentation. ", " Counsel’s motion must state that a copy thereof\n\nwas served on Newcomb.", "\n\n We dispense with oral argument because the facts and\n\nlegal contentions are adequately presented in the materials before\n\nthe court and argument would not aid the decisional process.", "\n\n\n\n AFFIRMED\n\n\n\n\n - 3 -\n\f" ]
{ "pile_set_name": "FreeLaw" }
[ 0.017543859649122806, 0.014150943396226415, 0, 0.023255813953488372, 0.014184397163120567, 0, 0.027210884353741496, 0.02054794520547945, 0, 0.010471204188481676, 0.007936507936507936, 0.006825938566552901, 0.009615384615384616, 0, 0, 0.014084507042253521, 0, 0, 0.003703703703703704, 0.012345679012345678, 0, 0, 0.017857142857142856, 0.013422818791946308, 0.004672897196261682, 0.013513513513513514, 0, 0 ]
0.008262
5
[ "Autor\nThema: Imladris Balance Suggestions (Gelesen 8243 mal)\n\nDunedain aren't eite units, they are regular units. ", "Every elven unit is defined as elite.", "\n\nZitat\n\nI actually agree with most of what you said, Imladris heroes aren't very useful or have as clear defined roles (compared to other factions)\n\nOf course they have defined roles, I cant follow you. ", "Arwen is a supporter, Glorfindel tank, Twins heroslayer (one of the best in game), Gildor is a scout, Halbarad is a dunedain supporter and Elrond is a mix between massslayer and tank -> as suggested by our fans.", "\n\nZitat\n\nSo when Elrond is already one of the tankier heroes in the game with his armour boost he still dies too quick for your liking? ", "I just don't get that, he is supposed to be a mass slayer after all. ", "Being more or less vulnerable to hero killers is basically part of the job description. ", "And units shouldn't catch him anyway, he's got a horse and his flood does knockback...\n\n+1\n\nZitat\n\nI personally don't see anything that negative, know-it-all, slightly provocative or harsh in Elendil's response\n\n@Draco: Imladris only has one batallion at the start to stop you from just running over to your enemy and pooping on everything he has - we tried several things during the beta phase and this solution works out really well, I think. ", "I have to say though, I don't think Imladris absolutely needs an outpost early on, I've done quite well without one^^The amount of archers defending the outpost just make it way too strong defensively and too good overall, so it's never a bad choice really, since it also produces money and can heal.", "\n\nAs for my above post, that was just me explaining why Sawman got a little heated, because from my perspective while reading Elendil's first reply there were a couple points that seemed off. ", "The biggest one being the \"Maybe pay a bit more attention \" with the inclusion of the winky face sounds very sassy or \"know-it-allish\" as I said.", "\n\nThat certainly wasn't my intention, I put the wink there for that reason: to make it sound not as harsh.", "\n\nOf course they have defined roles, I cant follow you. ", "Arwen is a supporter, Glorfindel tank, Twins heroslayer (one of the best in game), Gildor is a scout, Halbarad is a dunedain supporter and Elrond is a mix between massslayer and tank -> as suggested by our fans.", "\n\nI'm a huge fan too believe me! ", "I actually mean they don't serve as important a role within the faction when compared to others. ", "They're just not as useful for their cost especially when you have tons of upgrades to buy and so they get overlooked. ", "Maybe that is why I feel that way.", "\n\nI'd rather buy 5 Blademasters than Elrond for e.g and yes I do still think he dies far too quickly! ", "His abilities are good though, though I miss the hero lvl up thing lol\n\nGildor < Merry/Pippin/Shagrat/Drauglin/Lorien Twins, his abilities are good but he isn't very durable, he gets easily owned by goblin creeps unlike the other scouts that can hold their own.", "\n\nArwen needs a better last ability, as has been already stated. ", "She is only useful for her heal as it stands.", "\n\nGlorfindel is the only hero I make, usually just to take out enemy siege.", "\n\nForgive me for saying this, but as an English speaker (and a psychiatrist in training!) ", "I agree with Elite KryPtik. ", "Every time I say something here I feel like I'm attacked or pounced upon. ", "Isn't the whole point of these forums to get feedback? ", "The response is usually so weird that you think twice about even carrying on the conversation (and I'm not talking about Elendil). ", "Just the tone is a little stand-offish sometimes. ", "We all love the mod and are trying to be constructive here.", "\n\nMost Germans are quite direct usually, they don't beat around the bush. ", "Just keep in mind that the tone of debate employed by the majority of the team and the German users is not intended as denigrating in any form, even if you feel it does. ", "No hard feelings, right?", "\n\nA debate tends to get people excited more often than not, so do not fear the pouncing nor the assaults, you must never relent !", "\n\nGespeichert\n\n“For so sworn good or evil an oath may not be broken and it shall pursue oathkeeper and oathbreaker to the world's end.”", "\n\nNo, it's not just that. ", "Getting or giving feedback is a part of a more comprehensive sum of many other factors. ", "Not only does the forum ensure people to express themselves, but it also encourages them to debate among each other and to exchange opinions (particularly, when it comes to finding a compromise out of different proposals or polishing existing concepts). ", "In another perspective, these features can also lead to heated discussions or little 'skirmishes' among the very users (something that one could easily see as a harsh and very confrontational climate throughout some threads).", "\n\nI believe that the latter aspect is eventually as fundamental as the other ones indeed, in order to keep Modding Union alive; the challenge is rather how to control it and turn it into constructive activity, lest everything degenerate in open clashes. ", "Furthermore, instead of weird or bitter replies, I sincerely think that what often causes resentment is the fact that some people find it hard to accept that their suggestions/remarks could be countered by other people and also proven inappropriate for the game (very direct answers that are usually triggered by not so consistent posts, of a similar kind of the initial comment that started this debate). ", "I'm not referring to you specifically, but to the general experience I have had so far as a Moderator.", "\n\nI'm well aware that this might probably be one of the most difficult characteristics to get acquainted with, but it's nonetheless vital to the forum as well. ", "On a higher level, Administrators and Moderators have in fact the authority to censure people and to reject proposals whenever they deem it necessary (that is, closing topics or delete comments). ", "One can't thus expect every single post to be accepted anyway.", "\n\nAs Odysseus suggested, you shouldn't let criticism (when it's reasonable) constrain your will of participating in debates: getting used to presenting ideas, defending them and wisely replying to other people's responses will toughen you up concretely and it will certainly give you overall much more confidence in regards of your personal forum attendance. ", "Keep also in mind that insults and a manifestly aggressive behaviour are against the rules and so will be dealt with accordingly\n\nMost Germans are quite direct usually, they don't beat around the bush.", "\n\nThe English Community has had too its own renowned record in being brutally honest at times and setting up very 'dynamic' discussions\n\nAlso, speaking about brutal (but needed) attitudes, I have to remind all of you that we are now clearly diverting too much from the topic of this thread. ", "If anyone is willing to continue to discuss this theme, the Website Feedback board is the proper place (and I would be very pleased to join it too).", "\n\nSo having played with / against Imladris quite a few times now I have to say the faction overall seems great in terms of balance baring a few issues here and there. ", "The main one though I believe is Gildor.", "\n\nThe problem I find with Gildor is that as he is the weakest scout hero (in terms of combat) in the entire game right now being as he's the only exclusively ranged scout hero at present (barring Rumil however Orophin makes up for that). ", "I think he should get a damage buff or some form of melee attack (perhaps he uses a melee weapon when he's locked in close combat as opposed to a direct method of melee similar to Denethor).", "\n\nThe main issue he also has when compared to the other scouts is that because he is in fact ranged unlike the others, who can gain access to their abilities quite quickly and become a combat force since they can creep alongside the starter units, and in all honesty, I find his abilities (barring his global reveal) to be well and truly underwhelming.", "\n\nHis Song loses a lot of it's potency because he will always be behind your force, meaning the slow down effect isn't going to do much unless enemies have pushed through to Gildor himself (though it is good for getting him out alive).", "\n\nHis arrow comes far too late (level 7, which as I outlined above is problematic when you take into account that Rivendell needs a lot more work than the other factions to even get it's ranged units out with a total cost of 2000, excluding Gildor's own cost and then a further 600 to even give Gildor some leveling support from the backline unless you manage to rush the dunedain, and even that still requires a minimum investment of 1400).", "\n\nIn comparison to the other scout heroes, Gildor tends to gain 1 level to their 2-4 from personal experience.", "\n\nThere's just one other thing I feel like I should address even though it's not a major issue, nor is it really required, but I personally feel that the cost of Rivendell's Spearmen should be decreased to 500, perhaps at a slight decrease to their health as in this method it's at least viable to get a unit of them out on a 1k start to assist with creeping, while allowing an external farm to be created as well, since while most factions tend to focus their scout heroes in the early game, for the reasons outlined above, 99% of the time I don't find Gildor to even be worth recruiting until you have a level 2 barracks, and at that point his purpose is more or less gone.", "\n\nThe problem I find with Gildor is that as he is the weakest scout hero (in terms of combat) in the entire game right now being as he's the only exclusively ranged scout hero at present (barring Rumil however Orophin makes up for that). ", "I think he should get a damage buff or some form of melee attack (perhaps he uses a melee weapon when he's locked in close combat as opposed to a direct method of melee similar to Denethor).", "\n\nThe main issue he also has when compared to the other scouts is that because he is in fact ranged unlike the others, who can gain access to their abilities quite quickly and become a combat force since they can creep alongside the starter units, and in all honesty, I find his abilities (barring his global reveal) to be well and truly underwhelming.", "\n\nHis Song loses a lot of it's potency because he will always be behind your force, meaning the slow down effect isn't going to do much unless enemies have pushed through to Gildor himself (though it is good for getting him out alive).", "\n\nHim being ranged is awesome for Imladris. ", "He can kill trolls while your starting unit gets free settlements and/or kills an easy creep like orcs before destroying the troll creep. ", "The other two points I don't quite get tbh... did you ever try to, you know, push him up alongside your units? ", "Because that solves your entire problem^^\n\nHe may be the weakest scout hero in terms of fighting, but he has the best scouting ability in the game and is arguably the best at supporting your units, too.", "\n\nImladris walls and gate seem to be very weak on their fortress compared to everyone else's walls, it take only 3 Gondor trebuchet shots to take down one segment of the wall where as other factions take a lot more, not only that by the time you take down one segment the other segments around it are already at half health because of the massive splash damage they take.", "\n\nI hope Loremasters are nerfed in the next version, because they are too OP right now. ", "I was playing a 4-Factions free for all, and the only factions left were my Imladris and another one. ", "I had an 1000 command point fully upgraded army and I was destroying their castle, when Lore Masters happened, and I lost my whole army, just like that. ", "Both Loremasters and Towers are too powerful right now. ", "It's strange to see your enemy destroy your whole army and kill your Heroes just with Loremasters, catapults and towers.", "\n\nIt's not that strange if you blob up your troops, which I presume is what happened. ", "In almost all games I have ever watched, even the highest level in the olden BFME I and BFME II tournament days, even there, blobbing in some way occurs.", "\n\nThat said, Loremasters will probably be reworked in the future, as Ea mentioned some time ago. ", "They are indeed overperforming.", "\n\nGespeichert\n\n“For so sworn good or evil an oath may not be broken and it shall pursue oathkeeper and oathbreaker to the world's end.”", "\n\nBlob up, like sending them one by one? ", "Sorry if I'm wrong, I didn't know the meaning!", "\n\nI sent most of them together, while a few were destroying an outpost. ", "Those were the ones that survived at first, but the majority of the army got destroyed. ", "I didn't have Arwen to support (I don't usually recruit her), so maybe that was one of the problems.", "\n\nI'm happy to hear about Loremasters being probably reworked! ", "I like using them, but it gets boring because they are so overpowered.", "\n\nwhen a good player of Imladris start to spam swords since the beginning of the game don't matter the factions you play, you can´t fight your main force against his main force, and im cool with that, you dont need to fight all the time you can arras him and get more map control and then when you get more money igual the forces and win a fight... but the problem is when Imldris rush they libray and get the ability of speed, this added to the wind lord master ability make them insane fast, even more than cavalry and you cant run from them forcing you to lose the fight and probably the game (Consider that I omited all the slowdowns and stuns that habe the faction).", "\n\nalso i would like to add that the library is a little broken because when the library have all the upgrades give you 75 resources and in late game if you replacements all the farms for libraries it is insane the amount of money that you can get!", "\n\nImladris swords spam isn't hard to counter considering that all the other player needs to do is spam archers to counter it.", "\n\nImladris really struggles against factions who can get archers off the start from a barracks(angmar,mordor,And especially lothlorien) and then the rest of the factions it is fairly balanced between them. ", "I think allowing Imladris to purchase archers off the start might fix this problem because chances are that your archers could counter or balance out the others archers.", "\n\nAnd as for the library 75 I think is the perfect number for the library at level 3 because you have to take into consideration that the total to get it level 3 is 3000 resources and you could spam them after that but I feel like Imladris needs that because they really don't have a good way to boost their eco\n\nAnother problem I have with imladris is their spell book, their tier 1 and 2 spell powers are good but the ones I have problems with are Tom,flood,and last alliance\n\nTom's problem is that he dies way to fast, don't get me wrong his stats are great but an increase in armour would probably fix this.", "\n\nFloods problem is that it's only good on army of the dead and orcs, I think the ET tried to buff it but it wasn't enough\n\nThe last alliances problem is that it doesn't last long enough and that all the until are terrible late game and all the hero's except isldur are also terrible. ", "Increasing the timer and adding more units to the summon and making the hero's better would make this power more viable.", "\n\nHello! ", "Thank you for your titanic work, for the wonderful mod! ", "I play The Battle for Middle-earth on the network since 2006. ", "Your mod has become for me a practical bfme-3... The only mod, which have serious balance.", "\n\nSince the release of 4.4, I play only for Imladris, trying to learn how to play, against any faction.", "I play at a fairly good level, recently at the tournament came in second place of the 16 most active players in gameranger.", "\n\nImladris well balanced against Isengard, Angmar, Gondor.", "\n\nLorien:\n\nImladris at the start has only one detachment of swordsmen, you can quickly purchase a second ..However, Lorien may have at the start of so many groups that force swordsmen Imladris lose its value: one unit Lorien runs, and the second attack from the back, archers shoot ...But, at the beginning of the game, perhaps one troop swordsmen to kill the troll, received the money, do one cavalry troop.", "\n\nOkay, so, with a good micro control, in the early game Imladris can withstand Lorien...However, if Lorien builds an outpost, it is practically impossible to fight!", "\n\nExclusive spearmen, minstrels, Galadriel capture, the capture of the palantir roots, capture arrows of Haldir (recovers a very fast), the capture of Radagast. ", "Because of this, the cavalry can not neutralize the archers.", "Maybe all this is justified for Lorien, towards to other factions, but very expensive units of Imladris...\n\nImladris can not lose a lot of units, it is fatal for him.", "\n\nLorien has large and inexpensive units of outpost, star arrows of very quickly destroy costly troops of Imladris. ", "Loremasters not solve the problem: they are just as scared of arrows of Haldir, stop of Galadriel, skill of palantir.", "\n\nEred Luin:\n\nCatastrophe! ", "At the start of the game, the dwarves have the speed, have many units, cheaper, strong enough, the crows of the Palantir will help them to easily kill swordsmen of Imladris, and you can not escape! (", "soldiers of Imladris do not have the speed at the beginning of the game.", "Of course, you can start in the cavalry for 1600 (800 + 800 stable building unit cost) but you yourselves understand ...Well, you do not got a very strong opponent, and you miraculously survived in the early game. ", "But there is Torin ... he is able to destroy many swordsmen of Imladris, even with improved armor and blades of ErigionOf course, Torin will not fight alone, he will have the support units, less expensive than in of Imladris.", "\n\nBut the main problem at the start of the game, rather than Thorin.", "\n\nIron Hills:Perhaps, but extremely difficult.", "In the melee compete with these guys is impossible, and archers of Imladris unavailable, second level of the barracks, only 5 (!) ", "soldiers in troop and take the limit of 120 (!). ", "Arrows of Mitlond good, but the cost of the three units + Tower - 4300. ", "And these three groups is not enough in the late game. ", "Archers Dúnedain not solve the problem, they are built too long to get the fiery darts too hard, goats destroy them very rapidly)Well, fire catapults destroying the archers quickly and catapults of Imladris!And, the soldiers of iron hills can get together in a pile around the catapults + iron hills are resistant to the consternation (loremasters not work)\n\nMordor:In the late game Imladris good job, but at the start of Imladris has not masters of the blade, so the free orcs surround and kill swordsmen (by means of poison and Gorbag eye of Sauron) are free, numerous groups of orcs sent to many farms destroy them, and Imladris can not have a sufficient number of troops to protect them.", "\n\nRohan:Available cavalry, cheap peasants ... Imladris lacks pikemen units (cost of unit 600), a lot of farmers groups establish control of the map, destroying farms, and the horsemen of Rohan already at the start suppresses infantry of Imladris. ", "All this is happening even before the possible hiring Dúnedain,because Rohan may employ its forces immediately, Imladris need outpost (800 + cost of grade for the possibility of hiring troops) Money + time.", "\n\nIn conclusion, I want to say that all this applies to games between equally strong players. ", "Of course, you can win the race strong - weak. ", "But we want to have equal chances. ", "Imladris newest faction and needs correcting, she is beautiful, all the heroes, the warriors are exceptionally well designed and made in style, I was surprised - this is the only race in which I love every unit and I'm really looking forward to when Imladris can fight all factions on equal chances!" ]
{ "pile_set_name": "Pile-CC" }
[ 0, 0, 0, 0.018957345971563982, 0.007352941176470588, 0, 0, 0.0022471910112359553, 0, 0, 0, 0, 0, 0.018957345971563982, 0, 0, 0, 0, 0, 0.0038314176245210726, 0, 0, 0, 0, 0.03571428571428571, 0, 0, 0, 0, 0, 0, 0, 0, 0, 0.007407407407407408, 0, 0, 0, 0, 0.003937007874015748, 0, 0, 0, 0, 0, 0.002785515320334262, 0, 0.003436426116838488, 0.006756756756756757, 0, 0, 0.008403361344537815, 0.005263157894736842, 0, 0, 0.0045351473922902496, 0.00909090909090909, 0.0014814814814814814, 0.008403361344537815, 0.005263157894736842, 0, 0, 0, 0, 0, 0, 0, 0, 0, 0.006535947712418301, 0, 0, 0, 0.006535947712418301, 0.010309278350515464, 0, 0.007407407407407408, 0.024390243902439025, 0, 0, 0, 0.01, 0, 0, 0.0014903129657228018, 0, 0, 0, 0, 0.0032733224222585926, 0.0035087719298245615, 0, 0, 0, 0, 0, 0, 0, 0.017241379310344827, 0.004901960784313725, 0.006060606060606061, 0.012422360248447204, 0, 0.006024096385542169, 0.008620689655172414, 0.008547008547008548, 0, 0.005025125628140704, 0, 0, 0.013333333333333334, 0, 0, 0, 0, 0, 0, 0.001447178002894356, 0.004048582995951417, 0.0048543689320388345, 0, 0, 0, 0 ]
0.002579
5
[ "Q:\n\njQuery check which data-type exists?", "\n\nDoes anyone know how to test which data-type a fieldset has using jQuery Mobile? ", " I am attempting to change the background color of the label element for radio buttons that are positioned horizontally but only change the ui-icon background color of radio buttons that are positioned vertically. ", " Just can't figure out how to determine which type of radio button my code is looking at.", "\nAny ideas would be greatly appreciated! ", "\nThis is a sample to the html code block I am using to display horizontal radio buttons. ", "\n<div data-role=\"fieldcontain\">\n <fieldset data-role=\"controlgroup\" data-type=\"horizontal\">\n <legend>* Are you a smoker?</legend> \n <input type=\"radio\" name=\"smoke\" value=\"Yes\" id=\"smokeY\"/>\n <label for=\"smokeY\" class=\"radioButton\">Yes</label>\n\n <input type=\"radio\" name=\"smoke\" value=\"No\" id=\"smokeN\"/>\n <label for=\"smokeN\" class=\"radioButton\">No</label>\n </fieldset>\n</div>\n\nThis is a sample of the html code block I am using to display vertical radio buttons\n<div data-role=\"fieldcontain\">\n <fieldset data-role=\"controlgroup\">\n <legend>* Do you have a vehicle in good working order?</legend> \n <input type=\"radio\" name=\"car\" value=\"Yes\" id=\"carY\"/>\n <label for=\"carY\" class=\"radioButton\">Yes</label>\n\n <input type=\"radio\" name=\"car\" value=\"No\" id=\"carN\"/>\n <label for=\"carN\" class=\"radioButton\">No</label>\n </fieldset>\n</div>\n\nHere is the halfway working jQuery I am using\nfor (group in radio_groups) {\nif (!", "$(\":radio[name=\" + group + \"]:checked\").length) {\n isValid = false;\n $(\"input:radio[name=\" + group + \"]\").each(function() {\n\n /// this code changes the icon background of vertical radio buttons\n $(this).next().find('.ui-icon').css('background-color', '#FF7575');\n\n //// i think i need some kind of if statement here that can identify if the radio button\n //// is horizontal or vertical\n\n $(this).next().css('background', '#FF7575');\n //// the above changes the lable background of horizontal radio buttons\n\n });\n } else {\n $(\"input:radio[name=\" + group + \"]\").each(function() {\n $(this).next().find('.ui-icon').css('background-color', '#A7E9A7');\n });\n }\n}\n\nUPDATE:\nHERE IS MY WORKING JSFiddle\n\nA:\n\nTo read a data attribute called data-type with jQuery Mobile, use the jqmData() method:\n$(selector).jqmData(\"type\");\n\nTest for \"horizontal\", as vertical is default and could be undefined.", "\n\nHere is a DEMO\n\nUPDATE:\nAs you iterate the radio buttons, you can see if they are horizontal or vertical by getting the controlgroup parent and checking its type:\n$(\"input[type='radio']\").each(function() {\n var cg = $(this).parents(\"fieldset:jqmData(role='controlgroup')\");\n var IsHoriz = cg.jqmData(\"type\") =='horizontal';\n if (IsHoriz){\n alert('horiz');\n } else {\n alert('vert');\n }\n});\n\nUpdated FIDDLE\n\n" ]
{ "pile_set_name": "StackExchange" }
[ 0, 0.012048192771084338, 0, 0, 0, 0, 0.0019550342130987292, 0.001049317943336831, 0 ]
0.001673
5